Download as pdf or txt
Download as pdf or txt
You are on page 1of 103

Byeongho Ban bban@umass.

edu

MATH 611 : Algebra Homework Solutions


:Taught by Paul Gunnells
Lastly updated : July 12, 2019

Mathematics & Statistics


University of Massachusetts, Amherst
Byeong Ho Ban

1
MATH 611 Editor : Byeongho Ban

Due date : September 14th, 2018

1.1.9. √
Let G = {a + b 2 ∈ R : a, b ∈ Q}.
(a) Prove that G is a group under addition.
(b) Prove that the nonzero elements of G are a group under multiplication.
[Rationalize the denominators to find multiplicative inverses.]
Proof. (B. Ban) .

(a) First of all, note that


√ √ √
(a + b 2) + (c + d 2) = (a + c) + (b + d) 2 ∈ G ∀a, b, c, d ∈ Q.
So G is closed under addition.
Nextly note that, for any a, b, c, d, e, f ∈ Q ,
√ h √ √ i h √ √ i √
(a + b 2) + (c + d 2) + (e + f 2) = (a + b 2) + (c + d 2) + (e + f 2).
This is guaranteed by the associativity of (R, +). So, G is associative.
Thirdly, note that, for any a, b ∈ Q,
√ √ √
(a + b 2) + 0 = a + b 2 = 0 + a + b 2.
Since 0 ∈ G, G has the identity.
Lastly, note that, for any a, b ∈ Q,
√ √
(a + b 2) + (−a − b 2) = 0.

Since (−a − b 2) ∈ G, every element of G has an inverse.
Therefore, G is a group under addition.

(b)
(i) Closed under multiplication.
√ √ √
(a + b 2)(c + d 2) = (ac + 2bd) + (ad + bc) 2 ∈ G ∀a, b, c, d ∈ Q.
(ii) Associativity.
The associativity og G is endowed by the associativity of R with usual multiplication.
√ h √ √ i h √ √ i √
(a + b 2) (c + d 2)(e + f 2) = (a + b 2)(c + d 2) (e + f 2).
(iii) Existence of Identity.
√ √ √
(a + b 2) · 1 = (a + b 2) = 1 · (a + b 2) ∀a, b ∈ Q
Note 1 ∈ G, so G has an identity.
(iv) Existence of inverse.
√ !
√ a−b 2 √ 1
(a + b 2) 2 2
= (a + b 2) √ = 1.
a − 2b (a + b 2)

a−b 2
Since a2 −2b2 ∈ G, every elements in G has an inverse.


Page 2
MATH 611 Editor : Byeongho Ban

1.1.22.
If x and g are elements of the group G, prove that |x| = |g −1 xg|. Deduce that |ab| = |ba| for all a, b ∈ G.
Proof. (B. Ban) .

Let q = |x|. And observe that


(g −1 xg)q = (g −1 xg)(g −1 xg) · · · (g −1 xg)
= g −1 x(gg −1 )x(gg −1 ) · · · (gg −1 )xg
= g −1 xx · · · xg
= g −1 xq g
= g −1 g
= 1G
−1
Thus, |g xg| ≤ q = |x|.
On the other hands, if |g −1 xg| = p, then observe that
1G = (g −1 xg)p = g −1 xp g ⇐⇒ g1G g −1 = xp ⇐⇒ xp = 1G .
So, |x| ≤ p = |g −1 xg|.
Therefore, |x| = |g −1 xg|.

When |x| = ∞, assume that |g −1 xg| = n < ∞. Then we have


1 = (g −1 xg)n = g −1 xn g =⇒ xn = gg −1 = 1 =⇒ |x| < n < ∞
which is a contradiction. Thus, if |x| = ∞, then |g −1 xg| = ∞ and so they are equal. As for when |g −1 xg| = ∞, same argument is
applied by using x = gyg −1 for some y ∈ G.
In particular, note that, for given a, b ∈ G,
|ab| = |a−1 (ab)a| = |ba|
by the argument above. 
1.1.25
Prove that if x2 = 1 for all x ∈ G then G is abelian.
Proof. (B. Ban) .

Let a, b ∈ G be given. Observe that


a(ba)b = (ab)(ab) = (ab)2 = 1G .
Thus,
ba = a2 (ba)b2 = a[a(ba)b]b = a(1G )b = ab.
Therefore, G is an abelian.


Page 3
MATH 611 Editor : Byeongho Ban

1.1.28
Let (A, ?) and (B, ♦) be groups and let A × B be their direct product (as defined in Example 6.) Verify all the group axioms for
A×B :
(a) prove that the associative law holds:
for all (ai , bi ) ∈ A × B, i = 1, 2, 3,

(a1 , b1 )[(a2 , b2 )(a3 , b3 )] = [(a1 , b1 )(a2 , b2 )](a3 , b3 ),


(b) prove that (1A , 1B ) is the identity of A × B, and
(c) prove that the inverse of (a, b) is (a−1 , b−1 ).
Proof. (B. Ban) .
(a) Let (ai , bi ) ∈ A × B for i = 1, 2, 3 be given. And observe that
(a1 , b1 )[(a2 , b2 )(a3 , b3 )] = (a1 , b1 )[(a2 a3 , b2 b3 )]
= (a1 (a2 a3 ), b1 (b2 b3 ))
= ((a1 a2 )a3 , (b1 b2 )b3 )
= (a1 a2 , b1 b2 )(a3 , b3 )
= [(a1 , b1 )(a2 , b2 )](a3 , b3 ).
(b)
Let (a, b) ∈ A × B be given. Observe that
(a, b)(1A , 1B ) = (a1A , b1B ) = (a, b) = (1A a, 1B b) = (1A , 1B )(a, b).
Therefore, (1A , 1B ) is the identity of A × B.
(c)
Observe that

(a, b)(a−1 , b−1 ) = (aa−1 , bb−1 ) = (1A , 1B ) = (a−1 a, b−1 b) = (a−1 , b−1 )(a, b).
Therefore, (a−1 , b−1 ) is the inverse of (a, b).

1.2.3
D2n = r, s : rn = s2 = 1, rs = sr−1
Use the generators and relations above to show that every element of D2n which is not a power of r has order 2. Deduce that
D2n is generated by the two elements s and sr, both or which have order 2.
Proof. (B. Ban) .

Let srm ∈ D2n for 1 ≤ m < n be given. And observe that by the relation given above,
(srm )2 = srm srm = ssr−m rm = (ss)(r−m rm ) = 1.
Thus, |srm | ≤ 2. Since srm 6= 1, |srm | = 2.

Furthermore, note that s, sr ∈ D2n . On the other hands, note that


r = (ss)r = s(sr).
Therefore, s, r ∈ s, sr : rn = s2 = 1, rs = sr−1 . Thus,

s, sr : rn = s2 = 1, rs = sr−1 = D2n .

3
1.2.9 Let G be that group of rigid motions in R of tetrahedron. Show that |G| = 12.
Proof. (B. Ban) .

Think of rotating a tetrahedron fixing one vertex with opposite face staying in a plane. Then we have three different rotation.
Since one of them is equivalent to not rotating, we have two different rigid motion corresponding to each vertex. Since there are
four vertices, we have eight different rigid motion.

Now, think of two edges which are located in opposite sides each other. Then we can switch the location of the edges. Then we
have two different rigid motion corresponding to each such couple of sides. Since one of them is equivalent to not moving, we have
one for each such couple of edges. Since there are three such couples, we have three different motion of switching the edges.
Then by adding the motion of not moving, we have totally, 8 + 3 + 1 = 12 different rigid motions.

Page 4
MATH 611 Editor : Byeongho Ban

1.2.10 Let G be the group of rigid motions in R3 of a cube. Show that |G| = 24.
Proof. (B. Ban) .

Note that there are Six faces and there are Four vertices in each of Six faces. So, note that we can locate a face to the top of the
cube, rotate the face and make Four different arrangement. Since we can to same thing to each of Six faces, there are 6 × 4 = 24
different arrangements. Therefore, |G| = 24. 
1.3.2
Let σ be the permutation
1 7→ 13 2 7→ 2 3 7→ 15 4 7→ 14 5 7→ 10
6 7→ 6 7 7→ 12 8 →
7 3 9 7→ 4 10 →7 1
11 7→ 7 12 7→ 9 13 →
7 5 14 7→ 11 15 →7 8
and let τ be the permutation
1 7→ 14 2 7→ 9 3 7→ 10 4 7→ 2 5 7→ 12
6 7→ 6 7 7→ 5 8 →7 11 9 7→ 15 10 7→ 3
11 7→ 8 12 7→ 7 13 →
7 4 14 7→ 1 15 7→ 13
Find the cycle decompositions of the following permutations : σ, τ, σ 2 , στ, τ σ and τ 2 σ.
Proof. (B. Ban) .

(a)
σ = (1 13 5 10)(3 15 8)(4 14 11 7 12 9)
τ = (1 14)(2 9 15 13 4)(3 10)(5 12 7)(8 11)
σ 2 = (1 5)(3 8 15)(4 11 12)(7 9 14)(10 13)
στ = (1 11 3)(2 4)(5 9 8 7 10 15)(13 14)
τ σ = (1 4)(2 9)(3 13 12 15 11 5)(8 10 14)
τ 2 σ = (1 2 15 8 3 4 14 11 12 13 7 5 10)

1.3.6
Write out the cycle decomposition of each element of order 4 in S4 .
Proof. (B. Ban) .

In S4 , the elements of order 4 are only 4−cycles. So, we have


(1 2 3 4), (1 3 4 2), (1 2 4 3), (1 4 2 3), (1 3 2 4), (1 4 3 2).
We cannot decompose more than this. 

Page 5
MATH 611 Editor : Byeongho Ban

1.3.10
Prove that if σ is the m−cycle (a1 a2 . . . am ), then for all i ∈ {1, 2, . . . , m}, σ i (ak ) = ak+i where k + i is replaced by its least
positive residue mod m. Deduce that |σ| = m.
Proof. (B. Ban) .

Suppose that σ = (a1 a2 . . . am ). Observe that

σ m−k (ak ) = am , σ(am ) = a1 .


And further note that

σ i (ak ) = ak if i = nm ∀n ∈ Z.
Thus,

σ i (ak ) = aj
, where j = k + i mod(m).
Since

σ m (ak ) = ak ∀k ∈ {1, 2, . . . , m},


|σ| ≤ m.
But since

σ i (am ) = ai 6= am ∀i < m,
|σ| = m.

1.3.15
Prove that the order of an element in Sn equals the least common multiple of the lengths of the cycles in its cycle decomposition.[Use
Exercise 10 and Exercise 24 of Section 1.1]
Proof. (B. Ban) .

Let σ ∈ Sn be given and let |σ| = m. Suppose the cycle decomposition of σ is

σ = σ1 σ2 · · · σn .
Since σi ∀i ∈ {1, 2, . . . , n} are disjoint, by Exercise 24 of Section 1.1,

σ m = σ1m σ2m · · · σnm = 1.


Thus, σim = 1 ∀i ∈ {1, 2, . . . , n} which means that |σi ||m ∀i so lcm(|σ1 |, |σ2 |, . . . , |σn |) ≤ m.
Conversely, suppose that u = lcm1≤i≤n (|σi |). Then, since |σi ||u ∀i, observe that

σ u = σ1u σ2u · · · σnu = 11 · · · 1 = 1.


Thus, m ≤ u so m = u.


Page 6
MATH 611 Editor : Byeongho Ban

1.4.10   
a b
Let G = : a, b, c ∈ R, a 6= 0, c 6= 0 .
0 c
   
a b a b
(a) Compute the product of 1 1 and 2 2 to show that G is closed under matrix multiplication.
0 c1 0 c2
 
a b
(b) Find the matrix inverse of and deduce that G is closed under inverses.
0 c
(c) Deduce that G is a subgroup of GL2 (R) (cf. Exercise 26, Section 1).
(d) Prove that the set of elements of G whose two diagonal entries are equal (i.e. a = c) is also a subgroup of GL2 (R).
Proof. (B. Ban)
(a) Observe that
    
a1 b1 a2 b2 a a a1 b2 + b1 c2
= 1 2 .
0 c1 0 c2 0 c1 c2
Since a1 , a2 , a1 b2 + b1 c2 , c1 c2 ∈ R, a1 a2 6= 0 and c1 c2 6= 0 (since a1 , a2 , c1 , c2 ∈ R \ {0}), G is closed under multiplication.
(b)
From linear algebra, observe that
 −1     1 −b

−1 a b 1 c −b 1 c −b
A = = = = a ac
1
0 c det(A) 0 a ac 0 a 0 c
Since a 6= 0 and c 6= 0, A−1 ∈ G. Thus, G is closed under inverse.
(c)
Observe that
         
a b 1 0 a b 1 0 a b a b
= = ∀ ∈ G,
0 c 0 1 0 c 0 1 0 c 0 c
and that
 
1 0
∈ G.
0 1
Thus, G has an identity. Since G satisfies associativity from the associativity of matrix multiplication, G is a group by (a) and
(b). In particular, since
   
a b a b
det = ac 6= 0 ∀ ∈ G.
0 c 0 c
Thus, G ≤ GLn (R).
(d)   
a b
Let H = : a, b ∈ R, a 6= 0 . Note that H ⊂ G.
0 a
And observe that followings.
(1) H is associative by the associativity of G.
(2) H has an identity same with G due to (c).
(3) H is closed under identity. We can observe is by simply replacing c in (b) by a.
(4) H is closed under multiplication. We can observe it by simply replacing c1 and c2 by a1 and a2 respectively in (a).

Therefore, H ≤ G ≤ GLn (R).




Page 7
MATH 611 Editor : Byeongho Ban

1.6.13
Let G and H be groups and let ϕ : G → H be a homomorphism. Prove that the image of ϕ, ϕ(G) is a subgroup of H(cf. Exercise
26 of Section 1). Prove that if ϕ is injective then G ∼
= ϕ(G).
Proof. (B. Ban) .

Let a, b ∈ G be given. Then note that


ϕ(a)ϕ(b) = ϕ(ab) ∈ ϕ(G).
So, ϕ(G) is closed under the binary operation.
Also, when 1 ∈ G is an identity,

ϕ(1)ϕ(a) = ϕ(1a) = ϕ(a) = ϕ(a1) = ϕ(a)ϕ(1) ∀a ∈ G.


So, ϕ(1) ∈ ϕ(G) is an identity in H.
Lastly, observe that
ϕ(a)ϕ(a−1 ) = ϕ(aa−1 ) = ϕ(1) = ϕ(a−1 a) = ϕ(a−1 )ϕ(a).
So, ϕ(a−1 ) is the inverse of ϕ(a) for each a ∈ G.
Since ϕ(G) is associative by H, ϕ(G) is a group.
In particular, if ϕ is injective, note that ϕ : G → ϕ(G) is bijective. Therefore, ϕ is an isomorphism between G and ϕ(G), so
G= ∼ ϕ(G).

1.6.14
Let G and H be groups and let ϕ : G → H be a homomorphism. Define the kernel of ϕ to be {g ∈ G : ϕ(g) = 1H }(so the kernel
is the set of elements in G which map to the identity of H , i.e, is the fiber over the identity of H.) Prove that the kernel of ϕ is
a subgroup of G. Prove that ϕ is injective if and only if the kernel of ϕ is the identity subgroup of G.
Proof. (B. Ban) .

Claim 1 . ker(ϕ) ≤ G.
proof.
(1) ker(ϕ) is closed under the operation of G. Note that, for any a, b ∈ ker(ϕ)
ϕ(ab) = ϕ(a)ϕ(b) = 1H 1H = 1H =⇒ ab ∈ ker(ϕ)
(2) ker(ϕ) has an identity. Observe that
ϕ(1G )ϕ(a) = ϕ(1G a) = ϕ(a) = ϕ(a1G ) = ϕ(a)ϕ(1G ) ∀a ∈ G.
Thus, ϕ(1G ) is the identity in H. Thus, 1G ∈ ker(ϕ).

(3) ker(ϕ) is closed under inverse. Observe that, for any a ∈ ker(ϕ)
1H = ϕ(1G ) = ϕ(aa−1 ) = ϕ(a)ϕ(a−1 ) = 1H ϕ(a−1 ) = ϕ(a−1 ).
Therefore, a−1 ∈ ker(ϕ) and ker(ϕ) is closed under inverse.

Claim 2. ϕ is injective ⇐⇒ the kernel of ϕ is the identity subgroup of G


proof.
We will use contrapositive for one direction. Assume that ∃a ∈ ker(ϕ) with a 6= 1G . Then, observe that
ϕ(a) = 1H = ϕ(1G )
even if a 6= 1. Thus, ϕ is not injective.
Conversely, suppose that ker(ϕ) = {1}. Then observe that, ∀a, b ∈ G,
ϕ(a) = ϕ(b) ⇐⇒ ϕ(ab−1 ) = 1H =⇒ ab−1 = 1G =⇒ a = b.
Thus, ϕ is injective. 

Page 8
MATH 611 Editor : Byeongho Ban

1.6.18
Let G be any group. Prove that the map from G to itself defined by g 7→ g 2 is a homomorphism if and only if G is abelian.
Proof. (B. Ban) .

Suppose that the map ϕ : G → G defined by above is homomorphism. And let a, b ∈ G be given. Then observe that
abab = ϕ(ab) = ϕ(a)ϕ(b) = a2 b2 = aabb.
Further observe that
ab = a−1 (aabb)b−1 = a−1 (abab)b−1 = ba.
Therefore, G is abelian.
Conversely, suppose that G is abelian. And let a, b ∈ G be given. Then, observe that
ϕ(ab) = (ab)(ab) = a(ba)b = a(ab)b = a2 b2 = ϕ(a)ϕ(b).
Therefore, ϕ is a homomorphism.

1.6.20
Let G be a group and let Aut(G) be the set of all isomorphisms from G onto G. Prove that Aut(G) is a group under function
composition (called the automorphism group of G and the elements of Aut(G) are called automorphisms of G).
Proof. (B. Ban) .

(1) Aut(G) is closed under function composition.


Let f, g ∈ Aut(G) be given. Then observe that, for any a, b ∈ G,
f ◦ g(ab) = f (g(ab)) = f (g(a)g(b)) = f (g(a))f (g(b)) = f ◦ g(a)f ◦ g(b).
Since composition of bijections is again bijection, f ◦ g ∈ Aut(G). Thus, Aut(G) is closed under function composition.

(2) Associativity of Aut(G) is endowed by the associativity of function composition.

(3) Note that the identity map defined by I(x) = x is in Aut(G). Observe that
f ◦ I(x) = f (x) = I(f (x)) = I ◦ f (x) ∀x ∈ G.
Thus, Aut(G) has an identity, I.

(4) Let f ∈ Aut(G) be given. Note that, since f is a bijection, f −1 exists. Observe that, for given a, b ∈ G,
ab = f (f −1 (a))f (f −1 (b)) = f (f −1 (a)f −1 (b)) =⇒ f −1 (ab) = f −1 (a)f −1 (b).
Therefore, f −1 ∈ Aut(G) which means that Aut(G) is closed under inverse.
Based on the arguments above, Aut(G) is a group.

1.6.23
Let G be a finite group which possesses an automorphism σ such that σ(g) = g if and only if g = 1. If σ 2 is the identity map
from G to G, prove that G is abelian (such an automorphism σ is called fixed point free of order 2). [Show that every element of
G can be written in the form x−1 σ(x) and apply σ to such an expression.]
Proof. (B. Ban) .

Let H = {g −1 σ(g) : g ∈ G}, then note that H ⊂ G. And let ϕ : G → H defined by ϕ(g) = g −1 σ(g). Observe that
ϕ(x) = ϕ(y) ⇐⇒ x−1 σ(x) = y −1 σ(y)
⇐⇒ σ(yx−1 ) = yx−1
=⇒ yx−1 = 1
=⇒ x = y.
Thus, ϕ is injective. Then since G is finite and |G| ≤ |H| by injectivity, G = H. Therefore, ∀g ∈ G ∃h ∈ G such that g = h−1 σ(h).
Further, letting g = h−1 σ(h) be given, observe that
−1
σ(g) = σ(hσ(h)) = σ(h−1 )σ 2 (h) = σ(h−1 )h = h−1 σ(h) = g −1 .
It implies that, for any g, h ∈ G,
gh = σ((gh)−1 ) = σ(h−1 g −1 ) = σ(h−1 )σ(g −1 ) = hg.
Therefore, G is an abelian group.


Page 9
MATH 611 Editor : Byeongho Ban

2.2.6
Let H be a subgroup of the group G.
(a) Show that H ≤ NG (H). Give an example to show that this is not necessarily true if H is not a subgroup.
(b) Show that H ≤ CG (H) if and only if H is abelian.
Proof. (B. Ban) .

(a)
Firstly, observe that NG (H) is a group.
(1) NG (H) is closed under the binary operation.
Let a, b ∈ NG (H) and h ∈ H be given. Then note that

(ab)h(ab)−1 = a(bhb−1 )a−1 ∈ H,


since bhb−1 ∈ H and aga−1 ∈ H for all g ∈ H.
Therefore, ab ∈ NG (H).

(2) NG (H) has an identity.


Let 1G ∈ G be the identity of G. Then observe that, for any given h ∈ H,
1G h1−1
G = h ∈ H.
Thus, 1G ∈ NG (H) and it is the identity for it.

(3) The associativity is endowed by G.

(4) NG (H) is closed under inverse.


Let n ∈ NG (H) and h ∈ H be given. Then observe that
nhn−1 = h0 for some h0 ∈ H ⇐⇒ n−1 h0 n = h ∈ H.
Since ϕ : H → H defined by ϕ(h) = nhn−1 is bijective, we can say that for given h ∈ H, there exists h0 ∈ H such that
nh0 n−1 = h.
Thus, n−1 ∈ NG (H), so it is closed under inverse. Therefore, NG (H) is a subgroup of G. Since it is clear that H ⊂ NG (H), it is
true that H ≤ NG (H).
For the example, consider H = {1, (1 2), (2 3)} ⊂ S4 . And observe that
(2 3)H = {(2 3), (1 3 2), 1} =
6 {(2 3), (1 2 3), 1} = H(2 3).
Thus, H is not a subgroup of NG (H).

(b)
Suppose that H ≤ CG (H). Then note that
hg = gh ∀g ∈ CG (H)∀h ∈ H.
Since H ≤ CG (H), indeed, gh = hg ∀h, g ∈ H. Thus, H is abelian.
Conversely, suppose that h ∈ H and H is abelian. Then, obviously,
gh = hg ∀g ∈ H.
Thus, g ∈ CG (H) and H ≤ CG (H).

2.2.10
Let H be a subgroup of order 2 in G. Show that NG (H) = CG (H). Deduce that if NG (H) = G then H ≤ Z(G).
Proof. (B. Ban) .

Since H is a subgroup of order 2, we can say H = {1, h} where h2 = 1. Observe that


g ∈ NG (H) ⇐⇒ {g, gh} = {g, hg} ⇐⇒ gh = hg ⇐⇒ g ∈ CG (H).
Thus, NG (H) = CG (H).
Further suppose that NG (H) = G, then by the result above, CG (H) = G. It implies that hg = gh ∀g ∈ G. Thus, h ∈ Z(G). Since
1 ∈ Z(G) is obvious, H ≤ Z(G).


Page 10
MATH 611 Editor : Byeongho Ban

Due date : September 21st, 2018


2.3.24
Let G be a finite group and let x ∈ G.
(a) Prove that if g ∈ NG (hxi) then gxg −1 = xa for some a ∈ Z.
(b) Prove conversely that if gxg −1 = xa for some a ∈ Z, then g ∈ NG (hxi). [Show first that gxk g −1 = (gxg −1 )k = xak for
any integer k, so that g hxi g −1 ≤ hxi. If x has order n, show the elements gxi g −1 , i = 0, 1, . . . , n − 1 are distinct, so that
|g hxi g −1 | = | hxi | = n and conclude that g hxi g −1 = hxi.]
Proof. (B. Ban) .

(a)
Suppose that g ∈ NG (hxi). Then
gxg −1 ∈ gyg −1 : y ∈ hxi = g hxi g −1 = hxi = {xa : a ∈ Z}.


Thus, ∃a ∈ Z such that gxg −1 = xa for some a ∈ Z.

(b)
Let k ∈ Z be given. And suppose that gxg −1 = xa for some a ∈ Z. Then observe that
xak = (gxg −1 )k = (gxg −1 )(gxg −1 ) · · · (gxg −1 ) = gx(g −1 g)x(g −1 g)x(g −1 g) · · · (g −1 g)xg −1 = gxk g −1 .
Thus, ∀gxk g −1 ∈ g hxi g −1 , there is a ∈ Z such that gxk g −1 = xak ∈ hxi. Thus, g hxi g −1 ≤ hxi.

Now, note that ∃n ∈ N such that |x| = n since G is finite group and hxi ≤ G. Assume that ∃0 ≤ i < j ≤ n − 1 such that
gxi g −1 = gxj g −1 . Now, observe that

gxi g −1 = gxj g −1 ⇐⇒ xi = xj ⇐⇒ xj−i = 1.


and it is a contradiction, so gxi g −1 are distinct for all i = 0, 1, . . . , n − 1 which means that
|g hxi g −1 | = | hxi | = n
And this means that g hxi g −1 = hxi.
Therefore, g ∈ NG (hxi).


Page 11
MATH 611 Editor : Byeongho Ban

2.4.10    
0 −1 1 1
Prove that the subgroup of SL2 (F3 ) generated by and is isomorphic to the quaternion group of order 8. [Use
1 0 1 −1
a presentation for Q8 .]
Proof. (B. Ban) .

Let H be the subgroup of SL2 (F3 ) generated by the given two matrices.
Note that the presentation of Q8 is

Q8 = i, j| i2 = j 2 = k 2 = ijk = −1, (−1)2 = 1 .


Let’s define ϕ : Q8 → H as follows:
       
0 −1 1 1 −1 1 1 0
ϕ(±i) = ± ϕ(±j) = ± ϕ(±k) = ± ϕ(±1) = ± .
1 0 1 −1 1 1 0 1
In order to prove that ϕ is the isomorphism, it suffices to verify that
ϕ(i)2 = ϕ(j)2 = ϕ(k)2 = ϕ(i)ϕ(j)ϕ(k) = ϕ(−1) and ϕ(−1)2 = ϕ(1).
Observe that
 2  
2 0 −1 −1 0
ϕ(i) = = = ϕ(−1)
1 0 0 −1
 2    
1 1 2 0 −1 0
ϕ(j)2 = = = = ϕ(−1)
1 −1 0 2 0 −1
 2    
2 −1 1 2 0 −1 0
ϕ(k) = = = = ϕ(−1)
1 1 0 2 0 −1
          
0 −1 1 1 −1 1 −1 1 −1 1 2 0 −1 0
ϕ(i)ϕ(j)ϕ(k) = = = = = ϕ(−1)
1 0 1 −1 1 1 1 1 1 1 0 2 0 −1
 2  
−1 0 1 0
ϕ(−1)2 = = = ϕ(1).
0 1 0 1
Therefore, H and Q8 are isomorphic.


Page 12
MATH 611 Editor : Byeongho Ban

2.4.11
Show that SL2 (F3 ) and S4 are two nonisomorphic groups of order 24.
Proof. (B. Ban) .

 {−1,0, 1}. Let’s determine the order of SL2 (F3 ).


Note that F3 =
a b
Suppose that ∈ SL2 (F3 ). Then
c d
ad − cb = 1.
If a is 0, it should be c = −b = 1 or c = −b = −1 and d can be anything. So there are 3 × 2 = 6 possible cases when a = 0. We
can switch a to d and get another 6 ways. So we have 12 cases. But since we are counting when a = d = 0 twice, we subtract 2
cases. Therefore, there are totally 12 − 2 = 10 cases when a or d is zero.

If c = 0, it should be a = d = 1 or a = b = −1 and b can be anything . Thus, similarly, there are 6 cases and there are other 6
ways when we switch c to b. But since we are counting when c = b = 0 twice, we subtract 2 cases. Therefore, there are totally
12 − 2 = 10 cases when c or b is zero.

If none of a, b, c and d is zero, then it should be ad = −1 = −cb. There are only four possible cases when one of a = −d = 1 and
a = −d = −1 is true and one of c = −b = 1 and c = −b = −1 is true.
Therefore, |SL2 (F3 )| = 24.

Even if |SL2 (F3 )| = |S4 | = 24, they are not isomorphic since |C(S4 )| = |{1}| = 1 but
   
1 0 −1 0
, ∈ C(SL2 (F3 )).
0 1 0 −1

2.4.19
A nontrivial abelian group A (written multiplicatively) is called divisible if for each element a ∈ A and each nonzero integer k
there is an element x ∈ A such that xk = a, i.e, each element has a k th root in A (in additive notation, each element is the k th
multiple of some element of A.)
(a) Prove that the additive group of rational numbers, Q, is divisible.
(b) Prove that no finite abelian group is divisible.
Proof. (B. Ban) .

n n
(a) Let ∈ Q where gcd(n, m) = 1 and k ∈ Z be given. And note that
m km ∈ Q and observe that
n n
k = .
km m
Therefore, Q is divisible.

(b)
Suppose that G is a finite abelian group that is divisible. Since G is non trivial, ∃a ∈ G \ {1}. Then there exists x ∈ G such that
x|G| = a. However, since G is finite, x|G| = 1 6= a and it is a contradiction. Therefore, any finite abelian group is not divisible.


Page 13
MATH 611 Editor : Byeongho Ban

3.1.4
Prove that in the quotient group G/N , (gN )α = g α N for all α ∈ Z.
Proof. (B. Ban) .

Note that (gN )0 should be an identity of G/N which is N . Further note that (gN )1 = gN . Suppose that (gN )n = g n N for some
n ∈ Z+ . Then observe that
(gN )n+1 = (gN )(gN )n = (gN )(g n N ) = g n+1 N.
Thus, (gN )n = g n N for all n ∈ Z+ ∪ {0} by induction principle.

Now, recall that (gN )−1 = g −1 N . Suppose that


(gN )−n = g −n N.
Then, observe that
(gN )−(n+1) = (gN )−1 (gN )−n = (g −1 N )(g −n N ) = g −(n+1) N.
It means that (gN )−n = g −n N ∀n ∈ Z+ by induction principle.
Therefore, (gN )n = g n N ∀n ∈ Z.

3.1.5
Use the preceding exercise to prove that the order of the element gN in G/N is n, where n is the smallest positive integer such
that g n ∈ N (and gN has infinite order if no such positive integer exists.) Give an example to show that the order of gN in G/N
may be strictly smaller than the order of g in G.
Proof. (B. Ban) .

Let |gN | = m. Now observe that, by previous exercise,


(gN )n = g n N = N
since g n ∈ N . It tells us that m ≤ n.
Conversely, observe that
g m N = (gN )m = N
which means that g m ∈ N . It says that n ≤ m. Therefore, n = m.

Suppose that no such n exists. And assume that ∃m ∈ Z+ such that |gN | = m. But then
g m N = (gN )m = N
which means that g m ∈ N and it is a contradiction.

Consider G/N = Z/4Z. Note that |2| = 2 but |2| = ∞. Thus,


|2 + 0| = |2| = 2 < ∞ = |2|.


Page 14
MATH 611 Editor : Byeongho Ban

3.1.9
Define ϕ : CX → RX by ϕ(a + ib) = a2 + b2 . Prove that ϕ is a homomorphism and find the image of ϕ. Describe the kernel and
the fibers of ϕ geometrically (as subsers of the plane.)
Proof. (B. Ban) .

Let a + ib, c + id ∈ CX be given. And observe that


ϕ((a + ib)(c + id)) = ϕ((ac − bd) + i(ad + bc))
= (ac − bd)2 + (ad + bc)2
= a2 c2 − 2abcd + b2 d2 + a2 d2 + 2abcd + b2 c2
= a2 (c2 + d2 ) + b2 (d2 + c2 )
= (a2 + b2 )(c2 + d2 )
= ϕ(a + ib)ϕ(c + id).
Therefore, ϕ is a homomorphism. And note that
ϕ(C X ) = a2 + b2 : a + ib ∈ CX = (0, ∞).


Observe that
kerϕ = {a + ib ∈ CX : a2 + b2 = 1} = {cos t + i sin t : t ∈ R}.
Thus, the kernel of ϕ is a unit circle in complex plane and the fiber of any r ∈ (0, ∞) is a circle centered at 0 with radius r. 

Page 15
MATH 611 Editor : Byeongho Ban

3.1.11
Let F be a field and let
  
a b
G= : a, b, c ∈ F, ac 6= 0 ≤ GL2 (F ).
0 c

 
a b
(a) Prove that the map ϕ : 7→ a is a surjective homomorphism from G onto F × (recall that F × is the multiplicative group
0 c
of nonzero elements in F .) Describe the fibers and kernel of ϕ.

(b) Prove that the map


 
a b
ψ: 7 (a, c)

0 c
is a surjective homomorphism from G onto F × × F × . Describe the fibers and kernel of ψ.

(c) Let
  
1 b
H= :b∈F .
0 1
Prove that H is isomorphic to the additive group F .
Proof. (B. Ban) .

(a)    
a b1 a b2
Let 1 , 2 ∈ G be given. Observe that
0 c1 0 c2
  
 
a1 b1 a2 b2 a1 a2 a1 b2 + b1 c2
ϕ =ϕ
0 c1 0 c2 0 c1 c2
= a1 a2
   
a 1 b1 a2 b2
=ϕ ϕ .
0 c1 0 c2
 
a 0
Thus, ϕ is a homomorphism. Also, for given a ∈ F × , we can choose c ∈ F × such that ∈ G.
0 c
Observe that
 
a 0
ϕ = a.
0 c
Thus, ϕ is a surjection onto F × which means that ϕ(G) = F × . And note that
     
a b 1 b
kerϕ = ∈ G : a = 1, b ∈ F = : b, c ∈ F, c 6= 0 .
0 c 0 c
And for each a ∈ F × ,
  
−1 a b
ϕ ({a}) = : b, c ∈ F, c 6= 0 .
0 c


Page 16
MATH 611 Editor : Byeongho Ban

(b)
   
a1 b1 a b2
Let , 2 ∈ G be given. Observe that
0 c1 0 c2
    
a1 b1 a2 b2 a1 a2 a1 b2 + b1 c2
ψ =ψ
0 c1 0 c2 0 c1 c2
= (a1 a2 , c1 c2 )
= (a1 , c1 )(a2 , c2 )
   
a1 b1 a2 b2
=ψ ψ .
0 c1 0 c2
Thus, ψ is a homomorphism. Also, for given (a, c) ∈ F × × F × , note that
   
a 0 a 0
∈G and ψ = (a, c).
0 c 0 c
Thus, ψ is a surjection onto F × × F × . Observe that
     
a b 1 b
kerψ = ∈ G : (a, c) = (1, 1), b ∈ F = :b∈F .
0 c 0 1
and that, for given (a, c) ∈ F × × F × ,
  
−1 a b
ψ ({(a, c)}) = :b∈F
0 c

(c) Let f : H → F defined by


 
1 b
f = b.
0 1
Observe that, for any b1 , b2 ∈ F ,
        
1 b1 1 b2 1 b1 + b2 1 b1 1 b2
f =f = b1 + b2 = f ( )+f .
0 1 0 1 0 1 0 1 0 1
Thus, f is a homomorphism. And further observe that
   
1 b1 1 b2
f =f ⇐⇒ b1 = b2
0 1 0 1
and that, for any b ∈ F ,
 
1 b
f = b.
0 1
Therefore, f is a bijection, thus f is isomorphism between H and F which means that H and F are isomorphic.

Page 17
MATH 611 Editor : Byeongho Ban

3.2.9
This exercise outlines a proof of Cauchy’s theorem due to James McKay (Another proof of Cauchy’s group theorem, Amr. Math.
Monthly, 66(1959)).

Let G be a finite group and let p be a prime dividing |G|. Let S denote the set of p−tuples of elements of G the product of whose
coordinates is 1:
S = {(x1 , x2 , . . . , xp ) : xi ∈ G and x1 x2 . . . xp = 1}.
p−1
(a) Show that S has |G| elements, hence has order divisible by p.

Define the relation ∼ on S by letting α ∼ β if β is a cyclic permutation of α.

(b) Show that a cyclic permutation of an element of S is again an element of S.


(c) Prove that ∼ is an equivalence relation on S.
(d) Prove that an equivalence class contains a single element if and only if it is of the form (x, x, . . . , x) with xp = 1.
(e) Prove that every equivalence class has order 1 or p (this uses the fact that p is a prime.) Deduce that |G|p−1 = k + pd, where
k is the number of classes of size 1 and d is the number of classes of size p.
(f ) Since {(1, 1, . . . , 1)} is an equivalence class of size 1, conclude from (e) that there must be a nonidentity element x in G with
xp = 1, i.e, G contains an element of order p. [Show p|k and so k > 1.]
Proof. (B. Ban) .

(a) Let xi ∈ G for i = 1, 2, . . . , p − 1 be given. Then


x1 x2 · · · xp−1 = y for some y ∈ G.
−1
Since G is a group, we have y ∈ G. Then note that
x1 x2 · · · xp−1 y −1 = 1.
Note that there was no restriction for choosing xi for i = 1, 2, . . . , p − 1 and the last element xp = y −1 was uniquely determined
by previous p − 1 elements. Therefore, we have |G|p−1 ways to choose an element in S. It means that
|S| = |G|p−1 .
In particular, observe that p||G| =⇒ p||G|p−1 =⇒ p||S|.

(b)
Cyclic permutation of any α ∈ S is just a rearrangement of xi ’s of α, so it preserves the product. It means that the cyclic
permutation of any element in S is in S.

(c)
Since α ∈ S is a identity cyclic permutation of itself, α ∼ α.
If α ∼ β, β is a cyclic permutation of α. Then there is a cycle σ that maps α to β. Since σ is invertible, σ −1 maps β to α. Thus,
α is a cyclic permutation of β which means that β ∼ α.
Suppose that α ∼ β and β ∼ γ. Then there are two cycles σ and η which maps α to β and β to γ respectively. Then ησ is a cycle
maps α to γ. It means that α ∼ γ.
Thus, ∼ is an equivalence relation.

Page 18
MATH 611 Editor : Byeongho Ban

(d)
Suppose that [α] is the equivalence class that contains a single element and let α = (x1 , x2 , . . . , xp ). Then observe that , for any
p−cycle, σ,
σ · α = α.
and it means that xi = xj ∀i, j ∈ {1, 2, . . . , p}. Since i and j are arbitrary, ∃x ∈ G such that xi = x ∀i ∈ {1, 2, . . . , p}. Then
α = (x, x, . . . , x)
p
and x = 1 by (b)and the definition of S.

Conversely, suppose that α = (x, x, . . . , x) with xp = 1. Then α ∈ S and for arbitrary cycle σ, σ · α = α. Therefore, the
equivalence class [α] contains only one element.

(e)
By (d), there are equivalence classes of size 1.
If there are two distinct element in one class, since each p−cycle makes p different elements, the equivalence class of size bigger
than 1 should be of size of p. Therefore, the size of each equivalence class is 1 or p.
Furthermore, since equivalence classes partition S,
|G|p−1 = |S| = k + pd
where k is the number of the classes of size 1 and d is the number of the classes of size p.

(f )
Note that since {(1, 1, . . . , 1)} is the equivalence class of size of 1, k =
6 0. Since p||G| and p|pd, from (e), p should divide k. And
it means that there is another equivalence class of size 1. Thus, ∃x ∈ G \ {1} such that {(x, x, . . . , x)} is one of the equivalence
class of size 1. But, by definition of S, xp = 1.
3.4.4
Use Cauchy’s Theorem and induction to show that a finite abelian group has a subgroup of order n for each positive divisor n of
its order.
Proof. (B. Ban) .

Note that, when |G| = 1, the statement is trivially true since 1 has only one divisor which is itself and G is a subgroup of itself.
Now, suppose that for any finite abelian group of order less than to m, the given statement is true. Let G be a finite abelian group
with |G| = m. Suppose that n|m. And let p be a prime number which divides n. Observe that, by Cauchy’s theorem, ∃x ∈ G
such that xp = 1 and that G/ hxi is a group of order m + n + m n
p ∈ Z . Note that p ∈ Z divides p and p < m. Therefore, by induction
assumption, G/ hxi has a subgroup of order np which can be presented by H/ hxi with H ≤ G. Since |H| = n, we are done.

3.5.7
Prove that the group of rigid motions of a tetrahedron is isomorphic to A4 .
Proof. (B. Ban) .

Let’s index each vertices of a tetrahedron from 1 to 4. And note that the group of rigid motions of the tetrahedron is isomorphic
to G below.
G = {1, (1 2 3), (2 3 4), (1 3 4), (1 2 4), (3 2 1), (4 3 2), (4 3 1), (4 2 1), (1 2)(3 4), (1 4)(2 3), (1 3)(2 4)}
Observe that the sign of each element of G is 1 which means that G ⊂ A4 . Since |G| = 12, G = A4 . Therefore, the group of rigid
motions of a tetrahedron is isomorphic to A4 . 

Page 19
MATH 611 Editor : Byeongho Ban

3.5.14
Prove that the subgroup of A4 generated by any element of order 2 and any element of order 3 is all of A4 .
Proof. (B. Ban) (5/21/2019)

Claim. A4 does not have order 6 subgroup.


( =⇒ ) Suppose that H ≤ A4 is of order 6. Then, recalling that the set of left cosets, {H, gH} and that of right cosets, {H, Hg}
are two partitions of A4 so gH = Hg. Then observe that, for some h0 , h00 ∈ H
h ∈ gHg −1 ⇐⇒ h = gh0 g −1 ⇐⇒ h = h00 gg −1 ∈ H
since gh0 ∈ gH = Hg. Therefore, gHg −1 = H so H E A4 . Now, let an order 3 element g in A4 be given. Then observe that the
order of g = gH divides |g| = 3 and |A4 /H| = 2 at the same time. However, since gcd(|A4 /H|, |g|) = gcd(2, 3) = 1, |g| = 1 which
implies that g ∈ H. Therefore, all order 3 elements in A4 are in H. Now, note that there are 4!3 = 8 of order 3 elements in A4 ,
|H| > 8 which is a contradiction. Therefore, there does not exist such H.

Now, let σ, τ ∈ A4 with |σ| = 3 and |τ | = 2 be given. Then letting H := hσ, τ i, we have 3|H and 2|H, thus 6|H. So possible |H|
is 6 and 12. However, as we argued above, there does not exist order 6 subgroup in A4 so only possible |H| is 12 so H = A4 .

3.5.15
Prove that if x and y are distinct 3−cycles in S4 with x 6= y −1 , then the subgroup of S4 generated by x and y is A4 .
Proof. (B. Ban) .

Let x, y ∈ S4 be given as stated in the problem. Let H = hx, yi. Note that hxi ≤ H and y, y −1 ∈ H with y, y −1 6∈ hxi. Since
| hxi | = 3, we have |H| ≥ | hxi | + 2 = 5. Since all 3−cycles are in A4 , x, y ∈ A4 , so H ≤ A4 . Since A4 has no subgroup of order
6(mentioned in the book right before the exercise section), only possible subgroup of A4 with size bigger than 4 is A4 itself by
Lagrange’s Theorem. Therefore, H = A4 .

3.5.16
Let x and y be distinct 3 − cycles in S5 with x 6= y −1 .

(a) Prove that if x and y fix a common element of {1, . . . , 5}, then hx, yi ∼
= A4 .
(b) Prove that if x and y do not fix a common element of {1, . . . , 5}, then hx, yi = A5 .
Proof. (B. Ban) (5/21/2019) revised

(a)
Suppose that x and y fix a common element of I = {1, . . . , 5}. Eliminating the commonly fixed element and relabeling them
by 1, 2, 3 and 4, we get J = {1, 2, 3, 4}. Let’s say the relabeling homomorphism ϕ. Then S4 be a set of all permutation of set
J. Then, since ϕ(x), ϕ(y) ∈ S4 are distinct order 3 elements, by 3.5.15, ϕ(hx, yi) = hϕ(x), ϕ(y)i = A4 . Since relabeling is
isormophism, ϕ is an isomorphism so hx, yi ∼ = A4 .

(b)
Note that it is clear to say hx, yi ≤ A5 since x, y ∈ A5 ( ∵ 3−cycle is even permutation).
Without loss of generality, let x = (a b c) and y = (a d e) for distinct elements a, b, c, d, e ∈ {1, 2, 3, 4, 5}. Then observe that
(a d e)−1 (a b c)(a d e) = (b c e) (a d e)−1 (b c e)(a d e) = (b c d)
(abc)−1 (bce)(abc) = (abe) (ade)−1 (abc)(ade) = (bde)
(abc)−1 (abe)(abc) = (ace)−1 (ade)−1 (bde)(ade) = (abd)−1
(abc)−1 (abd)(abc) = (acd)−1 (abe)−1 (bcd)−1 (abe) = (cde)−1
Thus, we have all 3 cycles in H := hx, yi which means 20 ≤ |H|. Also, note that (abc)(ade) = (adebc) and it is order 5 and in A5 .
Thus, 25 ≤ |H|. However, by Lagrange’s theorem, only possible subgroup of A5 of order bigger than 25 is 30 and 60. If |H| = 30,
since |G : H| = 2, H E A5 and it is a contradiction since A5 is simple. Therefore, |H| = 60 and so H = A5 . 

Page 20
MATH 611 Editor : Byeongho Ban

3.5.17
If x and y are 3−cycles in Sn , prove that hx, yi is isomorphic to Z3 , A4 , A5 or Z3 × Z3 .
Proof. (B. Ban) .

Suppose that y ∈ hxi. Then hx, yi = hxi. But it is a cyclic group of order 3. Thus, hx, yi ∼
= Z3 .

If we fix all the point except less than or equal to 3 points, then this cases is in first case.

If we fix all the points except 4 points, by previous exercise, hx, yi ∼


= A4 .

If we fix all the points except 5 points, by previous exercise (a) and (b), hx, yi ∼
= A5 .

Now suppose that x = (a b c) , y = (d e f ) and


{a, b, c} ∩ {d, e, f } = ∅.
Then , since two cycles are disjoint, so commute,
hx, yi = 1, (a b c), (a b c)2 , (d e f ), (d e f )2 , (a b c)(d e f ), (a b c)2 (d e f ), (a b c)(d e f )2 , (a b c)2 (d e f )2 .


Then we have natural morphism ψ such defined by


ψ ((a b c)n (d e f )m ) = (n, m) ∈ Z/3Z × Z/3Z.
It is a homomorphism since
ψ ((a b c)n (d e f )m ) = (n, m) = (n, 0) + (0, m) = ψ (a b c)n (d e f )0 + ψ (a b c)0 (d e f )m .
 

Also, clearly,
(n1 , m1 ) = (n2 , m2 ) ⇐⇒ (a b c)n1 (d e f )m1 = (a b c)n2 (d e f )m2 .
And for any (n, m) ∈ Z/3Z × Z/3Z,
ψ ((a b c)n (d e f )m ) = (n, m).
Thus, ψ is an isomorphism between hx, yi and Z/3Z × Z/3Z which means that
hx, yi ∼
= Z/3Z × Z/3Z ∼
= Z3 × Z3 .


Page 21
MATH 611 Editor : Byeongho Ban

HW3- Due date : October 1st, 2018 Byeongho Ban

4.1.7
Let G be a transitive permutation group on the finite set A. A block is a nonempty subset B of A such that for all σ ∈ G either
σ(B) = B or σ(B) ∩ B = ∅(here σ(B) is the set {σ(b) : b ∈ B}.)

(a) Prove that if B is a block containing the element a of A, then the set GB defined by
GB = {σ ∈ G : σ(B) = B}
is a subgroup of G containing Ga .
(b) Show that if B is a block and σ1 (B), σ2 (B), . . . , σn (B) are all the distinct images of B under the elements of G, then these
form a partition of A.
(c) A (transitive) group of G on a set A is said to be primitive if the only blocks in A are the trivial ones: the sets of size 1 and
A itself. Show that S4 is primitive on A = {1, 2, 3, 4}. Show that D8 is not primitive as a permutation group on the four vertices
of a square.
(d) Prove that the transitive group G is primitive on A if and only if for each a ∈ A, the only subgroups of G containing Ga are
Ga and G (i.e, Ga is a maximal subgroup of G)[Use part(a)].
Proof. (B. Ban) .

(a)
Note that GB ⊂ G is clear and GB 6= ∅ since 1 ∈ GB . Let x ∈ B and σ, τ ∈ GB then note that ∃x0 , x00 ∈ B such that σ(x0 ) = x
and σ −1 (x00 ) = x since σ(B) = B. Now observe that
τ σ −1 (x) = τ σ −1 (σ(x0 )) = τ (x0 ) ∈ B =⇒ τ σ −1 (B) ⊆ B.
Also, note that, since τ (B) = B, ∃x000 ∈ B such that σ −1 (x00 ) = τ σ −1 (x000 ). Thus, observe that
x = σ −1 (x00 ) = τ σ −1 (x000 ) ∈ B =⇒ B ⊆ τ σ −1 (B).
Therefore,
τ σ −1 (B) = B =⇒ τ σ −1 ∈ GB
so GB ≤ G.
Further note that
σ ∈ Ga ⊂ G =⇒ σ(a) = a ∈ B =⇒ a ∈ σ(B).
Thus, a ∈ o(B) ∩ B 6= ∅ so by definition of block, o(B) = B and it means that o ∈ GB . Thus, Ga ⊆ GB .

(b)
Let i, j ∈ {1, 2, . . . , n} with j 6= j and let x ∈ σi (B) ∩ σj (B). Then ∃x0 , x00 ∈ B such that σi (x0 ) = σj (x00 ) so σi−1 σj (x00 ) = x00 ∈ B.
Thus, σi−1 σj (B) = B . Now, observe that
σj (B) = σi (σi−1 σj (B)) = σi (B).
It is a contradiction. Therefore, σi (B) ∩ σj (B) = ∅ ∀i, j ∈ {1, 2, . . . , n}.
Secondly, let
[n
B0 = σi (B)
i=1
and let x ∈ A \ B . Since G acts transitively on A, ∃τ ∈ G and ∃x0 ∈ B such that τ (x0 ) = x ∈ τ (B). And it is a contradiction
0

since σ1 (B), σ2 (B), . . . , σn (B) are all the distinct images of B under the elements of G. Therefore, B 0 = A.
In conclusion, σ1 (B), σ2 (B), . . . , σn (B) is a partition of A.

Page 22
MATH 611 Editor : Byeongho Ban

Proof. (B. Ban) (c)


Let i, j, k ∈ {1, 2, 3, 4} be given distinct elements. And note that
(i k){i, j} = {k, j} =
6 {i, j} and (i k){i, j} ∩ {i, j} =
6 ∅,
(i j k l){i, j, k} = {j, k, l} =
6 {i, j, k} and (i j k l){i, j, k} ∩ {i, j, k} =
6 ∅,
where l ∈ A \ {i, j, k}. Thus, there is no nontrivial blocks (of size 2 or 3) in A so S4 is primitive.
On the other hand, let’s label each vertices of a square with 1, 2, 3, 4 clockwisely. Then r is rotation of the square counterclokwisely
π/2 radian and s is the reflection of the square about the line connecting 1 and 3. Then {1, 3} is the nontrivial block since
r{1, 3} = {2, 4} and {2, 4} ∩ {1, 3} = ∅ and
s{1, 3} = {1, 3}
and all the element of D8 is a combination of s and r. It means that D8 is not primitive.

(d) (5/21/2019) revised


Suppose that G is primitive and that H is a subgroup of G containing Ga . Let B = {σ(a) : σ ∈ H}. Note that σ(B) = B
for all σ ∈ H. Let g ∈ G \ H be given and let x ∈ g(B) ∩ B, then σ 0 (a) = x = gσ(a) for some σ 0 , σ ∈ H. Then observe
that σ 0 gσ(a) = a so σ 0 gσ ∈ Ga ≤ H. Then, since H is closed under the function composition and inverse, we have g ∈ H
which is a contradiction. Therefore, g(B) ∩ B = ∅ and so B is a block. Since G is primitive, then either B = {a} or B = A.
In the first case (B = {a}), then every element of H stabilizes a, so H = Ga . On the other hand, when B = A, note that
H = GB = {σ ∈ G : σ(B) = B} = GA = {σ ∈ G : σ(A) = A} = G since H = GB .

Conversely, note that since the blocks partition A, we can choose a block B that contains a. When GB = G, we claim that
B = A, for if B ( A there is some b0 ∈ B \ A. Since G is transitive, there is some permutation σ ∈ G sending b to b0 . But that
permutation doesn’t fix B, and it is a contradiction.
On the other hand, suppose that GB = Ga . For contradiction, assume B 6= {a} so that there is some b 6= a with b ∈ B. Since G is
transitive, then there exists σ ∈ G such that σ(a) = b. Then b ∈ σ(B) ∩ B, so σ(B) = B. Then σ ∈ GB but σ ∈ / Ga , contradiction.


4.1.8
A transitive permutation group of G on a set A is called doubly transitive if for any (hence all ) a ∈ A the subgroup Ga is transitive
on the set A \ {a}.
(a) Prove that Sn is doubly transitive on {1, 2, . . . , n} for all n ≥ 2.
(b) Prove that a doubly transitive group is primitive. Deduce that D8 is not doubly transitive in its action on the 4 vertices of a
square.
Proof. (B. Ban) .

(a)
Let i ∈ A = {1, 2, . . . , n} with n ≥ 2 be given. And then let a, b ∈ A \ {i} be given. Note that (a b) ∈ (Sn )i (stabilizer of i in Sn )
and (a b)a = b. Thus, (Sn )i is transitive on A \ {i} so Sn is doubly transitive.

(b)
Suppose that A = {1, 2, . . . , n} and G is doubly transitive group on A. Assume that there exists a nontrivial block, B, in A and
let b ∈ B be given. Since G is doubly transitive group, Gb is transitive on A \ {b}. Note that B \ {b} is a block for Ga since B
was a block so σ(B) = B or σ(B) ∩ B = ∅.
If σ(B \ {b}) = B \ {b} ∀σ ∈ Gb , then by 4.1.7(b), B \ {b} = A \ {b}, so B = A and it is a contradiction to the assumption.
On the other hand, if ∃σ ∈ Gb such that σ(B \ {b}) ∩ B \ {b} = ∅, note that σ(B) ∩ B = {b} which means that B is not a block.
It is a contradiction.
Therefore, there is no nontrivial block in A so G is primitive. 

Page 23
MATH 611 Editor : Byeongho Ban

4.1.10
Let H and K be subgroups of the group G. For each x ∈ G define the HK double coset of x in G to be the set
HxK = {hxk : h ∈ H, k ∈ K}
(a) Prove that HxK is the union of the left cosets xi K where {xi K : i ∈ I} is the orbit containing xK of H acting by left
multiplication on the set of left cosets of K.
(b) Prove that HxK is a union of right cosets of H.
(c) Show that HxK and HyK are either the same set or are disjoint for all x, y ∈ G. Show that the set of HK double cosets
partitions G.
(d) Prove that |HxK| = |K| · |H : H ∩ xKx−1 |.
(e) Prove that |HxK| = |H| · |K : K ∩ x−1 Hx|.
Proof. (B. Ban) .

(a)
Note that, ∀i ∈ I ∃hi ∈ H such that xi = hi x, so xi K = hi xK ⊆ HxK. Thus, it is clear that
[
xi K ⊆ HxK.
i∈I

Now let y ∈ HxK be given. Then y = hxk for some h ∈ H and k ∈ K. Thus, y = hxk ∈ hxK. Since O = {xi K : i ∈ I} is the
orbit containing xK, hxK ∈ O which means that
[ [
y ∈ hxK ⊆ xi K so HxK ⊆ xi K.
i∈I i∈I
Therefore,
[
xi K = HxK.
i∈I

(b)
Let R = {Hyj : j ∈ J} be the orbit containing Hx of K acting by right multiplication on the set of right cosets of H. Since
∀j ∈ J ∃kj ∈ K such that yj = xkj , so Hyj = Hxkj ⊆ HxK. Thus, it is clear that
[
Hyj ⊆ HxK.
j∈J

On the other hand, let z ∈ HxK be given, then x = h xk 0 for some h0 ∈ H and k 0 ∈ K. Thus, z = h0 xk 0 ∈ Hxk 0 . Thus,
0
[ [
z = h0 xk 0 ∈ Hxk 0 ⊆ Hyj so HxK ⊆ Hyj .
j∈J j∈J

Therefore,
[
Hyj = HxK
j∈J

so HxK is a union of right cosets of H.

(c)
Suppose that z ∈ HxK ∩ HyK. Then ∃h ∈ H and k ∈ K such that x = hyk. Then observe that
HxK = H(hyk)K = HyK
since H and K are subgroups so Hh = H and kK = K. Therefore, HxK and HyK are same or disjoint. Furthermore, note that
g = 1g1 ∈ HgK ∀g ∈ G so
[
G⊆ HgK.
g∈G

Since it is clear that HgK ⊆ G(∵ H, K ⊆ G) ∀g ∈ G,


[
G= HgK.
g∈G

Page 24
MATH 611 Editor : Byeongho Ban

Proof. (B. Ban) .

(d)
Suppose that the number of left cosets of K is finite. Note that xKx−1 is a subgroup of G so xKx−1 ∩ H is a subgroup of H.
Therefore, H is the disjoint union of left cosets of H ∩ xKx−1 so
[
H= hi (H ∩ xKx−1 ).
i∈I
Observe that
[
HxK = hi xK.
i∈I

If hi xK = hj xK then x−1 h−1 −1


j hi x ∈ K so hj hi ∈ H ∩ xKx
−1
which means that hi = hj . Thus, the second union is also disjoint
union. Then
|HxK|
|I| = [H : H ∩ xKx−1 ] = [HxK : K] = =⇒ |HxK| = |K| · [H : H ∩ xKx−1 ]
|K|
since the number of left cosets is finite.

(e)
Suppose that the number of right cosets of H is finite.
Note that x−1 Hx is a subgroup of G so x−1 Hx ∩ H is a subgroup of K. Therefore, K is the disjoint union of left cosets of
K ∩ x−1 Hx so
[
K= ki (K ∩ x−1 Hx).
i∈I
Observe that
[
HxK = Hxki .
i∈I

If Hxki = Hxkj then xkj ki−1 x−1 ∈ H so kj ki−1 ∈ K ∩ x−1 Hx which means that ki = kj . Thus, the second union is also disjoint
union. Then
|HxK|
|I| = [K : K ∩ x−1 Hx] = [HxK : H] = =⇒ |HxK| = |H| · [K : K ∩ x−1 Hx]
|H|
since the number of right cosets of H is finite.


Page 25
MATH 611 Editor : Byeongho Ban

4.2.3
Let r and s be the usual generators for the dihedral group of order 8.
(a) List the elements of D8 as 1, r, r2 , r3 , s, sr, sr2 , sr3 and label these with the integers 1, 2, . . . , 8 respectively. Exhibit the image
of each element of D8 under the left regular representation of D8 into S8 .
(b) Relabel this same list of elements of D8 with the integers 1, 3, 5, 7, 2, 4, 6, 8 respectively and recompute the image of each
element of D8 under the left regular representation with respect to this new labeling. Show that the two subgroups of S8 obtained
in parts (a) and (b) are different.
Proof. (B. Ban) .

(a)
Let the let the left regular representation for the labeling be φ. Then observe that
φ(r) φ(r) φ(r) φ(r)
1 ∼ 1 7−−−→ r ∼ 2, 2 ∼ r 7−−−→ r2 ∼ 3, 3 ∼ r2 7−−−→ r3 ∼ 4, 4 ∼ r3 7−−−→ 1 ∼ 1
φ(r) φ(r) φ(r) φ(r)
5 ∼ s 7−−−→ rs ∼ 8, 6 ∼ sr 7−−−→ rsr ∼ 5, 7 ∼ sr2 7−−−→ rsr2 ∼ 6, 8 ∼ sr3 7−−−→ rsr3 ∼ 7
=⇒ φ(r) = (1 2 3 4)(5 8 7 6)

φ(r 2 ) φ(r 2 ) φ(r 2 ) φ(r 2 )


1 ∼ 1 7−−−→ r2 ∼ 3, 2 ∼ r 7−−−→ r3 ∼ 4, 3 ∼ r2 7−−−→ 1 ∼ 1, 4 ∼ r3 7−−−→ r ∼ 2
φ(r 2 ) φ(r 2 ) φ(r 2 ) φ(r 2 )
5 ∼ s 7−−−→ r2 s ∼ 7, 6 ∼ sr 7−−−→ r2 sr ∼ 8, 7 ∼ sr2 7−−−→ r2 sr2 ∼ 5, 8 ∼ sr3 7−−−→ r2 sr3 ∼ 6
=⇒ φ(r2 ) = (1 3)(2 4)(5 7)(6 8)

φ(r 3 ) φ(r 3 ) φ(r 3 ) φ(r 3 )


1 ∼ 1 7−−−→ r3 ∼ 4, 2 ∼ r 7−−−→ 1 ∼ 1, 3 ∼ r2 7−−−→ r ∼ 2, 4 ∼ r3 7−−−→ r2 ∼ 3
φ(r 3 ) φ(r 3 ) φ(r 3 ) φ(r 3 )
5 ∼ s 7−−−→ r3 s ∼ 6, 6 ∼ sr 7−−−→ r3 sr ∼ 7, 7 ∼ sr2 7−−−→ r3 sr2 ∼ 8, 8 ∼ sr3 7−−−→ r3 sr3 ∼ 5
=⇒ φ(r3 ) = (1 4 3 2)(5 6 7 8)

φ(s) φ(s) φ(s) φ(s)


1 ∼ 1 7−−−→ s ∼ 5, 2 ∼ r 7−−−→ sr ∼ 6, 3 ∼ r2 7−−−→ sr2 ∼ 7, 4 ∼ r3 7−−−→ sr3 ∼ 8
φ(s) φ(s) φ(s) φ(s)
5 ∼ s 7−−−→ ss ∼ 1, 6 ∼ sr 7−−−→ ssr ∼ 2, 7 ∼ sr2 7−−−→ ssr2 ∼ 3, 8 ∼ sr3 7−−−→ ssr3 ∼ 4
=⇒ φ(s) = (1 5)(2 6)(3 7)(4 8)

φ(sr) φ(sr) φ(sr) φ(sr)


1 ∼ 1 7−−−→ sr ∼ 6, 2 ∼ r 7−−−→ sr2 ∼ 7, 3 ∼ r2 7−−−→ sr3 ∼ 8, 4 ∼ r3 7−−−→ s ∼ 5
φ(sr) φ(sr) φ(sr) φ(sr)
5 ∼ s 7−−−→ srs ∼ 4, 6 ∼ sr 7−−−→ srsr ∼ 3, 7 ∼ sr2 7−−−→ srsr2 ∼ 2, 8 ∼ sr3 7−−−→ srsr3 ∼ 1
=⇒ φ(sr) = (1 6 3 8)(2 7)(4 5)

φ(sr 2 ) φ(sr 2 ) φ(sr 2 ) φ(sr 2 )


1 ∼ 1 7−−−−→ sr2 ∼ 7, 2 ∼ r 7−−−−→ sr3 ∼ 8, 3 ∼ r2 7−−−−→ s ∼ 5, 4 ∼ r3 7−−−−→ sr ∼ 6
φ(sr 2 ) φ(sr 2 ) φ(sr 2 ) φ(sr 2 )
5 ∼ s 7−−−−→ sr2 s ∼ 3, 6 ∼ sr 7−−−−→ sr2 sr ∼ 4, 7 ∼ sr2 7−−−−→ sr2 sr2 ∼ 1, 8 ∼ sr3 7−−−−→ sr2 sr3 ∼ 2
=⇒ φ(sr2 ) = (1 7)(2 8)(3 5)(4 6)

φ(sr 3 ) φ(sr 3 ) φ(sr 3 ) φ(sr 3 )


1 ∼ 1 7−−−−→ sr3 ∼ 8, 2 ∼ r 7−−−−→ s ∼ 5, 3 ∼ r2 7−−−−→ sr ∼ 6, 4 ∼ r3 7−−−−→ sr2 ∼ 7
φ(sr 3 ) φ(sr 3 ) φ(sr 3 ) φ(sr 3 )
5 ∼ s 7−−−−→ sr3 s ∼ 2, 6 ∼ sr 7−−−−→ sr3 sr ∼ 3, 7 ∼ sr2 7−−−−→ sr3 sr2 ∼ 4, 8 ∼ sr3 7−−−−→ sr3 sr3 ∼ 1
=⇒ φ(sr3 ) = (1 8)(2 5)(3 6)(4 7)

φ(1) φ(1) φ(1) φ(1)


1 ∼ 1 7−−−→ 1 ∼ 1, 2 ∼ r 7−−−→ r ∼ 2, 3 ∼ r2 7−−−→ r2 ∼ 3, 4 ∼ r3 7−−−→ r3 ∼ 4
φ(1) φ(1) φ(1) φ(1)
5 ∼ s 7−−−→ s ∼ 5, 6 ∼ sr 7−−−→ sr ∼ 6, 7 ∼ sr2 7−−−→ sr2 ∼ 7, 8 ∼ sr3 7−−−→ sr3 ∼ 8
=⇒ φ(1) = 1


Page 26
MATH 611 Editor : Byeongho Ban

Proof. (B. Ban) .

In the similar fashion, letting the left regular representation for the new labeling be ψ, note that

ψ(1) ψ(1) ψ(1) ψ(1)


1 ∼ 1 7−−−→ 1 ∼ 1, 3 ∼ r 7−−−→ r ∼ 3, 5 ∼ r2 7−−−→ r2 ∼ 5, 7 ∼ r3 7−−−→ r3 ∼ 7
ψ(1) ψ(1) ψ(1) ψ(1)
2 ∼ s 7−−−→ s ∼ 2, 4 ∼ sr 7−−−→ sr ∼ 4, 6 ∼ sr2 7−−−→ sr2 ∼ 6, 8 ∼ sr3 7−−−→ sr3 ∼ 8
=⇒ ψ(1) = 1

ψ(r) ψ(r) ψ(r) ψ(r)


1 ∼ 1 7−−−→ r ∼ 3, 3 ∼ r 7−−−→ r2 ∼ 5, 5 ∼ r2 7−−−→ r3 ∼ 7, 7 ∼ r3 7−−−→ 1 ∼ 1
ψ(r) ψ(r) ψ(r) ψ(r)
2 ∼ s 7−−−→ rs ∼ 8, 4 ∼ sr 7−−−→ rsr ∼ 2, 6 ∼ sr2 7−−−→ rsr2 ∼ 4, 8 ∼ sr3 7−−−→ rsr3 ∼ 6
=⇒ ψ(r) = (1 3 5 7)(2 8 6 4)

ψ(r 2 ) ψ(r 2 ) ψ(r 2 ) ψ(r 2 )


1 ∼ 1 7−−−→ r2 ∼ 5, 3 ∼ r 7−−−→ r3 ∼ 7, 5 ∼ r2 7−−−→ 1 ∼ 1, 7 ∼ r3 7−−−→ r ∼ 3
ψ(r 2 ) ψ(r 2 ) ψ(r 2 ) ψ(r 2 )
2 ∼ s 7−−−→ r2 s ∼ 6, 4 ∼ sr 7−−−→ r2 sr ∼ 8, 6 ∼ sr2 7−−−→ r2 sr2 ∼ 2, 8 ∼ sr3 7−−−→ r2 sr3 ∼ 4
=⇒ ψ(r2 ) = (1 5)(2 6)(3 7)(4 8)

ψ(r 3 ) ψ(r 3 ) ψ(r 3 ) ψ(r 3 )


1 ∼ 1 7−−−→ r3 ∼ 7, 3 ∼ r 7−−−→ 1 ∼ 1, 5 ∼ r2 7−−−→ r ∼ 3, 7 ∼ r3 7−−−→ r2 ∼ 5
ψ(r 3 ) ψ(r 3 ) ψ(r 3 ) ψ(r 3 )
2 ∼ s 7−−−→ r3 s ∼ 4, 4 ∼ sr 7−−−→ r3 sr ∼ 6, 6 ∼ sr2 7−−−→ r3 sr2 ∼ 8, 8 ∼ sr3 7−−−→ r3 sr3 ∼ 2
=⇒ ψ(r3 ) = (1 7 5 3)(2 4 6 8)

ψ(s) ψ(s) ψ(s) ψ(s)


1 ∼ 1 7−−−→ s ∼ 2, 3 ∼ r 7−−−→ sr ∼ 4, 5 ∼ r2 7−−−→ sr2 ∼ 6, 7 ∼ r3 7−−−→ sr3 ∼ 8
ψ(s) ψ(s) ψ(s) ψ(s)
2 ∼ s 7−−−→ 1 ∼ 1, 4 ∼ sr 7−−−→ r ∼ 3, 6 ∼ sr2 7−−−→ r2 ∼ 5, 8 ∼ sr3 7−−−→ r3 ∼ 7
=⇒ ψ(s) = (1 2)(3 4)(5 6)(7 8)

ψ(sr) ψ(sr) ψ(sr) ψ(sr)


1 ∼ 1 7−−−→ sr ∼ 4, 3 ∼ r 7−−−→ sr2 ∼ 6, 5 ∼ r2 7−−−→ sr3 ∼ 8, 7 ∼ r3 7−−−→ s ∼ 2
ψ(sr) ψ(sr) ψ(sr) ψ(sr)
2 ∼ s 7−−−→ srs ∼ 7, 4 ∼ sr 7−−−→ srsr ∼ 1, 6 ∼ sr2 7−−−→ srsr2 ∼ 3, 8 ∼ sr3 7−−−→ srsr3 ∼ 5
=⇒ ψ(sr) = (1 4)(2 7)(3 6)(5 8)

ψ(sr 2 ) ψ(sr 2 ) ψ(sr 2 ) ψ(sr 2 )


1 ∼ 1 7−−−−→ sr2 ∼ 6, 3 ∼ r 7−−−−→ sr3 ∼ 8, 5 ∼ r2 7−−−−→ s ∼ 2, 7 ∼ r3 7−−−−→ sr ∼ 4
ψ(sr 2 ) ψ(sr 2 ) ψ(sr 2 ) ψ(sr 2 )
2 ∼ s 7−−−−→ sr2 s ∼ 5, 4 ∼ sr 7−−−−→ sr2 sr ∼ 7, 6 ∼ sr2 7−−−−→ sr2 sr2 ∼ 1, 8 ∼ sr3 7−−−−→ sr2 sr3 ∼ 3
=⇒ ψ(sr2 ) = (1 6)(2 5)(3 8)(4 7)

ψ(sr 3 ) ψ(sr 3 ) ψ(sr 3 ) ψ(sr 3 )


1 ∼ 1 7−−−−→ sr3 ∼ 8, 3 ∼ r 7−−−−→ s ∼ 2, 5 ∼ r2 7−−−−→ sr ∼ 4, 7 ∼ r3 7−−−−→ sr2 ∼ 6
ψ(sr 3 ) ψ(sr 3 ) ψ(sr 3 ) ψ(sr 3 )
2 ∼ s 7−−−−→ sr3 s ∼ 3, 4 ∼ sr 7−−−−→ sr3 sr ∼ 5, 6 ∼ sr2 7−−−−→ sr3 sr2 ∼ 7, 8 ∼ sr3 7−−−−→ sr3 sr3 ∼ 1
=⇒ ψ(sr3 ) = (1 8)(2 3)(4 5)(6 7)

Now observe that


(1 2 3 4)(5 8 7 6) ∈ φ(D8 ) but (1 2 3 4)(5 8 7 6) 6∈ ψ(D8 ).
Therefore, ψ(D8 ) 6= φ(D8 ).


Page 27
MATH 611 Editor : Byeongho Ban

4.2.7
Let Q8 be the quaternion group of order 8.
(a) Prove that Q8 is isomorphic to a subgroup of S8 .
(b) Prove that Q8 is not isomorphic to a subgroup of Sn for any n ≤ 7. [If Q8 acts on any set A of order ≤ 7 show that the
stabilizer of any point a ∈ A must contain the subgroup h−1i.]
Proof. (B. Ban) .

(a)
Let’s list the elements of Q8 as 1, −1, i, −i, j, −j, k, −k and label these with the integer 1, 2, 3, 4, 5, 6, 7, 8 respectively. Recall
that the left regular representation of Q8 into S8 is homomorphism and it is bijection which means that the representation is
isomorphism. Since the image of homomorphism is a subgroup of the codomain, Q8 is isomorphic to s subgroup of S8 .

(b)
Suppose that Q8 acts on a set of order ≤ 7. Let a ∈ A be given and
(Q8 )a = {g ∈ Q8 : g · a = a} Oa = {g · a : g ∈ Q8 }.
Observe that
|Q8 : (Q8 )a | = |Oa | ≤ |A| ≤ 7.
Thus, (Q8 )a should be nontrivial subgroup of Q8 . And note that any nontrivial proper subgroup of Q8 contains h−1i. Since a ∈ A
is arbitrary, h−1i ≤ (Q8 )a ∀a ∈ A. Therefore, if ψ is the permutation representation of the given action, note that
\
|ker(ψ)| = (Q8 )a ≥ 2.
a∈A
Thus, any homomorphism from Q8 into SA has nontrivial kernel and so is not an isomorphism.


4.3.2
Find all conjugacy classes and their sizes in the following groups:
(a) D8 (b) Q8 (c) A4 .
Proof. (B. Ban) .

(a)
Firstly, note that Z(D8 ) = 1, r2 so there are trivial conjugacy classes {1} and {r2 }. And note sr3 s = r, srs = r3 . Since the
powers of r are commute, {r, r3 } is another conjugacy class. And note rsr−1 = sr2 and rsr2 r−1 = s. With easy computation,
{s, sr2 } is also a conjugacy class. Lastly, note that {sr, sr3 } is the last conjugacy class since ssrs = sr3 and ssr3 s = sr.
conjugacy classes : {1}, {r2 }, {r, r3 }, {s, sr2 }, {sr, sr3 }.
Thus, except for two size 1 conjugacy classes {1} and {r2 }, all the other 3 conjugacy classes have size of either 2.

(b)
Note that Z(Q8 ) = {1, −1}. And observe that ki(−k) = −i, j(−i)(−j) = i, so {i, −i} is a conjugacy class. Similarly, since we
can exchange i with k or j and can make same argument, {j, −j} and {k, −k} are also conjugacy classes.

Conjugacy classes : {1}, {−1}, {i, −i}, {j, −j}, {k, −k}.
Except for size 1 conjugacy classes, {1} and {−1}, all the other 3 conjugacy classes have size 2.

(c)
Firstly note that |A4 | = 12. The representatives of each conjugacy classes would be 1, (1 2 3), (1 2)(3 4). Note that (1 2 3)
and (1 3 2) are not conjugate since (1 2)(1 2 3)(1 2) = (1 3 2) but (1 2) 6∈ A4 .

Conjugacy classes :
{1}, {(1 2 3), (2 4 3), (1 3 4), (1 4 2)}, {(1 3 2), (2 3 4), (1 4 3), (1 2 4)}, {(1 2)(3 4), (1 3)(2 4), (1 4)(2 3)}
So, there are one of size 1, one of size 3, and two of size 4 conjugacy classes.


Page 28
MATH 611 Editor : Byeongho Ban

Proof. (B. Ban) 4.3.4

Observe that, for given g ∈ G,


gNG (S)g −1 = g{h ∈ G : hSh−1 }g −1
= {ghg −1 ∈ G : hSh−1 = S}
= {k ∈ G : g −1 kgSg −1 k −1 g = S} use ghg −1 = k
= {k ∈ G : k(gSg −1 )k −1 = gSg −1 }
= NG (gSg −1 )
and that
gCG (S)g −1 = g{h ∈ G : hsh−1 = s ∀s ∈ S}g −1
= {ghg −1 ∈ G : hsh−1 = s ∀s ∈ S}
= {k ∈ G : g −1 kgsg −1 k −1 g = s ∀s ∈ S} use ghg −1 = k
= {k ∈ G : k(gsg −1 )k −1 = gsg −1 ∀s ∈ S}
= CG (gSg −1 ).
Then we are done.

Proof. (B. Ban) 4.3.5

Let K be a conjugacy class. If |K| = 1, we are done. Otherwise, letting g be a representative of K, we have |K| = |G : CG (g)|.
However, note that Z(G) ≤ CG (g) so we have |K| = |G : CG (g)| ≤ |G : Z(G)| = n. Therefore, every conjugacy class has at most
n elements.

Proof. (B. Ban) 4.3.6

Suppose that Z(G) 6= 1. Since Z(G) ≤ G, by Lagrange’s theorem, the possible order of Z(G) are 3, 5 and 15. However, since
G is non-abelian, |Z(G)| 6= 15. Then note that |G/Z(G)| is either 3 or 5 and they are both prime. But if |G/Z(G)| is a prime,
then G/Z(G) should be a cyclic group by Lagrange’s theorem. So every element in G can by written by xk z for some k ∈ Z and
z ∈ Z(G) where G/Z(G) = hxi. Then observe that, for any α, β ∈ G, there exist n, m ∈ Z and a, b ∈ Z(G) such that α = xn a
and β = xm b, so we can observe that
αβ = (xn a)(xm b) = xn (axm )b = xn xm ab = xm xn ba = xm bxn a = βα
since a, b ∈ Z(G). Therefore, G is abelian and it is a contradiction. Therefore, only possible case is when Z(G) = 1.

Now, note that hgi ≤ CG (g) for any g ∈ G. It implies that |Kg | = |G : CG (g)| ≤ |G : hgi |. Thus, if g is order 3, then |Kg | = 5, if
g is of order 5, then |Kg | = 3. Due to Cauchy’s theorem, it is guaranteed that there exists the element of order 3 and 5. Thus,
including the conjugacy class of identity, we have conjucacy classes of order 1, 3 and 5. Note that 1 + 3 + 5 = 9 and 15 − 9 = 6.
Since Z(G) = 1 so only possible order of other conjucacy class is 3 and 5, 6 should be explained by two conjugacy classes of order
3. So there are conjugacy classes of order 1, 3, 3, 3 and 5 and it is uniquely determined.

Proof. (B. Ban) 4.3.8

Let g ∈ Sn be a given m−cycle. Then the order of the conjugacy class K of g is |Sn : CG (g)| = n(n−1)···(n−m+1)
m which implies
|CG (g)| = m · (n − m)! < n! = |Sn |. In general, the product of disjoint cycle is also having smaller order of centralizer. Thus,
CG (g) < Sn ∀g ∈ Sn \ {1} so Z(Sn ) = 1.

4.3.11
In each of (a) − (d) determine whether σ1 and σ2 are conjugate. If they are, give an explicit permutation τ such that τ σ1 τ −1 = σ2 .
(a) σ1 = (1 2)(3 4 5) and σ2 = (1 2 3)(4 5)
(b) σ1 = (1 5)(3 7 2)(10 6 8 11) and σ2 = (3 7 5 10)(4 9)(13 11 2)
(c) σ1 = (1 5)(3 7 2)(10 6 8 11) and σ2 = σ13
(d) σ1 = (1 3)(2 4 6) and σ2 = (3 5)(2 4)(5 6).

Page 29
MATH 611 Editor : Byeongho Ban

Proof. (B. Ban) .

(a)
Let τa = (1 4 2 5 3) and observe that
τa σ1 τa−1 = (1 2 3)(4 5) = σ2 .
Thus, σ1 and σ2 are conjugate.

(b)
Note that
σ1 = (4)(9)(12)(13)(1 5)(3 7 2)(10 6 8 11)
σ2 = (1)(6)(8)(12)(4 9)(13 11 2)(3 7 5 10)
Let τb = (1 4 6 7 11 10 3 13 12 8 5 9)(2), then observe that
τb σ1 τb−1 = (2 13 11)(3 7 5 10)(4 9) = σ2 .

(c)
Note that
σ1 = (4)(9)(1 5)(3 7 2)(10 6 8 11)
σ2 = σ13 = (2)(3)(4)(7)(9)(1 5)(10 11 8 6)
and that the cycle type of σ1 is 1, 1, 2, 3, 4 and that of σ2 is 1, 1, 1, 1, 1, 2, 4. Since their cycle types are different, they are not
conjugate.

(d)
Note that
σ1 = (5)(1 3)(2 4 6)
σ2 = (1)(3 5)(2 4)(5 6) = (1)(2 4)(3 5 6).
And let
τd = (6)(1 2 3 4 5).
And observe that
τd σ1 τd−1 = (1)(2 4)(3 5 6) = σ2 .
Therefore, σ1 and σ2 are conjugate.


Page 30
MATH 611 Editor : Byeongho Ban

4.3.17
Let A be a nonempty set and let X be any subset of SA . Let
F (X) = {a ∈ A : σ(a) = a∀σ ∈ X} − the fixed set of X.
Let M (X) = A \ F (X) be the elements which are moved by some element of X. Let D = {σ ∈ SA : |M (σ)| < ∞}. Prove that D
is a normal subgroup of SA .
Proof. (B. Ban) .

Firstly, let’s prove that D is a subgroup.


Let σ ∈ SA be given, note that
a ∈ F (σ) ⇐⇒ σ(a) = a ⇐⇒ σ −1 (a) = a ⇐⇒ a ∈ F (σ −1 ),
so F (σ) = F (σ −1 ) which implies that M (σ) = M (σ −1 ).
Note that M (1) = 0, so 1 ∈ D so D 6= ∅.
Let φ, ψ ∈ D be given then |M (σ1 )| < ∞ and |M (σ2 )| < ∞. Now, observe that
a ∈ F ({φ, ψ}) =⇒ φ(a) = a = ψ(a) =⇒ ψ −1 φ(a) = a =⇒ a ∈ F (ψ −1 φ).
So F ({φ, ψ}) ⊆ F (ψ −1 φ) and it implies that M (ψ −1 φ) ⊆ M ({φ, ψ}). Since F ({φ, ψ}) = F (ψ) ∩ F (φ) which means M ({φ, ψ}) =
M (φ) ∪ M (ψ), observe that

M (φψ −1 ) ⊆ M ({φ, ψ}) ≤ M (φ) ∪ M (ψ).


Therefore, |M (φψ −1 )| ≤ |M (φ)| + |M (ψ)| < ∞ so φψ −1 ∈ D. Thus, D is a subgroup of SA .

Now, let’s show that D is normal.


Let σ ∈ D and τ ∈ SA be given . Now observe that
x ∈ F (τ στ −1 ) ⇐⇒ τ στ −1 (x) = x ⇐⇒ στ −1 (x) = τ −1 (x) ⇐⇒ τ −1 (x) ∈ F (σ) ⇐⇒ x ∈ τ F (σ).
Thus, F (τ στ −1 ) = τ F (σ) and it implies that M (τ στ −1 ) = τ M (σ). Now, observe that
|M (τ στ −1 )| = |τ M (σ)| = |M (σ)| < ∞
since σ ∈ D and τ is a bijection. Therefore, τ στ −1 ∈ D ∀σ ∈ D ∀τ ∈ SA and it means that τ Dτ −1 ⊆ D ∀τ ∈ SA . Thus, D is a
normal subgroup of SA .


4.3.25   
a b
Let G = GL2 (C) ans let H = : a, b, c ∈ C, ac 6= 0 . Prove that every element of G is conjugate to some element of the
0 c
subgroup H and deduce that G is the union of conjugates of H. [Show that every element of GL2 (C) has an eigenvector.]
Proof. (B. Ban) .
 
α β
Let g = be given. Note that g has an eigenvector if and only if gx = λx for some x ∈ C2 if and only if (g − λI)x = 0 for
γ δ
some x ∈ C2 (I is identity matrix.) if and only if det(g − λI) = 0. Since det(g − λI) is complex polynomial, there exists at least
one solution λ and it is the eigenvalue(we can find nonzero λ since g is invertible). Thus, there is an non zero vector x ∈ C2 and
it is the eigenvector corresponding to λ. Let y ∈ C2 such that x and y are linearly independent be given. Note that gy = ax + by
for some a, b ∈ C since {x, y} is a basis for C2 and that [x y] ∈ G. Then note that
   
λ a λ a
g[x y] = [λx gy] = [x y] =⇒ g = [x y] [x y]−1 .
0 b 0 b
 
λ a
Therefore, g is a conjugate of a matrix ∈ H and we are done since
0 b
 
λ a
λb = det = det([x y]−1 g[x y]) 6= 0 (∵ [x y], g ∈ G).
0 b


Page 31
MATH 611 Editor : Byeongho Ban

4.4.1
If σ ∈ Aut(G) and ϕg is conjugation by g prove σϕg σ −1 = ϕσ(g) . Deduce that Inn(G) E Aut(G). (The group Aut(G)/Inn(G) is
called the outer automorphism group of G. )
Proof. (B. Ban) .

Observe that
σϕg σ −1 (h) = σϕg (σ −1 (h)) = σ(gσ −1 (h)g −1 ) = σ(g)hσ(g −1 ) = σ(g)hσ(g)−1 = ϕσ(g) (h) ∀h ∈ G.
Therefore, σϕg σ −1 = ϕσ(g) . And it means that
σϕg σ −1 = ϕσ(g) ∈ Inn(G) ∀σ ∈ Aut(G)∀ϕg ∈ Inn(G).
Thus, σInn(G)σ −1 ⊆ Inn(G) ∀σ ∈ Aut(G) so Inn(G) E Aut(G).


4.4.3
Prove that under any automorphism of D8 , r has at most 2 possible images and s has at most 4 possible images (r and s are the
usual generators - cf. Section 1.2). Deduce that |Aut(D8 )| ≤ 8.
Proof. (B. Ban) .

Observe that |r| = 4 and |s| = 2. Since only element in D8 of order 4 is r and r3 , and automorphism preserves order of each
elements, for any automorphism σ, only possible images of r under σ are r and r2 . Thus, there are only 2 possible images
for r.
On the other hand, note that the elements in D8 of order 2 are s, sr, sr2 , sr3 , r2 . However, r2 cannot be an image of s under any
automorphism. If it is, there is τ ∈ Aut(G) such that τ (s) = r2 and τ (r) = r. Then observe that
τ (sr) = τ (s)τ (r) = r3 = τ (r)τ (s) = τ (rs) = τ (sr3 ).
Since τ is bijection and sr 6= sr3 , it is a contradiction. Thus, s has at most 4 possible images under any automorphism.

Note that, since s and r are generators of D8 , once we specify the value of any automorphism at s and r, the automorphism is
totally specified. For example,
τ (sj ri ) = τ (s)j τ (r)i .
Thus, we have at most 2 × 4 = 8 choices of automorphism and it means that
|Aut(G)| ≤ 8.


Proof. (B. Ban) 4.4.4

Note that Q8 = i, j : i2 = j 2 = k 2 = −1 . Thus, any automorphism of Q8 is determined by the images of i and j. Since the order
of i and j is 4, the image should also be of order 4. Note that, so the image of i and j should be one of ±i, ±j, and ±k. If we
choose the image of i first, the number of the possible choices is 6. Then the possible images of j are one of the six except ϕ(±i).
Observe that, without loss of generality, when ϕ(i) = j and ϕ(j) = −j, then we have
ϕ(ij) = −j 2 = 1
but ij has order 4 so it is a contradiction. Therefore, the number of possible automorphism would be at most 6 × 4 = 24.


4.4.6
Prove that characteristic subgroups are normal. Give an example of a normal subgroup that is not characteristic.
Proof. (B. Ban) .

Let H ≤ G be a characteristic subgroup. Let ϕg ∈ Inn(G) ⊆ Aut(G), then observe that


gHg −1 = ϕg (H) = H
since H is characteristic subgroup. Since ϕg ∈ Inn(G), so g ∈ G is arbitrary, H E G.

Let G = Z × Z and a = (0, 1) and b = (1, 0). Since G is abelian, hai and hbi are normal subgroup of G. However, even if ϕ : G → G
defined by ϕ(x, y) = (y, x) is an automorphism, ϕ(hai) = hbi. Thus, hai is the normal subgroup that is not characteristic subgroup.


Page 32
MATH 611 Editor : Byeongho Ban

4.4.18
This exercise shows that for n 6= 6 every automorphism of Sn is inner. Fix an integer n ≥ 2 with n 6= 6.
(a) Prove that the automorphism group of a group G permutes the conjugacy classes of G, i.e., for each σ ∈ Aut(G) and each
conjugacy class K of G the set σ(K) is also a conjugacy class of G.
(b) Let K be the conjugacy class of transpositions in Sn and let K0 be the conjugacy class of any element of order 2 in Sn that
6 |K0 |. Deduce that any automorphism of Sn sends transpositions to transpositions . [See
is not a transposition. Prove that |K| =
Exercise 33 in Section 3.]
(c) Prove that for each σ ∈ Aut(Sn ),
σ : (1 2) 7→ (a b2 ), σ : (1 3) 7→ (a b3 ), . . . , σ : (1 n) 7→ (a bn )
for some distinct integers a, b1 , b2 , . . . , bn ∈ {1, 2, . . . , n}.
(d) Show that (1 2), (1 3), . . . , (1 n) generate Sn and deduce that any automorphism of Sn is uniquely determined by its
action on these elements. Use (c) to show that Sn has at most n! automorphisms and conclude that Aut(Sn ) = Inn(Sn ) for n 6= 6.

Page 33
MATH 611 Editor : Byeongho Ban

Proof. (B. Ban) .

(a)
Let σ ∈ Aut(G) be given. And let K be a conjugacy class of G, then we can present it as K = {gag −1 : g ∈ G} for some a ∈ G.
Observe that, letting σ(a) = b for some b ∈ G,
σ(K) = σ({gag −1 : g ∈ G}) = {σ(g)σ(a)σ(g)−1 : g ∈ G} = {gσ(a)g −1 : g ∈ G}
since σ is a bijection. Note that σ(K) is a conjugacy class of G containing σ(a). Therefore, Aut(G) permutes the conjugacy
classes of G.

(b)
When n = 2, it is clear, thus let’s suppose that n > 2.
Note that
n(n − 1)
. |K| =
2
(Choose two numbers from n different numbers and eliminate the case due to the cases for (1 2) = (2 1)).
And note that every element in K0 is composition of at least two disjoint transpositions. Then from Exercise 33 of section 3,
X n!
|K0 | =
k!2k
k
+ n 0
where k runs in Z ∩ [1, 2 ]. Then, |K| 6= |K | if n 6= 6.

(c)
Let i, j ∈ {2, 3, . . . , n} From (b), we know that σ(1 i) and σ(1 j) are transpositions for σ ∈ Aut(Sn ). Note that
(1 i)(1 j)(1 i) = (i j) 6= (1 j) σ(1 i)σ(1 j)σ(1 i) = σ(i j) 6= σ(1 j)
and it means that σ(1 i) and σ(1 j) are not disjoint. In addition, since σ is a bijection, σ(1 j) and σ(1 i) are not same. Therefore,
σ(1 i) = (a bi ) and σ(1 j) = (a bj ) for some a, bi , bj ∈ {1, 2, . . . , n}. Since i and j are arbitrary,
σ : (1 2) 7→ (a b2 ), σ : (1 3) 7→ (a b3 ), . . . , σ : (1 n) 7→ (a bn )
for some distinct integers a, b1 , b2 , . . . , bn ∈ {1, 2, . . . , n}.

(d)
Observe that for any distinct elements i, j, l, . . . , k ∈ {1, 2, . . . , n},
(i j) = (1 i)(1 j)
and that
(i j l · · · k) = (i k) · · · (i l)(i j).
Therefore, with A = {(1 a) : a ∈ {1, 2, . . . , n}}, we can create every cycles, so we can generates Sn . If, for given σ ∈ Sn , σ(1 a)
is specified ∀a ∈ {1, 2, . . . , n}, observe that
σ(i j l · · · k) = σ(i k) · · · σ(i l)σ(i j) = σ(1 i)σ(1 k) · · · σ(1 i)σ(1 l)σ(1 i)σ(1 j).
Thus, arbitrary cycle is specified once we specify the value of σ on A. Thus, any automorphism of Sn is uniquely determined by
its action on A.
Note that, since |A| = n − 1, there are (n − 1)! different ways to define the automorphisms. Since 1 is common integer to every
transposition in A, we can change 1 to any (n − 1) integers. So we have n ways to specify the common integer. Thus, The
total way to define the automorphism of Sn is n! which means that |Aut(Sn )| ≤ n!. Since Sn /Z(Sn ) ∼ = Inn(Sn ) ≤ Aut(Sn ) and
|Sn /Z(Sn )| = |Sn | = n!. Thus,
n! ≤ |Sn /Z(Sn )| = |Inn(Sn )| ≤ |Aut(Sn )| ≤ n!
so |Aut(Sn )| = |Inn(Sn )| for n 6= 6.


Page 34
MATH 611 Editor : Byeongho Ban

4.4.19
This exercise shows that |Aut(S6 ) : Inn(S6 )| ≤ 2 (Exercise 10 in Section 6.3 shows that equality holds by exhibiting an automor-
phism of S6 that is not inner).
(a)
Let K be the conjugacy class of transpositions in S6 and let K0 be the conjugacy class of any element of order 2 in S6 that is not
6 |K0 | unless K0 is the conjugacy class of products of three disjoint transpositions. Deduce that
a transposition. Prove that |K| =
Aut(S6 ) has a subgroup of index at most 2 which sends transpositions to transpositions.
(b) Prove that |Aut(S6 ) : Inn(S6 )| ≤ 2. [Follow the same steps as in (c) and (d) of the preceding exercise to show that any
automorphism that sends transpositions to transpositions is inner.]
Proof. (B. Ban) .

(a)


Page 35
MATH 611 Editor : Byeongho Ban

4.5.1
Prove that if P ∈ Sylp (G) and H is a subgroup of G containing P then P ∈ Sylp (H). Give an example to show that, in general,
a Sylow p−subgroup of a subgroup of G need not be a Sylow p-subgroup of G.
Proof. (B. Ban) .

Since P ∈ Sylp (G), we have |P | = pk for some k ∈ Z+ . Then since P ≤ H, by Lagrange;’s Theorem, pk ||H|. But note that
gcd(p, |G|/pk ) = 1 so gcd(p, |H|/pk ) = 1. Thus, P ∈ Sylp (H).
For example, in Q8 , hii ∈ Syl2 (hii) but hii 6∈ Syl2 (Q8 ) since sylow 2-group of Q8 has order 8. 

4.5.3
Use Sylow’s Theorem to prove Cauchy’s Theorem. (Note that we only used Cauchy’s theorem for abelian groups- Proposition
3.21 - in the proof of Sylow’s Theorem so this line of reasoning is not circular.)
Proof. (B. Ban) .

Suppose that G is finite group and p is a prime dividing |G|. Then by Sylow’s theorem, ∃P ∈ Sylp (G) with |P | = pk for some
k ∈ Z+ . Let x ∈ P \ {1} then |x||pk by Lagrange’s theorem since |x| = | hxi |. Thus, |x| = pl for some l ∈ {1, 2, . . . , k}. Then let
l−1 l−1 l−1 l
y = xp and note that |y| = |xp | = p since (xp )p = xp = 1 and pl is the order of x. Thus, there exists an element of order
p in G.


4.5.7
Exhibit all Sylow 2-subgroups of S4 and find elements of S4 which conjugate one of these into each of the others.
Proof. (B. Ban) .

Note that |S4 | = 4! = 24 = 23 3 then by Sylow’s theorem, n2 (S4 ) ≡ 1 mod 2 and n2 (S4 )|3. Then the possible n2 (S4 ) are 1 and
3. If n2 = 1, the sylow 2-subgroup is normal. However, note that there are subgroup of S4 that isomorphic to D8 and it is a
non-normal subgroup of S4 with order 8. Thus, n2 = 3. Then
A = {(1), (1234), (13)(24), (1432), (13), (24), (12)(34), (14)(23)}
B = {(1), (12), (34), (12)(34), (13)(24), (14)(23), (1324), (1423)}
C = {(1), (1243), (1342), (14), (23), (12)(34), (13)(24), (14)(23)}
are all Sylow 2−subgroups of S4 . And observe that (124)B(123)−1 = A, (123)B(123)−1 = C. Since conjugation is equivalence
relation, A, B and C are conjugation of each other. 

Proof. (B. Ban)


4.5.9

Let M ∈ SL2 (F3 ) be written as


 
a b
M= .
c d
First suppose d = 0. Then det M = −bc = 1, so b = −c . Hence M has the form
 
a b
M=
−b 0
with b 6= 0. There are 6 such matrices in total.
Now suppose d 6= 0. Then since det M = ad − bc = 1, we find a = (1 + bc)d, hence M have the form
 
(1 + bc)d b
M=
c d
There are in total 3 · 3 · 2 = 18 such matrices. In total there are 18 + 6 = 24 elements in SL2 (F3 ).

Suppose
  P ∈ Syl3 (SL2 (F3 )), then
that  |P | = 3 since |SL2 (F3 )| = 23 · 3 so, by Sylow’s theorem, n3 = 1 or 4. Note that if
1 1 1 −1 1 0
a= , then |a| = 3 with a2 = . Also note that b = is also of order 3 but b 6∈ hai, thus, n3 = 4. Then, we
0 1 0 1 1 1
have 4 distinct Sylow 3− subgroups
       
1 1 1 0 0 1 1 −1
, , , .
0 1 1 1 −1 −1 0 −1

Page 36
MATH 611 Editor : Byeongho Ban

Proof. (B. Ban)


4.5.10 (Incomplete... How can I prove the uniqueness??)
   
0 −1 1 1
Let a = and b = . We will show that ha, bi ≤ SL2 (F3 ) is the unique Sylow 2-subgroup. Letting I be a 2 × 2
1 0 1 −1
 
−1 1
identity matrix, observe that a2 = b2 = −I which implies that |a| = |b| = 4. In addition, observe that c := ab = and
1 1
(ab)2 = −I which also implies |c| = 4. Also, with simple computation, we can observe that ab = −ba. Thus, we get the relations
a2 = b2 = c2 = −I c = ab = −ba
− b = ac = aab = −aba = −ca a = bc = bab = −abb = −cb

which implies that ha, bi = Q8 by using the isomorphism a 7→ i, b 7→ j and c 7→ k.

Since |SL2 (F3 )| = 24 = 23 · 3, ha, bi is the Sylow 2−subgroup of SL2 (F3 ).




4.5.11
Show that the center of SL2 (F3 ) is the group of order 2 consisting of ±I, where I is the identity matrix. Prove that
SL2 (F3 )/Z(SL2 (F3 )) ∼
= A4 [Use facts about groups of order 12.]
Proof. (B. Ban) .

Note that SL2 (F3 ) a normal subgroup, and |GL2 (F3 )| = 3(3 + 1)(3 − 1)2 = 48. Thus, note that Z(SL2 (F3 )) = {±I}.


Page 37
MATH 611 Editor : Byeongho Ban

4.5.18
Prove that a group of order 200 has a normal Sylow 5-subgroup.
Proof. (B. Ban) .

Note that 200 = 23 · 52 . Let G be a group of order 200. By Sylow’s Theorem,


n5 (G) ≡ 1 mod 5 n5 (G)|8.
Thus, n5 (G) = 1. It means the group, G, has unique Sylow 5-subgroup, H. Then by a proposition in the book, H is a normal
subgroup of G.


4.5.30
How many elements of order 7 must there be in a simple group of order 168?
Proof. (B. Ban) .

Note that 168 = 23 · 3 · 7. Let G be the simple group of order 168. Note that any element of order 7 should generate a cyclic
group of order 7 and this cyclic group is Sylow 7-subgroup. By Sylow’s Theorem,
n7 ≡ 1 mod 2 n7 |24.
Since G is simple, n7 > 1. Thus, n7 = 8 and there are 8 cyclic subgroup of order 7. In each cyclic group, there are 6 different
generators. Therefore, we have 6 · 8 = 48 element of order 7 in the group G.


4.5.40
Prove that the number of Sylow p−subgroups of GL2 (Fp ) is p + 1. [Exhibit two distinct Sylow p−subgroups.]
Proof. (B. Ban) .

Let G = GL2 (Fp ). Note that |GL2 (Fp )| = (p2 − p)(p2 − 1) = p(p + 1)(p − 1)2 , so Sylow p-subgroup of G has order p.
Note that, for given a ∈ Fp \ {0, 1},
   
1 a 1 0
A= ∈ Sylp (G) B= ∈ Sylp (G)
0 1 a 1
since
 n    n  
1 a 1 na 1 0 1 0
= = .
0 1 0 1 a 1 na 1
Then, since np = 1 + kp for some k ∈ Z+ , np ≥ p + 1.
On the other hand, observe that, for a, b, b ∈ Fp ,
−1  −1
−a−1 bc−1
 
a b a
= .
0 c 0 c−1
And observe that, for t ∈ Fp \ {0, 1},
−1 
a−1 ct
    
a b 1 t a b 1
= .
0 c 0 1 0 c 0 1
Thus, the space of invertible upper triangular matrices, U , is a subspace of NG (A). Since any element of U has non zero diagonal
entries and any upper right entry, the order of U is p(p − 1)2 . Thus, p(p − 1)2 |NG (A). Then by Sylow’s theorem,
p(p − 1)2 (p + 1) p(p − 1)2 (p + 1)
np = [G : NG (A)] = ≤ =p+1
|NG (A)| p(p − 1)2
Therefore, np = p + 1. 

Page 38
MATH 611 Editor : Byeongho Ban

Due date: October 10th, 2018 Byeongho Ban

4.6.4
Prove that An is generated by the set of all 3−cycles for each n ≥ 3.
Proof. (B. Ban)
Let a natural number n ≥ 3 be given and then let Tn be the set of all three cycles in Sn . Observe that, for any a, b, c ∈ {1, 2, . . . , n},
(a b c) = (a c)(a b) ∈ An .
Thus, hTn i ≤ An .

On the other hand, let σ ∈ An be given. Then we have


σ = τ1 τ2 · · · τi−1 τi ,
where i is even number and each τj is transposition. Without loss of generality, we can assume that τ2j 6= τ2j−1 ∀j ∈ {1, 2, . . . , 2i }.
(Otherwise, τi τi−1 = 1.) If τ2j−1 and τ2j does share one element, for example, if τ2j−1 = (a b) and τ2j = (a c), then observe that
τ2j−1 τ2j = (a b)(a c) = (a c b).
If they does not share any element, for example, τ2j−1 = (a b) and τ2j = (c d), then observe that
τ2j−1 τ2j = (a b)(c d) = (a b)(a c)(a c)(c d) = (a c b)(c d a).
Thus, τ2k−1 τ2j is a product of three cycles ∀j ∈ {1, 2, . . . , 2i } and it means
σ = (τ1 τ2 ) · · · (τi−1 τi )
is a product of 3− cycles. Therefore, σ ∈ hTn i and thus An ≤ hTn i.
In conclusion, An = hTn i so An is generated by the set of all 3−cycles.

5.1.4
Let A and B be finite groups and let p be a prime. (1) Prove that any Sylow p-subgroup of A × B is of the form P × Q, where
P ∈ Sylp (A) and Q ∈ Sylp (B). (2) Prove that np (A × B) = np (A)np (B). (3) Generalize both of these results of a direct product
of any finite number of finite groups (so that the number of Sylow p− subgroups of a direct product is the product of the numbers
of Sylow p−subgroups of the factors).
Proof. (B. Ban) .

(1) If p 6 ||A| or p 6 ||B|, then it is trivially true. Thus, let’s suppose that p||A| and p||B| and that |A| = pα m and |B| = pβ n where
p 6 |m and p 6 |n.
Let G ∈ Sylp (A × B) be given which implies that |G| = pα+β . And let P ∈ Sylp (A) and Q ∈ Sylp (B) be given. Then note that
P × Q ∈ Sylp (A × B) since |P × Q| = |P ||Q| = pα pβ = pα+β . Then by Sylow’s theorem, we know that
G = (g, h)(P × Q)(g, h)−1 for some (g, h) ∈ A × B.
Observe that
G = (g, h)(P × Q)(g, h)−1 = (gP g −1 ) × (hQh−1 ).
Since conjugation of Sylow p-subgroup is again Sylow p-subgroup, we are done.

(2) From (1), we know that


Sylp (A × B) = Sylp (A) × Sylp (B).
Thus, observe that
np (A × B) = |Sylp (A × B)| = |Sylp (A)||Sylp (B)| = np (A)np (B).

(3) Let {Gn }n∈N be any countable sequence of finite groups. Note that
Sylp (G1 × G2 ) = Sylp (G1 ) × Sylp (G2 ).
And assume that
Sylp (G1 × · · · × Gn−1 ) = Sylp (G1 ) × · · · × Sylp (Gn−1 ).
Then let G ∈ Sylp (G1 × · · · × Gn ) be given. Then by (1), G = P × Q for some P ∈ Sylp (G1 × · · · × Gn−1 ) and Q ∈ Sylp (Gn ).
Thus,
Sylp (G1 × · · · × Gn ) = Sylp (G1 × · · · × Gn−1 ) × Sylp (Gn ) = Sylp (G1 ) × · · · × Sylp (Gn ).
Thus, by mathematical induction, for any finite product of finite groups, the two results hold.

Page 39
MATH 611 Editor : Byeongho Ban

5.1.5
Exhibit a nonnormal subgroup of Q8 × Z4 (note that every subgroup of each factor is normal.).
Proof. (B. Ban) Let
H = h(i, x)i = {(i, x), (−1, x2 ), (−i, x3 ), (1, x4 )},
where Z4 = hxi and x4 = 1.
Note that H ≤ Q8 × Z4 but it is not a normal subgroup since
(j, x)(i, x)(j, x)−1 = (ji(−j), xxx−1 ) = (−i, x) 6∈ H.

5.1.12
Let A and B be groups. Assume Z(A) contains a subgroup Z1 and Z(B) contains a subgroup Z2 with Z1 ∼ = Z2 . Let this
isomorphism be given by the map xi 7→ yi for all xi ∈ Z1 . A central product of A and B is a quotient
(A × B)/Z where Z = {(xi , yi−1 ) : xi ∈ Z1 }
and is denoted by A ∗ B− it is not unique since it depends on Z1 and Z2 and the isomorphism between them. (Think of A ∗ B as
the direct product of A and B ”collapsed” by identifying each element xi ∈ Z1 with its corresponding element yi ∈ Z2 ).
(a) Prove that the images of A and B in the quotient group A ∗ B are isomorphic to A and B, respectively, and that these images
intersect in a central subgroup isomorphic to Z1 . Find |A ∗ B|.
(b) Let Z4 = hxi. Let D8 = hr, si and Q = hi, ji be given by their usual generators ans relations. Let Z4 ∗ D8 be the central
product of Z4 and D8 which identifies x2 and r2 (i.e., Z1 = x2 , Z2 = r2 and the isomorphism is x2 7→ r2 .) and let Z4 ∗ Q8 be
the central product of Z4 and Q8 which identifies x2 and −1. Prove that Z4 ∗ D8 ∼= Z4 ∗ Q8 .
Proof. (B. Ban) .

(a)
Let q : A × B → (A × B)/Z, pA : A → A × B and pB : B → A × B defined by
q : (a, b) 7→ (a, b)Z
pA : a 7→ (a, 1B )
pB : b 7→ (1A , b).
Observe that
q ◦ pA : A → (A × B)/Z ker(q ◦ pA ) = {a ∈ A : (a, 1B ) ∈ Z} = {1A },
q ◦ pB : B → (A × B)/Z ker(q ◦ pB ) = {b ∈ B : (1A , b) ∈ Z} = {1B }.
since for any (a, b) ∈ A ∗ B, b is the image of inverse of a under the given isomorphism.
Thus, q ◦ pA and q ◦ pB are injections.
And note that Z ⊆ Z(A) × Z(B), so Z E A ∗ B. Thus, binary operation of quotient group is well defined and further observe that
q ◦ pA (a1 a2 ) = (a1 a2 , 1B )Z = q ◦ pA (a1 )q ◦ pA (a2 ) q ◦ pB (b1 b2 ) = (1A , b1 b2 )Z = q ◦ pA (b1 )q ◦ pA (b2 )
which implies that q ◦ pA and q ◦ pB are homomorphism. Therefore, A and B are isomorphic to the images under q ◦ pA and q ◦ pB .
Lastly, observe that
 
|A||B|
|A ∗ B| = |A × B : Z|. = if A and B are finite.
|Z1 |

(b) will be continued on next page. 

Page 40
MATH 611 Editor : Byeongho Ban

Proof. (B. Ban) .

(b)
Let
Z1 = {(1, 1), (x2 , r2 )} and Z2 = {(1, 1), (x2 , −1)}
Note that
Z4 ∗ D8 = (Z4 × D8 )/Z1 = {(a, b){(1, 1), (x2 , r2 )} : (a, b) ∈ Z4 × D8 } = h(1, r)Z1 , (1, s)Z1 , (1, x)Z1 i
Z4 ∗ Q8 = (Z4 × Q8 )/Z2 = {(a, b){(1, 1), (x2 , r2 )} : (a, b) ∈ Z4 × Q8 } = h(x, 1)Z2 , (1, i)Z2 , (x, j)Z2 i .
Let’s define a function ψ : Z4 ∗ D8 → Z4 ∗ Q8 as
ψ ((x, 1)Z1 ) = (x, 1)Z2 , ψ ((1, r)Z1 ) = (1, i)Z2 , ψ ((1, s)Z1 ) = (x, j)Z2 , ψ ((1, 1)Z1 ) = (1, 1)Z2
ψ((x, r)Z1 ) = ψ((x, 1)Z1 )ψ((1, r)Z1 ) ψ((x, s)Z1 ) = ψ((x, 1)Z1 )ψ((1, s)Z1 )
And observe that
4
ψ (x, 1)4 Z1 = ψ ((1, 1)Z1 ) = (1, 1)Z2 = (x, 1)4 Z2 = ψ ((x, 1)Z1 )


4
ψ (1, r)4 Z1 = ψ ((1, 1)Z1 ) = (1, 1)Z2 = (1, i)4 Z2 = ψ ((1, r)Z1 )


2
ψ (1, s)2 Z1 = ψ ((1, 1)Z1 ) = (1, 1)Z2 = (x, j)2 Z2 = ψ ((1, s)Z1 )


ψ ((1, rs)Z1 ) = ψ ((1, r)(1, s)Z1 ) = (1, i)(x, j)Z2 = (x, k)Z2 = (x, j)(1, i3 )Z2 = ψ((1, s)(1, r3 )Z2 ) = ψ((1, sr3 )Z2 ).
Therefore, ψ is a homomorphism. Clearly, by the definition, it is a bijection. Therefore, Z4 ∗ D8 ∼ = Z4 ∗ Q8 .

5.2.1
In each of parts (a) to (e) give the number of nonisomorphic abelian groups of the specified order − do not list the groups:
(a) order 100, (b) order 576, (c) order 1155, (d) order 42875, (e) order 2704.
Proof. (B. Ban) .

(a) 4
Note that 100 = 22 52 . Note that there are two different partitions for 2, (2) and (1,1). Thus, there are 2 × 2 = 4 different types
of nonisomorphic abelian groups of order 100.

(b) 22
Note that 576 = 26 32 and that there are 11 different partitions for 6, (6), (5 1), (4 2), (4 1 1), (3 3), (3 2 1), (3 1 1 1), (2 2 2),
(2 2 1 1), (2 1 1 1 1), and (1 1 1 1 1 1) and two different partitions for 2. Therefore, there are 11 × 2 = 22 different types of
nonisomorphic abelian groups of order 576.

(c) 1
Note that 1155 = 3 · 5 · 7 · 11. Since each prime factor of 1155 is one, there is only one possible abelian group of order 1155 which
is cyclic.

(d) 9
Note that 42875 = 53 · 73 and that there are three different partitions of 3, (3), (2 1), and (1 1 1). Thus there are 3 × 3 = 9
different types of nonisomorphic abelian groups of order 42875.

(e) 10
Note that 2704 = 24 132 and that there are 5 different partitions of 4, (4), (3 1), (2 2), (2 1 1), and (1 1 1 1). Thus, there are
5 × 2 = 10 different types of nonisomorphic abelian groups of order 2704. 

Page 41
MATH 611 Editor : Byeongho Ban

5.2.6
Prove that any finite group has a finite exponent. Give an example of an infinite group with finite exponent. Does a finite group
of exponent m always contain an element of order m?
Proof. (B. Ban) (Revised 6/1/2019)

Let G be a finite group of order n. Then for any x ∈ G, xn = 1. Thus, the exponent of G should be less than equal to n which is
finite.

Let

Y
H = Z2 × Z2 × · · · = Z2 .
n=1

Note that H is an infinite group. But note that, for any element h ∈ H, h2 = 1. Therefore, H has a finite exponent of 2 even if
it is infinite group.

Lastly, some finite group of exponent m can contain no element of order m. For example, consider S3 which has exponent 6 but
does not contain an element of order 6.

5.2.9
Let A = Z60 × Z45 × Z12 × Z36 . Find the number of elements of order 2 and the number of subgroups of index 2 in A.
Proof. (B. Ban) .

Observe that
A∼
= (Z4 × Z5 × Z3 ) × (Z5 × Z9 ) × (Z3 × Z4 ) × (Z4 × Z9 ).
Note that there are 1 element of order 2 in Z60 (let x1 ), 0 element of order 2 in Z45 , 1 element of order 2 in Z12 (let x2 ), and 1
element of order 2 in Z36 (let x3 ). Then there are 2 × 2 × 2 − 1 = 23 − 1 = 8 − 1 = 7 elements of order 2 since identity (1, 1, 1, 1)
has order 1.
Nextly, resetting xi for 1 = 1, 2, 3, 4, note that Z60 = hx1 i has has one index subgroup x30 1 , Z45 = hx2 i has no subgroup of
6
index 2, Z12 = hx3 i has one subgroup x3 of index 2 subgroup and Z36 = hx4 i has one subgroup, x14 8 of index 2. Since A is a
subgroup of index 1, there are 2 × 1 × 2 × 2 − 1 = 7 subgroups of A of index 2. 

Page 42
MATH 611 Editor : Byeongho Ban

5.2.14
For any group G define the dual group of G (denoted G) b to be the set of all homomorphisms from G into the multiplicative group
of roots of unity in C. Define a group operation in G b by pointwise multiplication of functions : if χ, ψ are homomorphisms from
G into the group of roots of unity then χψ is the homomorphism given by (χψ)(g) = χ(g)ψ(g) for all g ∈ G, where the latter
multiplication takes place in C.
(a) Show that this operation on G b makes Gb into an abelian group. [Show that the identity is the map g 7→ 1 for all g ∈ G and
−1
the inverse of χ ∈ G is the map g 7→ χ(g) .]
b
b∼
(b) If G is a finite abelian group, prove that G = G. [Write G as hx1 i×· · ·×hxr i and if ni = |xi | define χi to be the homomorphism
2πi/ni
which sends xi to e and sends xj to 1, for all j 6= i. Prove χi has order ni in G
b and Gb = hχ1 i × · · · × hχr i.]

Proof. (B. Ban) Let the multiplicative group of roots of unity in C be M .


(a)
Let ι(g) = 1 ∀g ∈ G. Then clearly, ι is a homomorphism from G to M , so ι ∈ G.
b And note that
(ιψ)(g) = ι(g)ψ(g) = ψ(g) = ψ(g)ι(g) = (ψι)(g) ∀g ∈ G.
Thus, ι is the identity element in G.
b
Let χ ∈ G be given. And let’s define φ(g) = χ(g)−1 for all g ∈ G. And note that
b
−1
φ(gh) = χ(gh) = χ(h)−1 χ(g)−1 = χ(g)−1 χ(h)−1 = φ(g)φ(h) ∀g, h ∈ G
since M is abelian. Then observe that
(χφ)(g) = χ(g)φ(g) = χ(g)χ(g)−1 = 1 = χ(g)−1 χ(g) = (φχ)(g) ∀g ∈ G.
Thus, φ is the inverse element in G.
b
Thirdly, letting ψ, ϕ, ξ ∈ G
b be given, observe that
((ψϕ)ξ)(g) = (ψϕ)(g)ξ(g) = [ψ(g)ϕ(g)]ξ(g) = φ(g)[ϕ(g)ξ(g)] ∀g ∈ G
due to associativity of M .
Therefore, Gb is a group.
Lastly, note that, for given η, ζ ∈ G,
b
(ηζ)(g) = η(g)ζ(g) = ζ(g)η(g) = (ζη)(g) ∀g ∈ G
due to commutativity of M . Therefore, G
b is an abelian group.

(b)
Suppose that G is a finite abelian group. Then there are set of integers n1 , . . . , nr such that
G∼ = Zn × · · · × Zn . 1 r

Let xi ∈ Zni be the generator of each Zni for each i ∈ {1, 2, . . . , r} so that
G∼ = hx1 i × · · · × hxr i .
Without loss of generality, assume that
G = hx1 i × · · · × hxr i .
Let’s define χi to be the homomorphism which sends xi to e2πi/ni and sends xj to 1, for all j 6= i. For each i, note that
ni
χini (g) = (χi (g)) = χi (g ni ) = (1, . . . , gini , . . . , 1) = (1, . . . , e2πia , . . . , 1) = 1G ∀g ∈ G
where g = (g1 , g2 , . . . , gr ) since gi = xai for some integer a. Thus, |χi ||ni . On the other hand, note that
|χi |
χi (g |χi | ) = (χi (g)) = 1G .
Thus, |χi | = ni ∀i ∈ {1, 2, . . . , r} so | hx1 i × · · · × hxr i | = | hχ1 i × · · · × hχr i |.
Let
f : hx1 i × · · · × hxr i → hχ1 i × · · · × hχr i
(xm mr m1 mr
1 , . . . , xr ) 7→ (χ1 , . . . , χr ).
1

is clearly homomorphism since


f ((xm mr u1 ur m1 +u1
1 , . . . , xr )(x1 , . . . , xr )) = (χ1
1
, . . . , χrmr +ur ) = (χm mr u1 ur m1 mr u1 ur
1 , . . . , χr )(χ1 , . . . , χr ) = f ((x1 , . . . , xr ))f ((x1 , . . . , xr )).
1

Also, note that


ker(f ) = {(xm mr mi
1 , . . . , xr ) : χi
1
= 1∀i} = {(xm 1 mr
1 , . . . , xr ) : ni |mi ∀i} = {1G }.
b = hχ1 i × · · · × hχr i, G ∼
Thus, f is an injection homomorphism. Since | hx1 i × · · · × hxr i | = | hχ1 i × · · · × hχr i | and G = G.
b
(∵ G ⊇ hχ1 i × · · · × hχr i is clear and other inclusion is from that every element in G can be represented by product of each χi .)
b b


Page 43
MATH 611 Editor : Byeongho Ban

5.4.1
Prove that if x, y ∈ G then [y, x] = [x, y]−1 . Deduce that for any subsets A and B of G, [A, B] = [B, A](recall that [A, B] is the
subgroup of G generated by the commutators [a, b].)
Proof. (B. Ban) .

By definition, observe that


−1
[y, x] = y −1 x−1 yx = x−1 y −1 xy = [x, y]−1 ∀x, y ∈ G.
By the result, further observe that, for given α ∈ A and β ∈ B,
[α, β] = [β, α]−1 ∈ h[b, a] : a ∈ A, b ∈ Bi = [B, A] =⇒ [A, B] ≤ [B, A]
[β, α] = [α, β]−1 ∈ h[a, b] : a ∈ A, b ∈ Bi = [A, B] =⇒ [B, A] ≤ [A, B].
Therefore, [A, B] = [B, A].

5.4.2
Prove that a subgroup H of G is normal if and only if [G, H] ≤ H .
Proof. (B. Ban) .

Observe that
H E G ⇐⇒ ghg −1 ∈ H ∀g ∈ G ∀h ∈ H
⇐⇒ ghg −1 h−1 ∈ H ∀g ∈ G ∀h ∈ H
⇐⇒ [g −1 , h−1 ] ∈ H ∀g ∈ G ∀h ∈ H
⇐⇒ [G, H] ≤ H.
Thus, we are done.

5.4.6
Exhibit a representative of each cycle type of A5 as a commutator in S5 .
Proof. (B. Ban) .

In A5 , there are four different cycle types, (5), (3 1 1), (2 2 1), (1 1 1 1 1). Observe that
[(1 2 3 4 5), (2 5)(4 3)] = (1 4 2 5 3).
So [(1 2 3 4 5), (2 5)(4 3)] is a representative of the cycle type, (5), of A5 .

Observe that
[(1 2 3),(1 3) ] = (1 2 3).
So [(1 2 3), (1 3)] is the representative of the cycle type, (3 1 1), of A5 .

Observe that
[(1 2), (1 2)] = 1.
So [(1 2), (1 2)] is the representative of the cycle type, (1 1 1 1 1), of A5 .
Lastly, observe that
[(1 3)(2 4), (3 4) ] = (1 2)(3 4).
So [(1 3)(2 4), (3 4) ] is the representative of the cycle type, (2 2 1), of A5 .


Page 44
MATH 611 Editor : Byeongho Ban

October 29th, 2018 Byeongho Ban

7.1.5
Decide which of the following (a) − (f ) are subgrings of Q :
(a) the set of all rational numbers with odd denominators (when written in lowest terms)
(b) the set of all rational numbers with even denominators (when written in lowest terms)
(c) the set of nonnegative rational numbers
(d) the set of squares of rational numbers
(e) the set of all rational numbers with odd numerators (when written in lowest terms)
(f ) the set of all rational numbers with even numerators (when written in lowest terms)
Proof. (B. Ban) .

(a) Yes
Let S1 = {r ∈ Q : the denominator of r is odd when it is written in lowest terms. }.
n
Firstly observe that 1 ∈ S1 so S1 6= ∅. S1 is clearly subgroup since m − pq = qn−pm
qm and any factor of qm is odd if q and m are
n p np
odd. Also it is abelian due to commutative of Q. Also, since m · q = mq and mq is odd if m and q are odd, S1 is closed under
multiplication. Therefore, S1 is a subring of Q.

(b) No
Let S2 = {r ∈ Q : the denominator of r is even. }.
Note that S2 is not a group under addition since 12 + 1
6 = 4
6 = 2
3 6∈ S2 , so it is not closed under addtion. Therefore, S2 is not a
subring.

(c) No
Let S3 = {r ∈ Q : r ≥ 0}.
Note that S3 is not a subgroup under addition since 1 + (−4) = −3 6∈ S3 even if 1, 4 ∈ S3 . Therefore, S3 is not a subring.

(d) No
Let S4 = {r ∈ Q : r = s2 for some s ∈ Q.}.
It is not a subgroup since 22 − 12 = 4 − 1 = 3 6∈ S4 even if 22 , 12 ∈ S4 . Therefore, S4 is not a subgring.

(e) No
Let S5 = {r ∈ Q : the numberator of r is odd}.
Note that 1/1 + 3/1 = 1 + 3 = 4 6∈ S5 . Thus, S5 is not closed under addition, so S5 is not a subgroup so is not a subring.

(f ) No
Let S6 = {r ∈ Q : the numerator of r is even}.
Note that 1/2 + 1/6 = 4/6 = 2/3 6∈ S6 even if 1/2, 1/6 ∈ S6 . Thus, S6 is not closed under addition, so is not a subgroup, so is not
a subring.


Page 45
MATH 611 Editor : Byeongho Ban

7.1.6
Decide which of the following are subrings of the ring of all functions from the closed interval [0, 1] to R :
(a) the set of all functions f (x) such that f (q) = 0 for all q ∈ Q ∩ [0, 1].
(b) the set of all polynomial functions
(c) the set of all functions which have only a finite number of zeros, together with the zero function
(d) the set of all functions which have an infinite number of zeros
(e) the set of all functions f such that limx→1− f (x) = 0.
(f ) the set of all rational linear combinations of the functions sin nx and cos mx, where m, n ∈ {0, 1, 2, . . . }.
Proof. (B. Ban) .

Let F = {f : f : [0, 1] → R}.


(a)
Let F1 = {f ∈ F : f (q) = 0 ∀q ∈ Q ∩ [0, 1]}.
Note that a constant function I(x) = 1 for all x ∈ [0, 1] is in F1 so F1 6= ∅. Note that (f − h)(q) = f (q) − h(q) = 0 ∀f, h ∈ F1 and
∀q ∈ [0, 1] ∩ Q. Thus, F1 is a subgroup of F. Also, note that (f g)(q) = f (q)g(q) = 0 ∀f, g ∈ F1 and q ∈ [0, 1] ∩ Q. Thus, F1 is
closed under multiplication, so is a subring.

(b)
Let F2 = {f ∈ F : f is a polynomial}.
Clearly, constant function is a polynomial so is in F2 . Thus, F2 is nonempty. Also, f − g is also a polynomial for any polynomials
f, g, F2 is a subgroup under addition. Lastly, since any product of any polynomial is a polynomial, it is closed under multiplication
so it a subring.

(c)
Let F3 = {f : [0, 1] → R : f is zero function or f has finite number of zeros.}.
Clearly, 0 ∈ F3 so it not empty. Note that f − g has finite number of zeros if f and g has finite number of zeros. So F3 is a
subgroup of F3 . Also, f · g has also finite number of zeros if f, g ∈ F3 since
{x ∈ [0, 1] : f g(x) = 0} = {x ∈ [0, 1] : f (x) = 0} ∪ {x ∈ [0, 1] : g(x) = 0}.
Thus, F3 is closed under product so is a subring.

(d)
Let F4 = {f ∈ F : f has infinitely many zeros}.
Note 0 ∈ F4 so F4 6= ∅. Note that f − g has infinitely many zeros if f, g ∈ F4 . Also, f · g has infinitely many zeros. Therefore, F4
is a subring.

(e)
Let F5 = {f ∈ F : limx→1− f (x) = 0}.
Note that 0 ∈ F5 so F5 6= ∅. Also, note that
lim (f − g) = lim f (x) − lim g(x) = 0 ∀f, g ∈ F5 =⇒ f − g ∈ F5
x→1− x→1− x→1−
lim (f g)(x) = lim f (x) lim g(x) = 0 ∀f, g ∈ F5 =⇒ f g ∈ F5 .
x→1− x→1− x→1−
Therefore, F5 is a subgroup and closed under product. Thus, F5 is a subring.

(f )
Let F6 = {f ∈ F : f is a rational linear combinations of the functions sin nx and cos mx, where m, n ∈ {0, 1, 2, . . . }}.
Note that 0 ∈ F6 , so F6 6= ∅. Also, note that sin nx cos mx = sin(nx + mx) + sin(nx − mx), cos nx cos mx = cos(nx + mx) +
cos(nx − mx), sin nx sin mx = cos(nx − mx) − cos(nx + mx) ∈ F6 . Then note that an any subtraction of two rational linear
combinations is also rational linear combination due to the argument above. And also F6 is closed under product due to the
argument above. Therefore, F6 is a subring.


Page 46
MATH 611 Editor : Byeongho Ban

7.1.7
The center of a ring R is {z ∈ R : zr = rz for all r ∈ R}(i.e, is the set of all elements which commute with every element of R. )
Prove that the center of a ring is a subring that contains the identity. Prove that the center of a division ring is a field.
Proof. (B. Ban) .

Let the center of R be Z(R).


Note that 1r = r1 for all r ∈ R so 1 ∈ R, so R 6= ∅. Observe that, for given z1 , z2 ∈ Z(R),
(z1 − z2 )r = z1 r − z2 r = rz1 − rz2 = r(z1 − z2 ) ∀r ∈ R =⇒ z1 − z2 ∈ R
z1 z2 r = z1 rz2 = rz1 z2 ∀r ∈ R =⇒ z1 z2 ∈ R.
Therefore, the center of R is a subring containing identity.

Suppose that R is a division. In order to show that R is a field, it suffices to prove that the center of R is a division ring and is
commutative. Let z ∈ Z(R) be given. Since R is a division ring, there is w ∈ R such that zw = wz = 1. Then note that
rz = zr ⇐⇒ r = zrw ⇐⇒ wz = zw ∀r ∈ R.
Thus, w ∈ Z(R), so Z(R) is closed under multiplicative inverse, so is a division ring. And note that for given z, w ∈ Z(R), note
that zw = wz since z ∈ Z(R) and w ∈ R. Thus, Z(R) is commutative. Therefore, Z(R) is a field.

7.1.8
Describe the center of the real Hamiltonian Quaternion H. Prove that {a + bi : a, b ∈ R} is a subring of H which is a field but is
not contained in the center of H.
Proof. (B. Ban) .

Let the center of the real Hamiltonian Quaternion be Z(H). For any z ∈ Z(H), it should be
iz = zi zj = jz kz = zk
And only elements satisfying the condition above are 1 and −1. Thus, Z(H) = {1, −1}.

Let S = {a + bi : a, b ∈ R}.

Now observe that 1 ∈ S, so S 6= ∅ and that


(a + bi) − (c + di) = (a − c) + (b − d)i ∈ S ∀a, b, c, d ∈ R
(a + bi)(c + di) = (ac − bd) + (ad + bc)i ∈ S ∀a, b, c, d ∈ R.
Thus, S is a subgroup with respect to addition and is closed under multiplication. Therefore, S is a subring.

And note that


a − bi a b
(a + bi)−1 = = 2 + 2 i ∈ S ∀a, b ∈ R
a2 + b2 a + b2 a + b2
(a + bi)(c + di) = (ac − bd) + (ad + bc)i = (c + di)(a + bi) ∀a, b, c, d ∈ R.
Therefore, S is closed under multiplicative inverse and commutative. Thus, S is a field.

And note that i ∈ S and i 6∈ Z(H). So S 6⊆ Z(H).




Page 47
MATH 611 Editor : Byeongho Ban

7.1.13
An element x in R is called nilpotent if xm = 0 for some m ∈ Z+ .
(a) Show that n = ak b for some integers a and b then ab is a nilpotent element of Z/nZ.
(b) If a ∈ Z is an integer, show that the element a ∈ Z/nZ is nilpotent if and only if every prime divisor of n is also a divisor of
a. In particular, determine the nilpotent elements of Z/72Z explicitly.
(c) Let R be the ring of functions from a nonempty set X to a field F . Prove that R contains no nonzero nilpotent elements.
Proof. (B. Ban) .

(a)
Observe that
(ab)k = ak bak−1 = nbk−1 ≡ 0 (mod n).
Thus, ab is nilpotent element of Z/nZ.

(b)
Suppose that a ∈ Z/nZ is nilpotent. Then ∃k ∈ Z such that
ak ≡ 0 (mod n).
It implies that ak = nm for some m ∈ Z. Note that if q is a prime divisor of n, q|ak so q|a. Therefore, every prime divisor of n is
also divisor of a.

Conversely, suppose that every prime divisor of n is also divisor of a. Note that, by fundamental theorem of arithmetic, n =
pk11 . . . pknn for some prime p1 , . . . , pn and some k1 , . . . , kn ∈ Z. Let k = maxk∈{1,...,n} kn . Also, since every prime divisor of n is
also divisor of a, note that a = p1 . . . pn b for some b ∈ Z and observe that
ak = pk11 . . . pknn p1k−k1 . . . pk−k
n b = npk−k
n k
1
1
. . . pk−k
n
n k
b ≡ 0 (mod n).
Thus, a is nilpotent element of Z/nZ.

In particular, note that 72 = 23 32 . According to the argument above, a is a nilpotent if and only if 2|a and 3|a. Thus, for any m,
a multiple of 6, m is a nilpotent element.
Thus, the twelve elements, 6, 12, 18, 24, 30, 36, 42, 48, 54, 60, 66, 72. are all and only nilpotent elements of Z/72Z.

(c)
Let R = {f : X → F } be the ring of functions where X is nonempty set and F is a field. Suppose that g ∈ R be a non zero
nilpotent element. Then ∃k ∈ Z such that f (x)k = 0 for any x ∈ X which means that f (x) = 0 ∀x ∈ X. Thus, f is zero function
and it is a contradiction. Therefore, there is no nonzero nilpotent element in R.


Page 48
MATH 611 Editor : Byeongho Ban

7.1.14
Let x be a nilpotent element of the commutative ring R (cf. the preceding exercise).
(a) Prove that x is either zero or a zero divisor
(b) Prove that rx is nilpotent for all r ∈ R.
(c) Prove that 1 + x is a unit in R.
(d) Deduce that the sum of a nilpotent element and a unit is a unit.
Proof. (B. Ban)
(a)
Since x is nilpotent, ∃k ∈ Z+ such that xk = 0. If x 6= 0, then note that xxk−1 = 0 thus, x and xk−1 is zero divisor. Thus, x is
either zero or zero divisor.

(b)
Observe that, since R is commutative, (rx)k = rk xk = rk 0 = 0 ∀r ∈ R. Thus, rx is nilpotent.

(c)
Observe that
k
X k
X k+1
X
(1 + x)( (−x)i ) = (−x)i + −(−x)i
i=0 i=0 i=1
k
X k
X
= (−x)i + −(−x)i ∵ xk+1 = 0,
i=0 i=1
= 1.
Pk i
Since i=0 (−x) ∈ R, (1 + x) is a unit.

(d)
Let n be a nilpotent element and u be a unit. Since u is a unit, ∃w ∈ R such that uw = wu = 1. And observe that
u + n = u(wu + wn) = u(1 + wn).
Then by (b), wn is nilpotent, and by (c), 1 + wn is a unit. Since u is a unit, u(1 + wn) is also a unit. Therefore, u + n, the sum
of a unit and a nilpotent element is a unit.


Page 49
MATH 611 Editor : Byeongho Ban

7.1.23 √
Let D be a squarefree integer, and let O be the ring of integers in the quadratic field Q( D). For any positive integer f , prove
that the set Of = Z[f w] = {a + bf w : a, b ∈ Z} is a subring of O containing the identity. Prove that [O : Of ] = f (index as
additive abelian groups.) Prove conversely that a subring of O containing the identity and having
√ finite index f in O (as additive
abelian group) is equal to Of . √(The ring Of is called the order of conductor f in the field Q( D)). The ring of integers O is
called the maximal order in Q( D).)
Proof. (B. Ban) .

(a) Prove that the set Of = Z[f w] = {a + bf w : a, b ∈ Z} is a subring of O containing the identity.
(P roof )
Note that 1 = 1 + 0f w ∈ Of , so Of 6= ∅. And observe that
(a + bf w) − (c + df w) = (a − c) + (b − d)f w ∈ Of
(a + bf w)(c + df w) = ac + bdf 2 w2 + adf w + cbf w.
Thus, Of is a subgroup of O. And observe the cases

D ≡ 2, 3 (mod 4) =⇒ w = D =⇒ (a + bf w)(c + df w) = (ac + bdDf 2 ) + (ad + cb)f w ∈ Of
√ √
1+ D adf 2 (1 + 2 D + D)
D ≡ 1 (mod 4) =⇒ w = =⇒ (a + bf w)(c + df w) = ac + + (ad + cb)f w
2 4
2

adf
= ac + (D − 1) + (ad + cb + adf )f w ∈ Of .
4
Thus, Of is closed under multiplication so is a subring of O containing identity.

(b) Prove that [O : Of ] = f (index as additive abelian groups.)


(P roof )
Let ϕ : Z × Z → O defined by
ϕ(a, b) = a + bw.
Note that ϕ is clearly a bijection since
ker(ϕ) = {(a, b) ∈ Z × Z : a + bw = 0} = {(0, 0)},
and
∀a + bw ∈ O, ϕ(a, b) = a + bw and (a, b) ∈ Z × Z. Also, for any a, b, c, d ∈ Z, observe that
ϕ((a, b) + (c, d)) = (a + c) + (b + d)w = a + bw + c + dw = ϕ(a, b) + ϕ(c, d).
Thus, ϕ is a group isomorphism. Therefore, Z × Z ∼ = O. And note that Z × Z and Of are abelian group under addition and
Z × fZ ∼
= Of
by the same isomorphism ϕ. Therefore, note that
(Z × Z)/(Z × f Z) ∼
= O/Of .
Therefore, observe that
[O : Of ] = |O/Of | = |(Z × Z)/(Z × f Z)| = |Z/f Z| = f.
(Note: every additive subgroup of additive abelian group is normal subgroup, so defining index has no problem.)
(c) Prove that a subring of O containing the identity and having finite index f in O is equal to Of .
(P roof )
Suppose that S be the subring of O that contains the identity and have finite index f in O. Then the only possibility for S is to
be one of Of and
Of = {f a + bw : a, b ∈ Z}
Off = {f a + bf w : a, b ∈ Z}
by the index condition since they should be isomorphic(as a additive abelian group) to one of Z × f Z, f Z × Z, and f Z × f Z.
However, note that, for any a, b, c, d ∈ Z,
(f a + bw)(f c + dw) = f 2 ac + adf w + bcf w + bdw2 6∈ Of (∵ bdw2 is not a form of bw or f a.)
so Of is not a subring and 1 6∈ Off , so Off does not contain the identity. Therefore, the only subring of O that contains the identity
and has finite index f is Of .


Page 50
MATH 611 Editor : Byeongho Ban

7.1.26
Let K be a field. A discrete valuation on K is a function ν : K x → Z satisfying
(i) ν(ab) = ν(a) + ν(b) (i.e, ν is a homomorphism from the multiplicative group of nonzero elements of K to Z),
(ii) ν is surjective, and
(iii) ν(x + y) ≥ min(ν(x), ν(y)) ∀x, y ∈ K x with x + y 6= 0.
x
The set R = {x ∈ K : ν(x) ≥ 0} ∪ {0} is called the valuation ring of ν.
(a) Prove that R is a subring of K which contains the identity. (In general, a ring R is called a discrete valuation ring if there is
some field K and some discrete valuation ν on K such that R is the valuation ring of ν.)
(b) Prove that for each nonzero element x ∈ K either x or x−1 is in R.
(c) Prove that an element x is a unit of R if and only if ν(x) = 0.
Proof. (B. Ban) .

(a)
Note that ν(1) = ν(1 · 1) = ν(1) + ν(1),so ν(1) = 0. Thus, 1 ∈ R and R 6= ∅. Also, observe that
0 = ν(1) = ν(−1) + ν(−1) =⇒ ν(−1) = −ν(−1) =⇒ ν(−1) = 0,
ν(x + (−y)) ≥ min{ν(x), ν(−y)} = min{ν(x), ν(y)} =⇒ ν(x + (−y)) ≥ 0 ∀x, y ∈ R with x 6= y,
ν(xy) = ν(x) + ν(y) ≥ 0
Since x − x = 0 ∈ R, with the observation above, x − y ∈ R for any x, y ∈ R, so R is a subgroup. Also, note that, by the
observation above, xy ∈ R. Therefore, R is a subring.

(b)
Let x ∈ K \ {0} be given. And observe that
0 = ν(1) = ν(xx−1 ) = ν(x) + ν(x−1 ) =⇒ ν(x) ≥ 0 or ν(x−1 ) ≥ 0.
Since, otherwise, 0 = ν(x) + ν(x−1 ) < 0 and it is a contradiction. Therefore, either x ∈ R or x−1 ∈ R.

(c)
Suppose that x is a unit. Then there is x−1 such that xx−1 = x−1 x = 1. Then observe that
0 = ν(1) = ν(xx−1 ) = ν(x) + ν(x−1 )
Since x, x−1 ∈ R so since ν(x) + ν(x−1 ) ≥ 0, by the observation above, ν(x) = ν(x−1 ) = 0.

Conversely, for x ∈ R if ν(x) = 0, then by the last argument, ν(x−1 ) = 0, so x−1 ∈ R. Therefore, x is a unit of R.

7.2.2
Let p(x) = an xn + an−1 xn−1 + · · · + a1 x + a0 be an element of the polynomial ring R[x]. Prove that p(x) is a zero divisor in
R[x] if and only if there is a nonzero b ∈ R such that bp(x) = 0. [Let g(x) = bm xm + bm−1 xm−1 + · · · + b0 be a nonzero
polynomial of minimal degree less than m that also gives 0 when multiplied by p(x). Conclude that an g(x) = 0. Apply a similar
to show by induction on i that an−1 g(x) = 0 for i = 0, 1, . . . , n, and show that this implies bm p(x) = 0.]
Proof. (B. Ban) .
Pn Pm
Assume that ∀r ∈ R\{0}, rp(x) 6= 0 . Since p(x) = k=0 ak xk is a zero divisor in R[x], we can choose q(x) = j=0 bj xj ∈ R[x]\{0}
such that q(x) is having the minimal degree of m satisfying p(x)q(x) = 0. Here, m > 0 since if m = 0, we have b0 ∈ R[x] \ {0}
and b0 p(x) = 0 which is a contradiction to our assumption.
Observe that
p(x)q(x) = an bm xn+m + · · · + a0 b0 = 0 =⇒ an bm = 0.
Thus, we have a polynomial an q(x) of degree m − 1. Let h(x) = an q(x) and note that h(x) ∈ R[x]. Now observe that
p(x)h(x) = p(x)(an q(x)) = an (p(x)q(x)) = 0.
Thus, h(x) ∈ R[x] is the polynomial of degree lower than m satisfying the condition p(x)h(x) = 0 and it contradicts to your
selection of q(x). Therefore, ∃b ∈ R such that bp(x) = 0.

Conversely, if bp(x) = 0 for some b ∈ R \ {0}, since b ∈ R[x], p(x) is a zero divisor in R[x].


Page 51
MATH 611 Editor : Byeongho Ban

7.2.3
Define the set R[[x]] of formal power series in the indeterminate x with coefficients from R to be all formal infinite sums
X∞
an xn = a0 + a1 x + · · ·
n=0
Define addition and multiplication of power series in the same way as for power series with real or complex coefficients i.e, extend
polynomial addition and multiplication to power series as though they were ”polynomials of infinite degree”:


X ∞
X ∞
X
an xn + bn x n = (an + bn )xn
n=0 n=0 n=0
∞ ∞ ∞ n
!
X X X X
n n
an x × bn x = ak bn−k xn .
n=0 n=0 n=0 k=0
(The term ”formal” is used here to indicate that convergence is not considered, so that formal power series need not represent
functions on R.)
(a) Prove that R[[x]] is a commutative ring with 1.
(b) Show that 1P− x is a unit in R[[x]] with inverse 1 + x + x2 + · · · .

(c) Prove that n=0 an xn is a unit in R[[x]] if and only if a0 is a unit in R.
Proof. (B. Ban) .

(a) P∞ P∞ P∞
Note that 1 = n=0 an xn ∈ R[[x]] with a0 = 1 and an = 0 for all n > 0. And further note that, for any n=0 an xn , n=0 bn xn ∈
R[[x]],
X∞ X∞ ∞
X
an xn − bn x n = (an − bn )xn ∈ R[[x]] ∵ an − bn ∈ R ∀n
n=0 n=0 n=0
∞ ∞ ∞ n
! n
X X X X X
n n n
an x × bn x = ak bn−k x ∈ R[[x]] ∵ ak bn−k ∈ R.
n=0 n=0 n=0 k=0 k=0
P∞ P∞
Thus, R[[x]] is a subgroup closed under multiplication, so is a subring. Moreover, observe that, for any n=0 an xn , n=0 bn xn ∈
R[[x]],
∞ ∞ ∞ ∞ ∞ ∞
n
! n
!
X X X X X X X X
n n n
an x × bn x = ak bn−k x = bk an−k xn = bn x n × an xn .
n=0 n=0 n=0 k=0 n=0 k=0 n=0 n=0

Therefore, R[[x]] is a commutative ring with 1.

(b)
Observe that

X ∞
X ∞
X ∞
X ∞
X ∞
X
(1 − x) xn = xn − xn+1 = xn − xn = 1 + (1 − 1)xn = 1.
n=0 n=0 n=0 n=0 n=1 n=1
P∞ n
Thus, (1 − x) is a unit with the inverse n=0 x .

(c) P∞ P∞
Suppose that n=0 an xn is a unit in R[[x]]. Then, there is n=0 bn xn ∈ R[[x]] such that
∞ ∞ ∞ n
!
X X X X
n n
an x × bn x = ak bn−k xn = 1.
n=0 n=0 n=0 k=0
It means that a0 b0 = 1. Therefore, a0 is a unit.

Conversely, suppose that a0 is a unit in R. Then there is b0 ∈ R such that a0 b0 = 1. Let b1 ∈ R such that a0 b1 + a1 b0 = 0 so
b1 = −aa10b0 . Suppose that we have choosen bk for all k ≤ n. Then we choose bn+1 such that
n+1 Pn+1
X − j=1 aj bn+1−j
aj bn+1−j = 0 so that bn+1 = .
j=0
a0
Thus,
∞ ∞ ∞ ∞
n
!
X X X X X
n n
an x × bn x = ak bn−k xn = 1 =⇒ an xn ∈ R[[x]] is a unit. .
n=0 n=0 n=0 k=0 n=0

Page 52
MATH 611 Editor : Byeongho Ban

7.2.7
Prove that the center of the ring Mn (R) is the set of scalar matrices (cf. Exercise 7, Section 1).[Use the preceding exercise.]
Proof. (B. Ban) Let Z(Mn (R)) be the center of the ring Mn (R).
Let Eij be the element of Mn (S) whose i, j entry is 1 and whose other entries are all 0.
Observe that, if A ∈ Z(Mn (R)) if and only if
AEii = Eii A.
Note that
 
0
 .. 
 . 
   
AEii = 0 ··· A∗i ··· 0 Ai∗ 
Eii A =  
 . 
 .. 
0
where Ai∗ is i th row of A and A∗i is i th column of A. Therefore, if AEii = Eii A, then Aij = 0 if i 6= j. Further observer that,
for i 6= j with 1 ≤ i, j ≤ n,
 
0
 .. 
  
 . 

AEij = Eij A =⇒ 0 · · · A∗i (j th ) · · · 0 = Aj∗ (i th )

.
 .. 
 . 
0
It implies that Ajj = Aii for any i, j. Therefore, if A ∈ Z(Mn (R)), we should have A = αIn where α ∈ R and In is an n × n
identity matrix. Since αIn commutes with every n × n matrices,
Z(Mn (R)) = {αIn : α ∈ R}.

7.3.6
Decide which of the following are ring homomorphisms from M2 (Z) to Z:
(a)
 
a b
7→ a ( projection onto the 1,1 entry)
c d
(b)
 
a b
7→ a + d ( the trace of the matrix)
c d
(c)
 
a b
7→ ad − bc ( the determinant of the matrix)
c d
Proof. (B. Ban) .

Let the mapping be ϕa . Then observe that


        
1 1 1 1 4 4 1 1 1 1
ϕa = ϕa = 4 6= 1 = ϕa ϕa
2 2 3 3 8 8 2 2 3 3
Therefore, ϕa is not a ring homomorphism.

(b)
Let the mapping be ϕb . Then observe that
        
1 3 1 1 10 10 1 3 1 1
ϕa = ϕa = 21 6= 12 = 3 · 4 = ϕa ϕa
5 2 3 3 11 11 5 2 3 3
Therefore, ϕb is not a ring homomorphism.

(c)
Let the mapping be ϕc . Then observe that
         
2 1 1 0 3 1 2 1 1 0
ϕa + = ϕa = 6 6= 2 = ϕa + ϕa .
3 2 0 1 3 3 3 2 0 1
Therefore, ϕc is not a ring homomorphism.

Page 53
MATH 611 Editor : Byeongho Ban

7.3.7
Let
  
a b
R= : a, b, d ∈ Z
0 d
be the subring of M2 (Z) of upper triangular matrices. Prove that the map
 
a b
ϕ : R → Z × Z defined by ϕ : 7 (a, d)

0 d
is a surjective homomorphism and describe its kernel.
Proof. (B. Ban)
Let a, b, c, d, e, f ∈ Z be given and observe that
         
a b d e a+d b+e a b d e
ϕ + =ϕ = (a + d, c + f ) = (a, c) + (d, f ) = ϕ +ϕ
0 c 0 f 0 c+f 0 c 0 f
        
a b d e ad ae + bf a b d e
ϕ =ϕ = (ad, cf ) = (a, c)(d, f ) = ϕ ϕ
0 c 0 f 0 cf 0 c 0 f
Therefore, ϕ is a ring homomorphism. And observe that, for any (a, b) ∈ Z × Z,
   
a 0 a 0
ϕ = (a, b) and ∈ R.
0 b 0 b
Thus, ϕ is a surjection. Lastly, note that
     
a b 0 a
ker(ϕ) = ∈R:a=d=0 = :a∈Z .
0 d 0 0

7.3.9
Decide which of the sets in Exercise 6 of Section 1 are ideals of the ring of all functions from [0, 1] to R.
Proof. (B. Ban)
I follow the definition of each sets from Exercise 7.1.6.

First of all, note that all sets from F1 to F6 are subgrings. Therefore, we only need to check if rFi ⊆ Fi and Fi r ⊆ Fi for all
i ∈ {1, 2, 3, 4, 5, 6}.
(a)
Observe that, for any r ∈ F, and any f ∈ F1 ,
r(q)f (q) = r(q) · 0 = 0 = 0 · r(q) = f (q)r(q) ∀q ∈ Q ∩ [0, 1].
So, rf, f r ∈ F1 .
Thus, rF1 ⊆ F1 and F1 r ⊆ F1 , so F1 is an ideal.

(b)
Observe that sin(x) ∈ F and 1 ∈ F2 . However, sin(x) · 1 6∈ F2 . Therefore, sin(x)F2 6⊆ F2 . Therefore, F2 is not an ideal.

(c)
Similarly, observe that sin(x) ∈ F and 1 ∈ F3 . However, 1 · sin(x) 6∈ F3 . So F3 sin(x) 6⊆ F3 . Therefore, F3 is not an ideal.

(d)
Let r ∈ F and f ∈ F4 be given. And observe that rf (x) = r(x)f (x) = 0 and f r(x) = f (x)r(x) = 0 for infinitely many x ∈ [0, 1]
since f (x) = 0 for infinitely many x ∈ [0, 1]. Therefore, rf, f r ∈ F4 so rF4 ⊆ F4 and F4 r ⊆ F4 for any r ∈ F. Thus, F4 is an ideal.

(e)
1
Note that, sin(x − 1) ∈ F5 and 1−x ∈ F. And observe that
1
lim ( ) sin(x − 1) = 1.
x→1− 1−x
1
Thus, 1−x F5 6⊆ F5 . Therefore, F5 is not an ideal.

(f )
Note that sin(x) ∈ F6 and ex ∈ F. However, observe that ex sin(x) 6∈ F6 since we cannot express it only in terms of sin and cos.
Therefore, F6 is not an ideal.


Page 54
MATH 611 Editor : Byeongho Ban

7.3.10
Decide which of the following are ideals of the ring Z[x]:
(a) the set of all polynomials whose constant term is a multiple of 3.
(b) the set of all polynomials whose coefficient of x2 is a multiple of 3.
(c) the set of all polynomials whose constant term, coefficient of x and coefficient of x2 are zero.
(d) Z[x2 ] (i.e, the polynomials in which only even powers of x appear)
(e) the set of polynomials whose coefficients sum to zero.
(f ) the set of polynomials p(x) such that p0 (0) = 0, where p0 (x) is the usual first derivative of p(x) with respect to x.
Proof. (B. Ban) .

Note that Z[x] is commutative ring with identity. Thus, in order to check if a subring is an ideal, it suffices to show that it is only
one of left or right ideal. And let the coefficient of xk of p be C(p, k).
(a)
Let
P1 = {p ∈ Z[x] : the constant term of p is the multiple of 3.}
Observe that 0 · 3 = 0 ∈ P1 , so P1 6= ∅ . And observe note that the constant term of any subtraction between any two polynomials
in P1 has the multiple of 3 and that of multiple of the two polynomials is also 3. Therefore, P1 is a subring. And lastly, observe
that for any q ∈ Z[x] and p ∈ P1 , the constant term of pq is the multiple of the constant terms of p and q. Thus, the constant
term of pq is multiple of 3(3 is from the constant term of p.) Therefore, P1 a = qP1 ⊆ P1 since P1 is commutative. Thus, P1 is an
ideal.

(b)
Let
P2 = {p ∈ Z[x] : C(p, 2) ≡ 0 (mod 3)}.
2 2
Note that C((3x + x + 1)(3x + 4x + 2), 2) = 13 6≡ 0 (mod 3). Thus, P2 is not closed under multiplication, so is not an subring,
so is not an ideal.

(c)
Let
P3 = {p ∈ Z[x] : C(p, 0) = C(p, 1) = C(p, 2) = 0}.
Note that, 0 ∈ P3 , and that, for any p, q ∈ P3 , the lowest possible term of (p − q) is x3 and that of pq is x9 . Thus, p − q, pq ∈ P3 .
Therefore, P3 is a subring. Furthermore, note that for any r ∈ Z[x], the lowest possible term of rp is x3 for all p ∈ P3 . Thus,
rp ∈ P3 ∀p ∈ P3 which implies rP3 ⊆ P3 for all r ∈ Z[x]. Therefore, P3 is an ideal.

(d)
Note that x2 ∈ Z[x] and x ∈ Z[x]. But x · x2 = x3 6∈ Z[x2 ] which means that xZ[x2 ] 6⊆ Z[x]. Therefore, Z[x2 ] is not an ideal.

(e)
Let
 
 Np 
X
P5 = p ∈ Z[x] : C(p, n) = 0 where Np = deg(p) .
 
n=0

Note that 0 ∈ P5 , so P5 6= ∅. Let p, q ∈ P5 be given. And observe that


Np−q Np Nq
X X X
C(p − q, n) = C(p, n) − C(q, n) = 0 − 0 = 0
n=0 n=0 n=0

Thus, P5 is a subgroup. And let g ∈ Z[x] be given. And note that


  
Npg Ng Np Ng
X X X X
C(pg, n) = C(g, n)  C(p, k) = [C(g, n)0] = 0 ∀p ∈ P5 .
n=0 n=0 k=0 n=0

Thus, P5 g ∈ P5 , so P5 is an subring and so an ideal.

Page 55
MATH 611 Editor : Byeongho Ban

(f )
Let
P6 = {p ∈ Z[x] : p0 (0) = 0}.
Note that (x2 + 1) ∈ P6 and x + 1 ∈ Z[x] then let f (x) = (x2 + 1)(x + 1). And observe that
0
(x + 1)(x2 + 1) = (x2 + 1) + (x + 1)(2x).


Then we have f 0 (0) = 1 6= 0. Therefore, f 6∈ P6 so P6 is not an ideal.


7.3.12
Let D be an integer that is not a perfect square in Z and let
  
a b
S= : a, b ∈ Z .
Db a
(a) Prove that S is a subring of M2 (Z).
√ √
 
a b
(b) If D is not a perfect square in Z prove that the map ϕ : Z[ D] → S defined by ϕ(a + b D) = is a ring isomorphism.
Db a
(c) If D ≡ 1 (mod 4) is squarefree, prove that the set
  
a b
(D−1)b : a, b ∈ Z
4 a+b
is a subring of M2 (Z) and is isomorphic to the quadratic integer ring O.
Proof. (B. Ban) .

(a)
Note that the 2 × 2 identity map I2 is in S, so S is nonempty. Observe that, for any a, b, c, d ∈ Z,
     
a b c d a−c b−d
− = ∈S
Db a Dd c D(b − d) a − c
    
a b c d ac + Dbd ad + bc
= ∈ S.
Db a Dd c D(bc + ad) Dbd + ac
Thus, S is a subring.

(b)
Observe that, for any a, b, c, d ∈ Z,
√ √ √
 
a+c b+d
ϕ((a + b D) + (c + d D)) = ϕ((a + c) + (b + d) D) =
D(b + d) a + c
√ √
   
a b c d
= + = ϕ(a + b D) + ϕ(c + d D)
Db a Dd c
√ √ √
 
ac + Dbd ad + bc
ϕ((a + b D)(c + d D)) = ϕ((ac + bdD) + (ad + bc) D) =
D(bc + ad) Dbd + ac
√ √
  
a b c d
= = ϕ(a + b D)ϕ(c + d D).
Db a Dd c
Therefore, ϕ is a ring homomorphism.

Furthermore observe that


√ √ √
   
a b
ker(ϕ) = a + b D ∈ Z[ D] : ϕ(a + b D) = = 0 = {0}.
Db a

   
a b a b
It implies that ϕ is an injection. Furthermore, for given ∈ S, ϕ(a + b D) = . Therefore, ϕ is a surjection.
Db a Db a
Therefore, ϕ is an isomorphism.

Page 56
MATH 611 Editor : Byeongho Ban

(c)
Let
  
a b
W = (D−1)b : a, b ∈ Z .
4 a+b
Note that I2 ∈ W , so W 6= ∅. Observe that, for any a, b, c, d ∈ Z,
     
a b c d (a − c) (b − d)
(D−1)b − (D−1)d = (D−1)(b−d) ∈W
4 a+b 4 c+d 4 (a − c) + (b − d)
" #
ac + (D−1)bd
  
a b c d 4 ad + b(c + d)
(D−1)b = (D−1)(bc+d(a+b)) ∈ W.
4 a + b (D−1)d
4 c+d 4
(D−1)bd
4 + (a + b)(c + d)
Thus, W is a subring of M2 (Z). And let ϕ : O → W defined by
 
a b
ψ(a + bw) = (D−1)b ,
4 a+b

1+ D
where w = 2 . Observe that, for any a, b, c, d ∈ Z,
     
a+c b+d a b c d
ψ((a + bw) + (c + dw)) = (D−1)(b+d) = (D−1)b + (D−1)d = ψ(a + bw) + ψ(c + dw)
4 a+c+b+d 4 a+b 4 c+d
and that
√ ! !   
1+D+2 D D−1
ψ((a + bw)(c + dw)) = ψ ac + bd + (ad + bc)w = ψ ac + bd + (ad + bc + bd)w
4 4
ac + bd D−1
 
4 ad + bc + bd
= (D−1)
4 (ad + bc + bd) ac + bd D−14 + ad + bc + bd
  
a b c d
= (D−1)b (D−1)d = ψ(a + bw)ψ(c + dw).
4 a+b 4 c+d
Therefore, ψ is a homomorphism.

Also, observe that


   
b a
ker(ψ) = a + bw ∈ O : ψ(a + bw) = = 0 = {0}.
(D−1)b
a+b 4
   
a b a b
It implies that ψ is an injection. And also, for any (D−1)b ∈ W , ψ(a + bw) = (D−1)b , it is a surjection.
4 a + b 4 a+b
Therefore, ψ is an isomorphism. Thus, W and O are isomorphic.

Page 57
MATH 611 Editor : Byeongho Ban

7.3.13
Prove that the ring M2 (R) contains a subring that is isomorphic to C.
Proof. (B. Ban) .
Let
  
a −b
S= : a, b ∈ R .
b a
Note that an identity matrix I ∈ S, so S =
6 ∅. And observe that, for any a, b, c, d ∈ R,
     
a −b c −d a − c −(b − d)
− = ∈S
b a d c b−d a−c
    
a −b c −d ac − bd −ad − bc
= ∈ S.
b a d c bc + ad −bd + ac
Thus, S is a subgroup closed under multiplication, so is a subring of M2 (R).
And then define ϕ : S → C such that
 
a −b
ϕ = a + bi.
b a
Then observe that, for any a, b, c, d ∈ R,
         
a −b c −d a + c −(b + d) a −b c −d
ϕ + =ϕ = (a + c) + (b + d)i = (a + bi) + (c + di) = ϕ +ϕ
b a d c b+d a+c b a d c
        
a −b c −d ac − bd −ad − bc a −b c −d
ϕ =ϕ = (ac − bd) + (bc + ad)i = (a + bi)(c + di) = ϕ ϕ .
b a d c bc + ad −bd + ac b a d c
Thus, ϕ is a ring homomorphism. And observe that
  
a −b
ker(ϕ) = ∈ S : a + bi = 0 .
b a
Note that ker(ϕ) = {0} since a + bi = 0 =⇒ a = 0 and b = 0. Thus, ϕ is an injection. Furthermore, note that, for any a + bi ∈ C,
 
a −b
ϕ = a + bi.
b a
Thus, ϕ is a surjection. Therefore, ϕ is an isomorphism.

7.3.20
Let I be an ideal of R and let S be a subring of R. Prove that I ∩ S is an ideal of S. Show by example that not every ideal of a
subring S of a ring R need be of the form I ∩ S for some ideal I of R.
Proof. (B. Ban) .

Note that 0 ∈ I ∩ S, so I ∩ S 6= ∅. And, for any r, s ∈ I ∩ S, observe that note that r − s, rs ∈ I and r − s, rs ∈ S since I and S
are subrings. So r − s, rs ∈ I ∩ S. Thus, I ∩ S is a subring.
Let s ∈ S be given. Suppose that r ∈ I ∩ S. Since r ∈ I, sr, rs ∈ I. Also, note that rs = sr ∈ S since R is commutative and S is
a subring. Thus, rs, sr ∈ I ∩ S. Therefore, s(I ∩ S) ⊆ I ∩ S and (I ∩ S)s ⊆ I ∩ S, so (I ∩ S) is an ideal.

For instance, note that Z is a subring of R. And note that only ideal of R is a principal ideal (0). Observe that (2) is an ideal of
Z. However, (2) 6= (0) ∩ Z.


Page 58
MATH 611 Editor : Byeongho Ban

7.3.22
Let a be an element of the ring R.
(a) Prove that {x ∈ R : ax = 0} is a right ideal and {y ∈ R : ya = 0} is a left ideal (called respectively the right and left
annihilators of a in R).
(b) Prove that if L is a left ideal of R then {x ∈ R : xa = 0 for all a ∈ L} is a two-sided ideal (called the left annihilator or L in
R.)
Proof. (B. Ban) .

Let

AR = {x ∈ R : ax = 0} AL = {y ∈ R : ya = 0}.
(a)
Note that 0 ∈ AR and 0 ∈ AL , so they are nonempty. Now observe that
x, y ∈ AR =⇒ a(x − y) = ax − ay = 0 =⇒ x − y ∈ AR
x, y ∈ AL =⇒ (x − y)a = xa − ya = 0 =⇒ x − y ∈ AL
x, y ∈ AR =⇒ a(xy) = axy = 0y = 0 =⇒ xy ∈ AR
x, y ∈ AL =⇒ (xy)a = x(ya) = x0 = 0 =⇒ xy ∈ AL .
Thus, AL and AR are both subrings. Also, note that for any r ∈ R, observe that
x ∈ AR =⇒ axr = 0r = 0 =⇒ xr ∈ AR
x ∈ AL =⇒ rxa = r0 = 0 =⇒ rx ∈ AL
Thus, rAL ⊆ Al , so AL is a left ideal and AR x ⊆ AR , so AR is a right ideal.

(b)
Let
H = {x ∈ R : xa = 0 for all a ∈ L}.
Note that 0 ∈ H so H 6= ∅. And note that for any x, y ∈ H,
(x − y)a = xa − ya = 0 ∀a ∈ L =⇒ x − y ∈ H
xya = x0 = 0 ∀a ∈ L =⇒ xy ∈ H.
Thus, H is a subring.
Observe that, for any r ∈ R,
x ∈ H =⇒ rxa = r0 = 0 =⇒ rx ∈ H
x ∈ H =⇒ xra = x(ra) = 0 ∀a ∈ L (∵ L is a left ideal so ra ∈ L) =⇒ xr ∈ H.
Thus, rH ⊆ H and Hr ⊆ H. Therefore, H is a two-sided ideal.


Page 59
MATH 611 Editor : Byeongho Ban

7.3.26
The characteristics of a ring R is the smallest positive integer n such that 1 + 1 + · · · + 1 = 0 (n times) in R; if no such integer
exists the characteristic of R is said to be 0. For example, Z/nZ is a ring of characteristic n for each positive integer n and Z is a
ring of characteristic 0.
(a) Prove that the map Z → R defined by

1 + 1 + · · · + 1(k times)
 if k > 0
k 7→ 0 if k = 0

−1 − 1 − · · · − 1(−k times) if k < 0

is a ring homomorphism whose kernel is nZ, where n is the characteristic of R (this explains the use of the terminology ”charac-
teristic 0” instead of the archaic phrase ”characteristic ∞” for rings in which no sum of 1’s is zero.)
(b) Determine the characteristics of the rings Q, Z[x], Z/nZ[x].
(c) Prove that if p is a prime and if R is a commutative ring of characteristic p, then (a + b)p = ap + bp for all a, b ∈ R.
Proof. (B. Ban) .

(a)
Let the mapping be ϕ. Then note that, for any k, m ∈ Z,
ϕ(k + m) = sgn(k + m)(1 + · · ·k+m times · · · + 1) = sgn(k)(1 + · · ·k times · · · + 1) + sgn(m)(1 + · · · m times · · · + 1) = ϕ(k) + ϕ(m)
ϕ(km) = sgn(km)(1 + · · ·kmtimes · · · + 1) = sgn(k)sgn(m)((1 + · · ·k times + 1) + · · ·m times · · · + (1 + · · ·k times + 1)) = ϕ(k)ϕ(m).
where sgn(k) is a sign of k.
Thus, ϕ is a ring homomorphism. And note that
ker(ϕ) = {k ∈ Z : (1 + · · ·k times · · · + 1) = 0} = {k ∈ Z : k ≡ 0 (mod n)} = nZ

(b)
Note that there does not exist n ∈ Z such that n × 1 = 0 when 1 ∈ Q. Thus, the characteristic of Q is 0.

Similarly, the characteristic of Z[x] is 0.

Note that 1 ∈ Z/nZ[x] and ϕ(kn) = 0 for any k ∈ Z. Thus, the characteristic is n.

(c)
Observe that
p
X p!
(a + b)p = ak bp−k .
k!(p − k)!
k=0
Let
(p − 1)!
Lk = .
k!(p − k)!
Note that, when 0 < k < p
p! p(p − 1)!
pLk = = .
k!(p − k)! k!(p − k)!
and that (p − k) < p and k < p. Since p is prime, there is no factor of denominator that divides p which means that k!(p − k)!
divides (p − k)!. Thus, Lk is integer so p|(pLk ). Therefore, pLk = 0 when 0 < k < p and thus
(a + b)p = ap + bp ∀a, b ∈ R.


Page 60
MATH 611 Editor : Byeongho Ban

7.3.29
Let R be a commutative ring. Recall (cf. Exercise 13, Section 1) that an element x ∈ R is nilpotent if xn = 0 for some n ∈ Z+ .
Prove that the set of nilpotent elements form an ideal-called the nilradical of R and denoted by N(R). [Use the Binomial Theorem
to show N(R) is closed under addition.]
Proof. (B. Ban) .

Note that 0 ∈ N(R), so N(R) 6= ∅. And let x, y ∈ N(R) be given. Then ∃n, m ∈ Z+ such that xn = 0 = y m . And note that, by
the Binomial Theorem,
m+n  
k m+n
X
m+n
(x − y) = (−1) xk y m+n−k
k
k=0
   
X m + n k m+n−k X m + n k m+n−k
= (−1)k x y + (−1)k x y
k k
0≤k≤n n+1≤k≤m+n
   
k m+n k m+n
X X
k m +n−k
= (−1) x (y )y + (−1) (xn )xk−n y m+n−k .
k k
0≤k≤n n+1≤k≤m+n

Since n − k ≥ 0 when 0 ≤ k ≤ n and k − n ≥ 0 when n + 1 ≤ k ≤ m + n, observe that


   
k m+n k m+n
X X
m+n k +n−k
(x − y) = (−1) x (0)y + (−1) (0)xk−n y m+n−k = 0.
k k
0≤k≤n n+1≤k≤m+n

Thus, x − y ∈ N(R), so N(R) is a subgroup. And let x ∈ N(R) and r ∈ R be given. Then ∃n ∈ Z+ such that xn = 0. Since R is
commutative, it suffices to show that N(R) is one of left or right ideal in order to show that it is an two sided ideal. Then observe
that, since R is commutative,
(xr)n = xn rn = 0rn = 0.
Thus, N(R) is closed under multiplication, so is a subring. Also, note that N(R)r ⊆ N(R), so it is a right ideal. Therefore, N(R)
is an ideal of R.

7.3.30
Prove that if R is a commutative ring and N(R) is its nilradical (cf.the preceding exercise) then zero is the only nilpotent element
of R/N(R) i.e, prove that N(R/N(R)) = 0.
Proof. (B. Ban) .

Let rN(R) ∈ N (R/N(R)). Then there exist n ∈ Z+ such that (rN(R))n = rn N(R) = N(R). It means that rn ∈ N(R)
and furthermore, it means that ∃m ∈ Z+ such that rmn = (rn )m = 0. Since mn ∈ Z+ , r ∈ N(R). And it implies that
rN(R) = N(R) = 0. Therefore, N (R/N(R)) = 0.

7.3.31    
0 1 0 0
Prove that the elements and are nilpotent elements of M2 (Z) whose sum is not nilpotent (note that these two
0 0 1 0
matrices do not commute). Deduce that the set of nilpotent elements in the noncommutative ring M2 (Z) is not an ideal.
Proof. (B. Ban) .

Observe that
 2    2  
0 1 0 0 0 0 0 0
= and = .
0 0 0 0 1 0 0 0
Therefore, the two matrices are nilpotent elements of M2 (Z). However note that
     
0 1 0 0 0 1
A= + =
0 0 1 0 1 0
and that
   
2k 1 0 2 2k+1 0 1
A = = A and that A = =A ∀k ∈ Z.
0 1 1 0
Therefore, ∀n ∈ Z+ , An 6= 0 which implies that A is not a nilpotent element. From here, we can deduce that N(M2 (Z)) is not
closed under addition, so is not a subgroup so not a subring so not an ideal.


Page 61
MATH 611 Editor : Byeongho Ban

7.3.34
Let I and J be ideals of R.
(a) Prove that I + J is the smallest ideal of R containing both I and J.
(b) Prove that IJ is an ideal contained in I ∩ J.
(c) Give an example where IJ 6= I ∩ J.
(d) Prove that if R is commutative and if I + J = R, then IJ = I ∩ J.
Proof. (B. Ban) .

(a)
Recall that
I + J = {i + j : i ∈ I and j ∈ J}.
Note that
I = {i + 0 : i ∈ I} ⊆ I + J J = {j + 0 : j ∈ J} ⊆ I + J.
Thus, I + J 6= ∅.
Observe that, for any x, y ∈ I + J,
x − y = (ix + jx ) − (iy − jy ) = (ix − iy ) + (jx − jy ) ∈ I + J
xy = (ix + jx )(iy + jy ) = ix iy + ix jy + jx iy + jx jy ∈ I + J (∵ I and J are ideals so ix jy , jx iy ∈ I).
where x = ix + jx and y = iy + jy with ix , iy ∈ I and jx , jy ∈ J. Thus, I + J is a subring.
Let r ∈ R be given. Then observe that, for any i ∈ I and j ∈ J,
r(i + j) = ri + rj ∈ I + J (∵ ri ∈ I rj ∈ J.)
(i + j)r = ir + jr ∈ I + J (∵ ir ∈ I jr ∈ J.)
Therefore, I + J is an ideal.
Suppose that M ⊆ R be an ideal containing I and J. Then it is clear that i + j ∈ M for any i ∈ I and j ∈ J. Thus, I + J ⊆ M .
Therefore, I + J is the smallest ideal containing I and J.

(b)
Recall that IJ is the set of all finite sums of element of the form ab with a ∈ I and b ∈ J.
Pn
Observe that, for any r ∈ IJ, we can describe as r = k=1 ik jk where ik ∈ I jk ∈ J and that each ik jk ∈ I and ik jk ∈ J for all
1 ≤ k ≤ n since I and J are ideals. Therefore, r ∈ I ∩ J, so IJ ⊆ I ∩ J. And further note that 0 ∈ IJ so IJ 6= ∅ and for any
r, s ∈ IJ,
r − s = (ir1 jr1 + · · · + irn jrn ) − (is1 js1 + · · · + isn jsn ) = ir1 jr1 + · · · + irn jrn − is1 js1 − · · · − isn jsn ∈ IJ
rs = (ir1 jr1 + · · · + irn jrn )(is1 js1 + · · · + isn jsn ) ∈ IJ (∵ each irk jrk ∈ I so r ∈ I and each isk jsk ∈ J so s ∈ J )
where r = (ir1 jr1 + · · · + irn jrn ) and s = (is1 js1 + · · · + isn jsn ).
Therefore, IJ is an ideal contained in I ∩ J.

(c)
Let I = (2) and J = (6). Then observe that I ∩ J = (6) and IJ = (12) which implies that IJ ( I ∩ J. So IJ 6= I ∩ J.

(d)
It suffices to prove that I ∩ J ⊆ IJ due to (b).
Suppose that R is commutative and I + J = R. Note that since R is commutative,
IJ = {ij : i ∈ I and j ∈ J}.
Also, note that ∃i ∈ I and ∃j ∈ J such that i + j = 1 since 1 ∈ R. Suppose that r ∈ I ∩ J ⊆ R. Then observe that
a = a1 = a(i + j) = ai + aj ∈ JI + IJ = IJ.
Therefore, I ∩ J ⊆ IJ, so IJ = I ∩ J.

Let R be a ring with identity 1 6= 0 for Section 7.4.

Page 62
MATH 611 Editor : Byeongho Ban

Proof. (B. Ban) 7.4.2

Assume R is commutative.

Let the augmentation ideal of the group ring RG be I. P It is clear that g − 1 ∈ I ∀g ∈ G,Pso ({g − 1 : g ∈ G}) ⊆ I. Let
P n n n
r g
i=1 i i + r0 ∈ I with {ri } ⊆ R, {gi } ⊆ G be given, then i=0 ri = 0. It implies that r0 = − i=1 ri . And note that
n
X n
X Xn n
X
ri (gi − 1) = ri gi − ( ri ) = ri gi + r0 .
i=1 i=1 i=1 i=1
Therefore, any element in I can be represented by finite sum of element in {g − 1 : g ∈ G}, so I ⊆ ({g − 1 : g ∈ G}). Thus,
I = ({g − 1 : g ∈ G}) , so the augmentation ideal of RG is generated by {g − 1 : g ∈ G}.

Now let G = hσi and the augmented ideal RG be I. Clearly, σ − 1 ∈ I, so (σ − 1) ⊆ I. From the argument above, we know that
I = {σ k − 1 : k ∈ Z} .


Thus, it suffices to prove that σ k − 1 ∈ (σ − 1) ∀k ∈ Z+ . And observe that


σ k − 1 = (σ − 1)(σ k−1 + σ k−2 + · · · + 1) so σ k − 1 ∈ (σ − 1) ∀k ∈ Z+ \ {0, 1}.
When k = 0, 1, trivially, σ 0 − 1 = 0, σ − 1 ∈ (σ − 1). Therefore, {σ k − 1 : k ∈ Z+ } ⊆ (σ − 1), so I ⊆ (σ − 1). Thus, we have
I = (σ − 1).

Proof. (B. Ban) 7.4.6

Suppose that R is a division ring and I is nonzero left ideal. Let r ∈ R and u ∈ I be given, then ∃v ∈ R such that vu = 1 and
observe that
r = r(1) = r(vu) = (rv)u ∈ (rv)I = I.
Thus, R ⊆ I, so I = R.

Conversely, suppose that only left ideals of R are (0) and R. Let u ∈ R, then (0) 6= (u) = R, so 1 ∈ (u). It implies that ∃v ∈ R
such that vu = 1. Thus, u is a unit. Therefore, every elements in R are unit so R is a division ring.

Proof. (B. Ban) 7.4.8
Let R be an integral domain.

Suppose that (a) = (b) for some elements a, b ∈ R. Then, since R is commutative, ∃u, v ∈ R such that a = ub and b = va. Observe
that
a = ub = uva =⇒ (1 − uv)a = 0 =⇒ uv = 1.
Therefore, u and v are units and a = ub.

Conversely, suppose that a = ub for some unit u ∈ R. Then clearly a ∈ (b). Since (a) is the smallest ideal containing a, (a) ⊆ (b).
Also, observe that, since u ∈ R is a unit, ∃v ∈ R such that vu = 1, so b = (vu)b = v(ub) = va. Thus, b ∈ (a), so (b) ⊆ (a) since
(b) is the smallest ideal containing b. Therefore, (a) = (b).

Proof. (B. Ban) 7.4.9

Let R be the ring of all continuous functions on [0, 1] and let I be the collection of functions f (x) ∈ R with
f (1/3) = f (1/2) = 0. First of all note that 0 ∈ I, so I 6= ∅. Let g, h ∈ I be given . And observe that
(g − h)(1/3) = g(1/3) − h(1/3) = 0 = g(1/2) − h(1/2) = (g − h)(1/2). Thus, g − h ∈ I, so I is a subgroup. Nestly, Let
g ∈ R and f ∈ I be given. And note that (f g)(1/2) = f (1/2)g(1/2) = 0 = f (1/3)g(1/3) = f g(1/3). Therefore, f g ∈ I, so I is an
ideal(Note R is commutative.).

Note that (x − 1/2), (x − 1/3) ∈ R and (x − 1/2)(x − 1/3) ∈ I. However, neither (x − 1/2) nor (x − 1/3) is in I. Thus, I is not a
prime ideal.


Page 63
MATH 611 Editor : Byeongho Ban

Proof. (B. Ban) 7.4.10

Assume R is commutative. And suppose that P is a prime ideal of R and P contains no zero divisors. We should prove that R
has no zero divisor. Suppose that ab = 0 for some a, b ∈ R. Since 0 ∈ P , ab ∈ P . Since P is a prime ideal, either a or b is in
P . But P has no zero divisor. Therefore, a = 0 or b = 0. It implies that R is a commutative ring with identity that has no zero
divisor which is an integral domain.

Proof. (B. Ban) 7.4.11

Assume R is commutative. Let I and J be ideals of R and assume P is a prime ideal of R that contains IJ.

Assume that neither I nor J is in P , then ∃i ∈ I and ∃j ∈ J such that i 6∈ P and j 6∈ P . But we have ij ∈ P since IJ ⊆ P , so
either i ∈ P or j ∈ P since P is a prime ideal. It is a contradiction. Therefore, either I or J is contained in P .

Proof. (B. Ban) 7.4.12

Assume R is commutative and suppose I = (a1 , a2 , . . . , an ) and J = (b1 , b2 , . . . , bm ) are two finitely generated ideals in R.

Since ai ∈ I ∀i = {1, . . . , n} and bj ∈ J for j ∈ {1, . . . , m}, clearly, ai bj ∈ IJ for ∀i, j, so ({ai bj : 1 ≤ i ≤ n, 1 ≤ j ≤ m}) ⊆ IJ.
Conversely, let c ∈ IJ, then ∃a ∈ I and ∃b ∈ J such that, for some ri , sj ∈ R for 1 ≤ i ≤ n and 1 ≤ j ≤ m,
n
X m
X
a= ri ai b= sj bj .
i=1 j=1

Then, further observe that


n
! m 
X X
c = ab = ri ai  sj bj  = rn sm an bm + · · · + r1 s1 a1 b1 ∈ ({ai bj : 1 ≤ i ≤ n, 1 ≤ j ≤ m}).
i=1 j=1

It implies that IJ ⊆ ({ai bj : 1 ≤ i ≤ n, 1 ≤ j ≤ m}), so IJ = ({ai bj : 1 ≤ i ≤ n, 1 ≤ j ≤ m}). Therefore, IJ is finitely generated.



Proof. (B. Ban) 7.4.15

Let x2 + x + 1 be an element of the polynomial ring E = F2 [x] and use the bar notation to denote passage to the quotient ring
F2 [x]/(x2 + x + 1).

(a)
Note that it is clear that 0, 1, x, x + 1 ∈ F2 [x]/(x2 + x + 1) since 1, x and x + 1 are of degree less than 2 so they cannot be
multiple of x2 + x + 1 and for some nonzero r ∈ E. Note that any element in F2 [x] is of degree less than equal to 2 so there are at
most three terms and there are two choice for each coefficients. Therefore, the order to F2 [x] is 23 = 8. Therefor, by Lagrange’s
theorem, the order of E should be 4 or 8. However (x2 + x + 1) 6= (0) thus E = {0, 1, x, x + 1}.

(b)
Observe that additive table below.
+ 0 1 x x+1
0 0 1 x x+1
1 1 0 x+1 x
x x x+1 0 1
x+1 x+1 x 1 0
Note that the table is isomorphic to Klein 4-group.

(c)
Observe that multiplicative table below.
× 0 1 x x+1
0 0 0 0 0
1 0 1 x x+1
x 0 x x+1 1
x+1 0 x+1 1 x
×
Note that the table for E is isomorphic to 3-cyclic group which implies multiplicative abelian structure. Therefore, E is a field.


Page 64
MATH 611 Editor : Byeongho Ban

Proof. (B. Ban) 7.4.19

Let R be a finite commutative ring with identity.

Let P be a prime ideal of R, then R/P is a finite integral domain. Since any finite integral domain is a field, R/P is a field.
Therefore, P is a maximal ideal.

Proof. (B. Ban) 7.4.33

Let R be the ring of all continuous functions from the closed interval [0, 1] to R and for each c ∈ [0, 1] let
Mc = {f ∈ R : f (c) = 0}(recall that Mc was shown to be a maximal ideal of R).

(a)
Let M be any maximal ideal of R. Assume that M 6= Mc ∀c ∈ [0, 1]. Then, ∃fc ∈ M such that fc (c) 6= 0. Without loss of
generality, we can assume that fc (c) > 0, otherwise just replace this with fc2 . Sine fc is continuous, there exists  > 0 such that
fc (x) > 0 ∀x ∈ [0, 1] ∩ (c − , c + ). Now the sets of the form [0, 1] ∩ (c − , c + ) form an open cover of [0, 1] as c runs through
[0, 1],Pbut [0, 1] is compact, so there exist finitely many c1 , . . . , cn ∈ [0, 1] such that [0, 1] ⊂ ∪ni=1 ([0, 1] ∩ (ci − , ci + )). Let
n
f = i=1 fci , then by the construction, f (x) > 0 ∀x ∈ [0, 1] and it implies that f −1 ∈ M so M contains a unit. Then M = R so
M cannot be a maximal ideal so it is a contradiction. Thus, M = Mc for some c ∈ [0, 1].

(b)
Note that x − b ∈ Mb but x − b 6∈ Mc because c − b 6= 0. Thus, Mb 6= Mc .

(c)
Observe that |x − c| ∈ Mc . Assume that |x − c| ∈ (h(x)) for some h ∈ Mc , then ∃g ∈ M such that h(x)g(x) = |x − c|. However,
|x − c|/h(x) is not vanishing at x = c or is discontinuous function and it contradicts to the fact that g is continuous. Therefore,
|x − c| 6∈ (x − c) so Mc is not a principal ideal.

(d)
Assume that Mc is finitely generated ideal, for example, Mc = hf1 , f2 , · · · , fn i, such that |fk (x)| ≤ 1 for x ∈ [0, 1].
n p
P
Consider, f (x) = |fk (x)| ∈ Mc .
k=1
n
P n
Then, there are functions {gk }1≤k≤n ∈ C[0, 1], such that, f (x) = gk (x)fk (x) say, max gk (x) = M .
k=1 k=1
1
Then, choose an open interval U ⊂ [0, 1] near c, such that |fk (x)| ≤ , for each k since it exists by continuity of fk and
M2
fk (c) = 0)
n
P n p
P
So for x ∈ U , we have, |f (x)| ≤ M |fk (x)| < |fk (x)| = f (x) and it is a contradiction.
k=1 k=1

Proof. (B. Ban) 7.5.2

There is no ring of fractionas of D when D contains 0. Thus, let’s consider only when 0 6∈ D. Let Q be the field of fraction of
R. Then there exists an injection map ι : R → Q. Note that ι(d) is a unit of Q for any d ∈ D \ {0}. Then we have an injective
homomorphism Φ : D−1 R → Q such that Φ|R = ι. Then it implies that Q contains isomorphic copy of D−1 R, namely Φ(D−1 R).
In other words, D−1 R is isomorphic to a subring of the quotient field of R. Finally, note that Φ(D−1 R) is commutative and does
not contain any zero divisor since it is a subset of a field R. Therefore, Φ(D−1 R) is an integral domain, so is D−1 R.


Page 65
MATH 611 Editor : Byeongho Ban

Proof. (B. Ban) 7.5.3

Let F0 is a subfield of F that is generated by {0, 1}. In other words, F0 is the smallest subfield containing {0, 1} and the
intersection of all fields that contains {0, 1}. Then, since any subfield of F should contain {0, 1}, it should contains F0 so F0 is
the unique smallest subfield of F .

Let’s consider φ : Z → F defined by φ(n) = n · 1 = 1 + 1 + · · · + 1(adding n copies of 1). Then note that φ(Z) ∼
= Z/ ker φ.

Suppose that φ(n) 6= 0 ∀n 6= 0. Thus, kerφ = {0} and φ(Z) ∼ = Z. Observe that φ(1) = 1 so φ(Z) ⊆ F0 since φ is a
homomorphism and since F0 is a field so φ(n) = n · φ(1) = n · 1 ∈ F0 ∀n ∈ Z. Note that Z is an integral domain so is φ(Z).
Then note that there is an unique homomorphism ϕ mapping from φ(Z) to the field of fractions of φ(Z), say R. Observe
that R is isomorphic to Q since the field of fractions of Z is Q. Note that the inclusion map ι : φ(Z) ,→ F0 is clearly a
homomorphism. Thus, there exists an injective homomorphism Φ : R → F0 , so Φ(R) is a subfield of F0 . Note that ϕ(R)
is a field so contains {0, 1}. Therefore, F0 = Φ(R) and Φ is an isomorphism. Therefore, F0 is isomorphic to R so is isormorphic to Q.

Now, suppose that ∃n ∈ Z such that φ(n) = n · 1 = 0. Since Z is a UFD, we have unique prime number decomposition of
n = pk11 pk22 · · · pknn . Then there is one prime number p such that φ(p) = p · 1 = 0. (For example, ab · 1 = 0 =⇒ a · 1 = 0 or
b · 1 = 0.) Then kerφ = pZ. Thus, φ(Z) ∼ = Z/pZ = Zp so φ(Z) is a field. Note that φ(Z) = {n · 1 : n ∈ Z} ⊆ F0 since F0 is a field.
But F0 is the smallest field containing {0, 1} which is also contained in φ(Z). Therefore, F0 ⊆ φ(Z) ∼ = Zp . Therefore, F0 = φ(Z)
and F0 is isomorphic to Zp .

Proof. (B. Ban) 7.5.5

Page 66
MATH 611 Editor : Byeongho Ban

Proof. (B. Ban) 7.6.1

Let r, s ∈ R be given. And observe that


re − se = (r − s)e ∈ Re ∵r−s∈R
r(1 − e) − s(1 − e) = (r − s)(1 − e) ∈ R(1 − e) ∵ r − s ∈ R.
It implies that Re and R(1 − e) are subgroups under +. Now, further observe that
s(re) = sr(e) ∈ Re (re)s = r(es) = r(se) = (rs)e ∈ Re
s(r(1 − e)) = (sr)(1 − e) ∈ R(1 − e) (r(1 − e))s = r((1 − e)s) = r(s − es) = r(s − se) = rs(1 − e) ∈ R(1 − e).
It implies that Re and R(1 − e) are closed under left and right multiplication, so they are two sided ideals of R.

Now, observe that 1 = e + (1 − e) ∈ Re + R(1 − e), so R = Re + R(1 − e). Thus, Re and R(1 − e) are comaximal and we can
apply Chinese Remainder Theorem as below,
∼ R/Re × R/R(1 − e).
R/((Re) ∗ (R(1 − e))) =
(For making clear, we use Re ∗ R(1 − e) to denote ReR(1 − e))
Note that, for any s, r ∈ R,
(re)(s(1 − e)) = rs(e − e2 ) = re(e − e) = 0.
It implies that Re ∗ R(1 − e) = {0} so R/(Re ∗ R(1 − e)) ∼
= R.
Now, consider the map ϕ1 : R → Re and ϕ2 : R → R(1 − e) defined by ϕ1 (r) = re and ϕ2 (r) = r(1 − e). Clearly, ϕ1 and ϕ2 are
surjective homomorphism because for any r, s ∈ R,
ϕ1 (r + s) = (r + s)e = re + se = ϕ1 (r) + ϕ1 (s)
ϕ2 (r + s) = (r + s)(1 − e) = r(1 − e) + s(1 − e) = ϕ2 (r) + ϕ2 (s)
ϕ1 (rs) = rse = rse2 = rese = ϕ1 (r)ϕ1 (s)
ϕ2 (rs) = rs(1 − e) = rs(1 − e − e + e2 ) = rs(1 − e)(1 − e) = r(1 − e)s(1 − e) = ϕ2 (r)ϕ2 (s).
And note that, for any r ∈ R, ϕ1 (r(1 − e)) = r(1 − e)e = r(e − e2 ) = 0 and ϕ2 (re) = re(1 − e) = r(e − e2 ) = 0. Thus, we have
ker(ϕ1 ) ⊇ R(1 − e) and ker(ϕ2 ) ⊇ Re. And note that Re ∩ ker(ϕ1 ) = {0} and R(1 − e) ∩ ker(ϕ2 ) = {0}. Since R = Re + R(1 − e),
ker(ϕ1 ) = R(1 − e) and ker(ϕ2 ) = Re. Therefore, Re ∼= R/ker(ϕ1 ) ∼ = R/R(1 − e) and R(1 − e) ∼= R/ker(ϕ2 ) ∼ = R/Re. Then, we
have
∼ R/Re × R/R(1 − e) =
R= ∼ R(1 − e) × Re ∼ = Re × R(1 − e).
Lastly, observe that, for any r ∈ R,
e(re) = re2 = re, (re)e = re2 = re
(1 − e)(r(1 − e)) = r(1 − e)(1 − e) = r(1 − e − e + e ) = r(1 − e), (r(1 − e))(1 − e) = r(1 − e)2 = (1 − e).
2

Thus, e and 1 − e are identities for Re and R(1 − e) respectively.



Proof. (B. Ban) 7.6.3

Let L be an ideal of R × S. And let


I = {i ∈ R : ∃s ∈ S such that (i, s) ∈ L}
J = {j ∈ S : ∃r ∈ R such that (r, j) ∈ L}.
0 0
Observe that, for any i, i ∈ I, j, j ∈ J, r ∈ R and s ∈ S, since L is two sided ideal,
∃z, z 0 ∈ S such that (i, z), (i0 , z 0 ) ∈ L =⇒ (ri, rz), (ir, zr), (i − i0 , z − z 0 ) ∈ L =⇒ ir, ri, i − i0 ∈ I
∃o, o0 ∈ R such that (o, j), (o0 , j 0 ) ∈ L =⇒ (os, js).(so, sj), (o − o0 , j − j 0 ) ∈ L =⇒ js, sj, j − j 0 ∈ J
Therefore, I and J are ideals of R and S respectively.
Now, let (i, j) ∈ L, then i ∈ I and j ∈ J, so (i, j) ∈ I × J. Thus, we have L ⊆ I × J. Conversely, suppose (i, j) ∈ I × J. Then
∃r ∈ R and s ∈ S such that (i, s), (r, j) ∈ L. And observe that
(i, j) = (i, 0) + (0, j) = (i, s)(1, 0) + (r, j)(0, 1) ∈ L.
Therefore, I × J ⊆ L and I × J = L. 
Proof. (B. Ban) 7.6.4

Note that (1, 0) ∈ R × S and (1, 0) 6= (0, 0). But it does not have a multiplicative inverse. If it has, say (r, s) ∈ R × S, then
(1, 1) = (r, s)(1, 0) = (r, 0). But we know 1 6= 0, thus it is a contradiction. Therefore, a nonzero element of R × S is not a unit so
it is not a field.


Page 67
MATH 611 Editor : Byeongho Ban

Proof. (B. Ban) 7.6.8

(a)
It is clearly reflexive since, if a ∈ Ai for some i, ρii (a) = ρii (a).
It is also symmetric since, if a ∼ b and a ∈ Ai and b ∈ Aj for some i, j, observe that
a ∼ b ⇐⇒ ∃k ≥ i, j such that ρik (a) = ρjk (b) ⇐⇒ ∃k ≥ i, j such that ρjk (b) = ρik (a) ⇐⇒ b ∼ a.
Lastly, it is transitive. Let a ∈ Ai , b ∈ Aj , and c ∈ An . And let a ∼ b and b ∼ c. Then ∃k ≥ i, j such that ρ(ik)(a) = ρjk (b) and
∃m ≥ j, n such that ρjm (b) = ρnm (c). Without loss of generality, let m ≥ k. Then observe that
ρkm ρik (a) = ρkm ρjk (b) ⇐⇒ ρim (a) = ρjm (b) =⇒ ρim (a) = ρjm (b) = ρnm (c)
Thus, ρim (a) = ρnm (c). Therefore, a ∼ c. Therefore, ∼ is an equivalence relation.

(b)
Suppose that ρij ∀i, j are injective. Then, for given i, observe that
ρi (a) = ρi (b) ⇐⇒ a = b ⇐⇒ a ∼ b ⇐⇒ ∃k ≥ i such that ρik (a) = ρik (b) ⇐⇒ a = b
since ρik is injective.
Therefore, ρi is injective for any i.

(c)
Suppose that a ∈ Ai and b ∈ Aj . Note that a ∼ ρik (a) and b ∼ ρjk (b) for some k ≥ i, j. Thus, the operation does not depend on
selection of representative so it is well defined due to ρi and ρj .
Clearly, it is commutative and associative. The identity of 0. And the inverse of a is


Page 68
MATH 611 Editor : Byeongho Ban

Due date : November 5th, 2018

Proof. (B. Ban)


8.1.3

Suppose that u ∈ R and N (u) = m. Since R is a Euclidean domain, we have there exist q, r ∈ R such that
1 = uq + r
where N (r) < N (u) or r = 0. If r 6= 0, N (r) < N (u) = m but there does not exist such nonzero r due to the minimality of m.
Thus, r = 0 which implies 1 = uq so u is unit.

Let u ∈ R \ {0} be given such that N (u) = 0. Since 0 is the minimal integer in the set of norms of nonzero element, by previous
argument u is a unit.

Proof. (B. Ban)
8.1.4

(a)
Since (a, b) = 1, ∃x, y ∈ R such that ax + by = 1. And since a divides bc, au = bc for some u ∈ R. Then, by multiplying c on
both sides, we have
c = acx + bcy = acx + auy = a(cx + uy).
Therefore, a divides c.
a a a
Moreover, even if (a, b) 6= 1, since (a,b) and b are relatively prime, if a divides bc, then (a,b) divides bc so (a,b) divides c by
preceding argument.

(b)
Suppose x, y is another solution than x0 , y0 , then
ax0 + by0 = N ax + by = N
a
so a(x0 − x) = b(y − y0 ). Since n = (a,b) divides b(y − y0 ), n divides (y − y0 ) so
a a
m = y − y0 =⇒ y = y0 + m
(a, b) (a, b)
b b
for some m ∈ Z. And similarly, since (a,b) divides a(x0 − x), (a,b) divides (x0 − x) so
b b
= x0 − x =⇒ x = x0 − m
(a, b) (a, b)
for some m ∈ Z. And clearly, those another solution since
   
b a
ax + by = a x0 − m + b y0 + m = ax0 + by0 = N.
(a, b) (a, b)
Therefore, for any m ∈ Z, those are the all and only possible solutions.


Page 69
MATH 611 Editor : Byeongho Ban

Proof. (B. Ban)


8.1.5
(a)
Since 2 and 4 are even, any linear combination of them is even. However, 5 is not even so it is a contradiction. Thus, there is no
solution for this equation.

(b)
Observe that
29 = 1 · 17 + 12
17 = 1 · 12 + 5
12 = 2 · 5 + 2
5=2·2+1
2=2·1
so we can easily compute −7 · 29 + 12 · 17 = 1. Then, by multiplying 31 on both sides, we have −217 · 29 + 372 · 17 = 31 so
(−217, 372) is a solution to the equation. Then by using 8.1.4, other solutions are (−217 + 17m, 372 − 29m) ∀m ∈ Z.

(c)
Note 85x + 145y = 505 can be written by 17x + 29y = 101 by dividing 5 from both sides. Then, by same way with (b), we get
the solutions (−707 + 17m, 1212 − 29m) ∀m ∈ Z.

Proof. (B. Ban)
8.1.6 (The postage stamp problem)

Note that, since (a, b) = 1, there exist n, m ∈ Z such that an + bm = 1. Then observe that
a(|n| + N n) + b(|m| + N m) = (a|n| + b|m|) + N (an + bm) = (a|n| + b|m|) + N
for any N ∈ Z+ . Note that (|n| + N n) and (|m| + N m) are both nonnegative. Thus, by setting N0 = (a|n| + b|m|), for any
N ≥ N0 , the solution to the equation ax + by = N exist with x and y nonnegative.

Suppose that ab − a − b = αa + βb for some positive integers α and β. Then we have


ab = (1 + α)a + (1 + β)b.
Note that a divides ab and (1 + α)a so divides (1 + β) since (a, b) = 1. Then 1 + β = au for some positive integer u. Similarly,
1 + α = bw for some positive integer w. Then observe that
ab = (1 + α)a + (1 + β)b = wab + uab = (w + u)ab > ab
which is a contradiction. Therefore, ab − a − b cannot be written as a positive linear combination of a and b.

Let an+bm = 1 and, without loss of generality, let n < m which means n < 0. We can make |n| ≤ b−1 sinc a(n+bt)+b(m+at) = 1
for any t ∈ Z. Then for any positive integer N > 0, observe that
a(N n − 1 + b) + b(N m − 1) = N (an + bm) + ab − a − b = N + (ab − a − b)
and N n − 1 + b > n − 1 + b = (n + b) − 1 ≥ 1 − 1 = 0. Also, N m − 1 ≥ 0 is obvious since m ≥ 1. Thus, for any positive integer
N > ab − a − b, ax + by = N has a solution such that both x and y is nonnegative.

Proof. (B. Ban) 8.1.7

Observe that
85 = (−6i)(1 + 13i) + (7 + 6i)
1 + 13i = (1 + i)(7 + 6i)
so (85, 1 + 13i) = 7 + 6i.


Page 70
MATH 611 Editor : Byeongho Ban

8.1.8 √
Let F = Q( D) be a quadratic field with associated quadratic integer ring O and field norm N as in Section 7.1.
(a)
Suppose D is −1, −2. − 3. − 7 or −11. Prove that O is a Euclidean Domain with respective to N . [Modify the proof for Z[i]
(D = −1) in the text. For D = −3, −7, −11 prove that every element of F differs from an element in O by an element whose
norm is at most (1 + |D|)2 /(16|D|), which is less than 1 for these values of D. Plotting the point of O in C may be helpful.]
Proof. (B. Ban)
When D = −1, −2. √ √ √ √
Let a, b, c, d ∈ Z with√c 6= 0 or d 6= 0 be given and then let α = a + b D ∈ Z[ D] and β = c + d D ∈ Z[ D]. And let’s define a
norm N by N (g + h D) = g 2 − Dh2 for any g, h ∈ Z. And without loss of generality, let N (α) > N (β). Now, observe that
√ √ √ √
α a+b D (a + b D)(c − d D) (ac + Dbd) + (bc − ad) D
= √ = = .
β c+d D c2 − Dd2 c2 − Dd2
For convenience, let r = cac+Dbd bc−ad
2 −Dd2 and s = c2 −Dd2 . Note that r, s ∈ Q. Let p be an integer closet to r and q be an integer closet to

s, then |r − p| ≤ 12 and |q − s| ≤ 12 . Let γ = (r − p) + (s − q) D. Then observe that
 2  2
√ 1 1 3
N (γ) = N ((r − p) + (s − q) D) = (r − p)2 − D(s − q)2 ≤ +2 = for D = −1 or − 2
2 2 4
since |r − p| ≤ 1/2 and |s − q| ≤ 1/2 by our choice of p and q. Now, note that
√ √
α = β(r + s D) = β(p + q D) + γβ
√ √ √
and that γβ = α − (p + qi)β ∈ Z[ D] and p + q D ∈ Z[ D]. Finally note that N (γβ) ≤ N (γ)N (β) ≤ 34 N (β) < N (β). Therefore,

Z[ D] is Euclidean domain.

When D = −3, −7, −11.


Note that −3, −7, −11 ≡ 1 (mod 4). Thus, when our D is like them,
" √ #
1+ D
O=Z = Z[ω].
2
And we can define a norm by N (a + bω) = (a + bω)(a + bω) for any a, b ∈ Z. Let a, b, c, d ∈ Z with c 6= 0 and d 6= 0 be given and
let N (α) < N (β) without loss of generality. Observe that

α a + bω (a + bω)(c + dω) ac + bd + 1−D
4 bd  1+ D bc − ad
= = = 2 +
c + cd + b2 1−D 2 + cd + b2 1−D d2

β c + dω N (c + dω) d2 2 c
4 4
ac+bd+ 1−D
4 bd bc−ad
For convenience, let r = and s = and note that r, s ∈ Q.
( 1−D
c2 +cd+b2 4 d )
2 c2 +cd+b2 ( 1−D
4 d )
2

2
Claim. Every element of F differs from an element in O by an element whose norm is at most (1+|D|) 16|D| , which is less than 1 for
these values of D.
(Proof of claim.)
Let γ = r0 + s0 ω ∈ F be given. Let p be the closet integer to r0 among the integer less than r0 . And q be the closet integer to s0
among the integer less than s0 . Then γ is in a parallelogram with vertex at p + qω, (p + 1) + ω, p + (q + 1)ω, and (p + 1) + (q + 1)ω.
Let’s connect p + (q + 1)ω and (p + 1) + qω with a line. Note that we get two triangle. And let the distance between the center
of the triangle and one of the vertex be l. Then we can observe that, for any element in the parallelogram, we can find at least
one vertex that is located in the distance
2
of l.
Now we need to show that l = (1+|D|)
16|D| . Observe that, by Pythagrean Theorem, we have
r √
2
1 D
l + +l = .
4 2
Thus, by subtracting l from both sides and squaring both sides, we get
(1 + |D|)2 (1+|3|)2 1 (1+|7|)2 4 (1+|11|)2 9
l= , and note that 16|3| = 3 <1 16|7| = 7 <1 16|11| = 11 < 1.
16|D|
So we are done.

Thus, for these D values, we can find an element η ∈ O such that N (η − (r + sω)) ≤ l < 1. And note that α = βη + ψβ, where
ψ = η − (r + sω). And finally observe that
N (ψβ) = N (ψ)N (β) < 1 · β = β.
Therefore, we successfully constructed Euclidean Algorithm.


Page 71
MATH 611 Editor : Byeongho Ban

8.1.9 √
Prove that the ring of integers O in the quadratic integer ring Q( 2) is a Euclidean Domain with respect to the norm given by
the absolute value of the field norm N in Section 7.1.
Proof. (B. Ban) .
√ √
Let r, s ∈ Q and p, q ∈ Q \ {0} be given. And let α = r + s 2 and β = p + q 2. Observe that
√ √ √ √
α r+s 2 (r + s 2)(p − q 2) (rp − 2sq) + (sp − rq) 2
= √ = = .
β p+q 2 p2 − 2q 2 p2 − 2q 2
Let g = (rp−2sq) (sp−rq)
p2 −2q 2 and h = p2 −2q 2 and note that g, h ∈ Q. Let a and b are integers closest to g and h respectively. Note that |g − a|
and |h − b| are at most 21 . Thus, we have
 √  1 2 3
N (a − g) + (b − h) 2 = |(a − g)2 − 2(b − h)2 | ≤ |a − g|2 + 2|b − h|2 ≤ + = < 1.
4 4 4
√ √
Let γ = (a − g) + (b − h) 2 and δ = g + h 2 then
α = βδ + βγ.
And finally,
3
N (βγ) = N (β)N (γ) ≤N (β) < N (β).
4
Thus, there exists an Euclidean Algorithm for O, so it a Euclidean domain.

8.1.10
Prove that the quotient ring Z[i]/I is finite for any nonzero ideal I of Z[i]. [Use the fact that I = (α) for some nonzero α and
then use the Division Algorithm in this Euclidean Domain to see that every coset of I is represented by an element of norm less
than N (α).]
Proof. (B. Ban) .

Let a nonzero ideal I ⊆ Z[i] be given. Since Z[i] is a Euclidean domain so is a PID, I = (α) for some nonzero α ∈ Z[i]. Since
Z[i] is an Euclidean domain, for any x ∈ Z[i], there are r, q ∈ Z[i] such that x = αq + r with N (r) < N (α). Then note that
a representative of xI is r in the case above. Since N (α) is finite, there are only finite number of r such that N (r) < N (α).
Therefore, Z[i]/I should be finite.

Proof. (B. Ban) 8.1.11

(a)
Let a, b ∈ R be given and let e be the least common multiple of them. Note that a and b divide e so bt = e = ak for some k, t ∈ R.
Then e ∈ (a) ∩ (b) so (e) ⊆ (a) ∩ (b).
Now, suppose that (e) ⊂ (e0 ) ⊂ (a) ∩ (b). Then bt0 = e0 = ak 0 so a and b divide e0 . Then by the definition of e, e divides e0 so
e0 = es for some s ∈ R so (e0 ) ⊆ (e) so (e) = (e0 ) and (e) is the largest principal ideal contained in (a) ∩ (b).

(b)
Let a, b ∈ R be given. Then since R is Euclidean so is PID, (a) ∩ (b) = (e) for some e ∈ R. Since (e) is the largest principal ideal
contained in (a) ∩ (b), e is the least common multiple of a and b. And (e) = (ue) for any unit u ∈ R.

(c)
ab
Let a, b ∈ R be given. By (b), we know that there exists a least common multiple of a and b. Clearly, a and b divide (a,b) , so
 
ab
⊆ (a) ∩ (b).
(a, b)
Now, suppose that a and b divide e. Let a = a0 (a, b) and b = b0 (a, b). Then
ab
= a0 b0 (a, b).
(a, b)
Since a and b divide e, a0 (a, b) divides e. Since b0 is relatively prime to a and b0 divides e, a0 b0 (a, b) divides e so we are done.

8.2.1
Prove that in Principal Ideal Domain two ideals (a) and (b) are comaximal (cf. Section 7.6) if and only if a greatest common
divisor of a and b is 1 (in which case a and b are said to be coprime or relatively prime)

Page 72
MATH 611 Editor : Byeongho Ban

Proof. (B. Ban) .

Suppose that (a) and (b) are comaximal ideals of a ring R, then (a) + (b) = R. Recall that (a) + (b) is the smallest ideal containing
(a) and (b) therefore, (a, b) = R. If d is a greatest common divisor, R = (a, b) ⊆ (d) ⊆ R. Therefore, R = (d) which forces d to be 1.

Conversely, suppose that gcd(a, b) = 1. Then there are x, y ∈ R such that ax + by = 1 ∈ (a, b) = (a) + (b). Therefore, 1 ∈ (a) + (b)
and it means that (a) + (b) = R and (a) and (b) are comaximal.

Proof. (B. Ban)
8.2.2

Let R be a PID and let a, b ∈ R be given. Then (a) ∩ (b) = (e) for some e ∈ R since R is PID. Then e ∈ (a) and e ∈ (b) so
bv = e = ak for some k, v ∈ R which means b and a divide e. Nextly, suppose that a and b also divide e0 . Then (e0 ) ⊆ (a) ∩ (b)
which implies (e0 ) ⊆ (e) and so e divides e0 . Therefore, e is the least common multiplier of a and b.

8.2.3
Prove that a quotient of a P.I.D by a prime ideal is again a P.I.D.
Proof. (B. Ban) .

Let R be a P.I.D. And let I be a prime ideal of R. If I = (0), R/P = R is trivially a principal ideal.

If P is nonzero prime ideal, then P is a maximal ideal, so R/P is a field. But field is always a principal ideal since only ideal of a
field F is (0) and (1) = F .

Proof. (B. Ban)
8.2.4

Assume that there is a non-principal ideal I ⊂ R. Let a1 ∈ I \ {0} be given. Since I is not principal, (a1 ) ( I. Then let
b1 ∈ I \ (a1 ). Then let gcd(a1 , b1 ) = a2 so a2 a1 . Also, note that (a1 ) ( (a2 ) since, if (a1 ) = (a2 ), then (a1 , b1 ) ⊆ (a2 ) = (a1 )
which contradicts to the fact that b1 ∈ I \ (a1 ). Choosing an such that an an−1 , choose bn ∈ I \ (an ) and say an+1 = gcd(an , bn )
and so an+1 an . Also, (an ) ( (an+1 ) since, if (an ) = (an+1 ) then (an , bn ) ⊆ (an+1 ) = (an ) which contradicts to the fact that
bn ∈ I \ (an ). However, by the condition (ii), ∃N ∈ Z+ such that an = uan+1 for all n ≥ N where u is a unit which means
(an ) = (an+1 ) ∀n ≥ N . Therefore, it is a contradiction. Thus, there does not exist non-principal ideal in R.

8.2.5 √ √ √ √
Let R be the quadratic integer ring Z[ −5]. Define the ideals I2 = (2, 1 + −5), I3 = (3, 2 + −5), and I30 = (3, 2 − −5).
(a) Prove that I2 , I3 and I30 are nonprincipal ideals in R. [Note that Example 2 following Proposition 1 proves this for I3 .]
(b) Prove that the product of two nonprincipal ideals can be principal by showing that I22 is the principal ideal generated by 2,
i.e., I22 = (2). √ √
(c) Prove similarly that I2 I3 = (1 − −5) and I2 I30 = (1 + −5) are principal. Conclude that the principal ideal (6) is the product
of 4 ideals : (6) = I22 I3 I30 .

Page 73
MATH 611 Editor : Byeongho Ban

Proof. (B. Ban) .

(a) √ √ √
Suppose that
√ I2 is a principal ideal of R. Then
√ ∃a + b −5 √ ∈ Z[ −5] such
√ = (a + b −5). Let the field norm
that I2√ √ be N defined

by N (a +√b −5) = a2 +√5b2 . Since √ 2, 1 + −5 ∈ (a + b −5), ∃α + β −5, γ + δ −5 ∈ R such that 2 = (1 + −5)(α + β −5)
and (1 + −5) = (a + b −5)(γ + δ −5), then
√ √
(a2 + 5b2 )(α2 + 5β 2 ) = N (a + b −5)N (α + β −5) = N (2) = 4
√ √ √
(a2 + 5b2 )(γ 2 + 5δ 2 ) = N (a + b −5)N (γ + δ −5) = N (1 + −5) = 6.
From the first equation, note that if one of b or β is not zero, √
left hand side should be greater than 5 which √is contradiction.
Thus, b = β = 0. Then we have a2 α2 =√4 which implies a + b −5 = ±1√or ±2. If a = ±1, then 2k + (1 + −5)h = ±1 for
h, k ∈ I2 . But, by multiplying 1 − −5 on both sides we deduce 1 − −5 is divisible by 2 which is a contradiction. Thus,
some √
a + b√ −5 = ±2. However, √ then from the second equation, 4 divides 6 which is a contradiction. Thus, there does not exist
a + b −5 and so (2, 1 + −5) is not principal.
√ √ √ √ √
Similarly,
√ √ that I3 = (a√+ b −5) for
suppose √ some a + b −5 ∈ R. Then ∃α + β −5, γ + δ −5 ∈ R such that 3 = (α + β −5)(a +
b −5) and 2 + −5 = (γ + δ −5)(a + b −5). Thus, observe that
√ √
(a2 + 5b2 )(α2 + 5β 2 ) = N (a + b −5)N (α + β −5) = N (3) = 9
√ √ √
(a2 + 5b2 )(γ 2 + 5δ 2 ) = N (a + b −5)N (γ + δ −5) = N (2 + −5) = 9.
2 2

Thus, only
√ possible value of a + 5b is 1, 3 and 9. With similar √ reason with last √ argument, a2 + 5b2 6= 1 (if ±1 = a + b −5,
3h+(2+ −5)k = ±1 for some h, k ∈ R but by multiplying 2− −5 we conclude 2− −5 is divisible by 3 which is a contradiction)
2
. And note
√ that we don’t
√ have two integers
√ a and b satisfying a√ + 5b2 = 3,√
so a2 + 5b2 6= 3. Lastly, if a2 + 5b2 = 9, then it should
be 2√+ −5 = 1(a + b −5) = ±2 ± −5. But √ since then (2 + −5)(h + k −5) = 3 for some integers h, k. Then by multiplying
2− −5 on both sides, we conclude that 2− −5 is divisible by 3 which is a contradiction. Therefore, I3 cannot be a principal ideal.
√ √ √ √ √
Lastly, suppose that I30 = (a+b −5) with a+b −5 ∈ R. Then ∃α, β ∈ R such that 3 = α(a+b −5) and 2− −5 = β(a+b −5).
Thus, observe that

(a2 + 5b2 )α = N (a + b −5)N (α) = N (3) = 9
√ √
(a2 + 5b2 )β = N (a + b −5)N (β) = N (2 − −5) = 9.
√ √
But it is a contradiction by same argument above replacing 2 + −5 with 2 − −5 at the last. Therefore, I30 cannot be a principal
ideal.

(b) √ √ √ √
Observe that I22 = (4, 2 + 2 −5, −4 + 2 −5). And note that −4 + (2 + 2 −5) − (−4 + 2 −5) = 2 ∈ I22 so (2) ⊆ I22 . And note
that 2 divides each of generator of I22 , so I22 ⊆ (2). Therefore, I22 = (2).

(c)
Observe that
√ √ √ √ √
I2 I3 = (2, 1 + −5)(3, 2 +
−5) = (6, 4 + 2 −5, 3 + 3 −5, −3 + 3 −5)
√ √ √ √ √
I2 I30 = (2, 1 + −5)(3, 2 − −5) = (6, 4 − 2 −5, 3 + 3 −5, 7 + −5).
Note that
√ √ √ √
1− −5 = (4 + 2 −5) − (3 + 3 −5) ∈ I2 I3 =⇒ (1 − −5) ⊆ I2 I3
√ √ √ √
1 + −5 = (4 − 2 −5) + (3 + 3 −5) − 6 ∈ I2 I30 =⇒ (1 + −5) ⊆ I2 I30 .
Furthermore, observe that
√ √ √ √ √
6 = (1 − −5)(1
√ √ + −5),√ 4 + 2 √−5 = (1 − √−5)(−1 + √−5),
3 + 3 −5 = (1 − −5)(−2 + −5), −3 + 3 −5 = (1 − −5)(−2 + −5).
√ √ √
Thus, all generators of I2 I3 are divisible by (1 − −5), so are I2 I3 ⊆ (1 − −5), so I2 I3 = (1 − −5). Similarly, observe that
√ √ √ √ √
6 = (1√+ −5)(1 − √ −5), 4 − 2 √−5 = (1 + √−5)(−1 −√ −5),
3 + 3 −5 = 3(1 + −5), 7 + −5 = (1 + −5)(2 − −5).
√ √ √
Thus, all generators of I2 I3 are divisible by (1 + −5), so are I2 I30 ⊆ (1 + −5), so I2 I30 = (1 − −5).
0

Lastly, observe that, since R is commutative,


√ √
I22 I3 I30 = I2 I3 I2 I30 = (1 − −5)(1 + −5) = (6).


Page 74
MATH 611 Editor : Byeongho Ban

8.2.6
Let R be an integral domain and suppose that every prime ideal in R is principal. This exercise proves that every ideal of R is
principal, i.e., R is a P.I.D.
(a) Assume that the set of ideals of R that are not principal is nonempty and prove that this set has a maximal element under
inclusion (which, by hypothesis, is not prime). [Use Zorn’s lemma]
(b) Let I be an ideal which is maximal with respect to being nonprincipal, and let a, b ∈ R with ab ∈ I but a 6∈ I and b 6∈ I. Let
Ia = (I, a) be the ideal generated by I and a, let Ib = (I, b) be the ideal generated by I and b, and define J = {r ∈ R : rIa ⊆ I}.
Prove that Ia = (α) and J = (β) are principal ideals in R with I ( Ib ⊆ J and Ia J = (αβ) ⊆ I.
(c) If x ∈ I show that x = sα for some s ∈ J. Deduce I = Ia J is principal, a contradiction, and conclude that R is a P.I.D.
Proof. (B. Ban) .

(a)
Assume N = {I ⊆ R : I is a nonprincipal ideal of R} 6= ∅. Then N is partially ordered by the inclusion. And let C = {Cα } be a
given chain in N .
S
Let U = C∈C C. It is clear that U is nonempty. And if a, b ∈ U , a ∈ Ca and b ∈ Cb for some Ca , Cb ∈ C. Since C is partially
ordered by inclusion, without loss of generality, let Ca ⊆ Cb . Then a, b ∈ Cb so a − b, ab ∈ Cb ⊆ U . Thus, U is a subring.

Let r ∈ R be given. Suppose that a ∈ U , then a ∈ Ca for some Ca ∈ C. Since Ca is an ideal, ra ∈ rCa ⊆ Ca ⊆ U . So U is an
ideal (Since R is commutative, being one sided ideal implies it is two sided ideal.).

And assume that U = (r) for some r ∈ R. Then r ∈ Cr for some Cr ∈ C. Then U = (r) ⊆ Cr ⊆ U so Cr = (r) which is a
contradiction since Cr is not principal ideal. Therefore, U is not principal.

Since C was arbitrary, any chain in N has an upper bound in it. Thus, by Zorn’s lemma, N has an maximal element under inclusion.

(b)
Note I ( Ia and I ( Ib since a, b 6∈ I. Then due to maximality of I, Ia and Ib should be principal ideals, so Ia = (α) for some
α ∈ R. And observe that
bIa = b(I, a) ⊆ (I, ab).
Since ab ∈ I, we have bIa ⊆ I, so b ∈ J. Further note that, if i ∈ I,
iIa = i(I, a) ⊆ (I, ai).
Since ai ∈ I (I is an ideal), iIa ⊆ I so I ⊂ J. Therefore, Ib = (I, b) ⊆ J so
I ( Ib ⊆ J.
Then by the maximality of I, J = (β) for some β ∈ R.
Lastly note that, Ia j ⊂ I for j ∈ J, so Ia J ⊆ I. Then observe that
( n ) ( n )
X X
0 + 0 +
Ia J = (ri α)(ri β) : n ∈ Z and ri , ri ∈ R = si αβ : n ∈ Z and s ∈ R = (αβ).
i=1 i=1

(c)
Since I ( Ia = (α), x = sα for some s ∈ R. Since sα ∈ I, sIa = s(α) ⊆ I, so s ∈ J. Therefore, I ⊆ sIa ⊆ JIa . Then, by (b),
I = JIa = (αβ), so I is a principal ideal. And it is a contradiction since we have assumed that I is nonprincipal ideal. Therefore,
N = ∅ so R is a PID.

Proof. (B. Ban) 8.2.8

We only consider when 0 6∈ D here. Let Q be the field of fraction of R. Then the inclusion map ι : R → D−1 R is injective
ring homomorphism. Since ι(d) is unit in Q for any d ∈ D, the inclusion map φ : D−1 R → Q is again a ring homomorphism
with φ R = ι. Then by the first ring isomorphism theorem, φ(D−1 R) ∼
= D−1 R/Ker(φ) = D−1 R since φ is injective so D−1 R is
integral domain.

Let I 0 ⊆ D−1 R be a given ideal. Then let I = ι−1 (I 0 ) which is clearly an ideal in R. Then for any di ∈ D−1 I, di = d1 1i ∈ I 0 so
D−1 I ⊆ I 0 . And let dr ∈ I 0 be given. Then since r = d dr ∈ I 0 , r ∈ I so dr = d1 1r ∈ D−1 I which means I 0 ⊆ D−1 I so I 0 = D−1 I.
Since R is PID, I = (α) for some α ∈ R. Note that αd ∈ I 0 so note that α = d αd ∈ I 0 so (α) ⊆ I 0 . And also for any rα 0
d ∈ I ,
rα r 0 0 −1
d = α d ∈ (α) so I = (α) which implies that I is a principal so D R is PID.


Page 75
MATH 611 Editor : Byeongho Ban

8.3.5 √
Let R = Z[ −n] where
√ n is a squarefree
√ integer greater than 3.
(a) Prove that 2, −n and 1 + −n are irreducibles in R.
(b) Prove that R is not a U.F.D. Conclude that the quadratic
√ √ ring O is not a U.F.D. for D ≡ 2, 3 mod 4, D < −3(so
integer
also not Euclidean and not a P.I.D.). [Show that either −n or 1 + −n is not prime.] [Show that 2 is not prime] √
√ an explicit ideal in R that is not principal. [Using (b) consider a maximal ideal containing the nonprime ideal ( −n) or
(c) Give
(1 + −n).]
Proof. (B. Ban) .

(a) √
Suppose that 2 = αβ for some α, β ∈ R. Then, by the norm N , we have 4 = N (α)N (β) (where N (a + b −n) = a2 + nb2 ). Then
without loss of generality, let’s consider only α. Note that N (α) can be 1, 2, 4. If N (α) = 1, α = ±1 which means α is unit. If
N (α) = 2, note that a2 + nb2 = 2 for some a, b ∈ Z. Since n is greater than 3, there is no such a, b ∈ Z so it is a contradiction.
Lastly, if N (α) = 4, automatically N (β) = 1 so β = ±1. There results shows that 2 is irreducible in R.

Suppose that −n = αβ for some α, β ∈ R. Then, by the norm N , we have n = N (α)N (β). Note that N (α) can be 1, any
nontrivial divisor of n, and n. If N (α) = 1, α = ±1 which means α is unit. If N (α) is a nontrivial divisor m of n, note that
a2 + nb2 = m for some a, b ∈ Z . Since n is greater than m, b = 0 but since n is squarefree, a2 6= m. So, there
√ is no such a, b ∈ Z
so it is a contradiction. Lastly, if N (α) = n, automatically N (β) = 1 so β = ±1. There results shows that −n is irreducible in
R.

Suppose that 1 + −n = αβ for some α, β ∈ R. Then, by the norm N , we have 1 + n = N (α)N (β). Note that N (α) can be 1, any
nontrivial divisor of n+1, and n+1. If N (α) = 1, α = ±1 which means α is unit. If N (α) is a nontrivial divisor m of n+1, note that
a2 + nb2 = m for some a, b ∈ Z . Since n is greater than m, b = 0 but since n is squarefree, a2 6= m. So, there
√ is no such a, b ∈ Z so
it is a contradiction. Lastly, if N (α) = n+1, automatically N (β) = 1 so β = ±1. There results shows that −n is irreducible in R.

(b) √
In U.F.D., irreducible elements
√ are prime. Therefore, it suffices to show √
that an irreducible
√ Z[ −n].
element 2 is not a prime in √
is even then 2|(−n) = ( −n)2 and if n is odd then 2|(1 + n) = (1 + −n)(1 − −n). The element 2 never divides −n nor
If n √
1 ± −n so we see that 2 is irreducible
√ yet not prime.
If the quadratic integer ring Z[ D] for D ≡ 2, 3 mod 4 and D < −3 is a Euclidean domain, it is P.I.D, so it should be U.F.D.
and it is a contradiction based on the argument above.

(c) √ √ √ √
Let I = (2, −n).
√ √ If I is principal, I = (2a + b −n) for some a, b ∈ Z. Thus, ∃α, β ∈ Z[ −n] such that 2 = α(2a + b −n) and
−n = β(2a + b −n). And observe that
4 = N (2) = N (α)(4a2 + b2 n)

n = N ( −n) = N (β)(4a2 + b2 n)
Note that only possible choice of (4a2 + b2 n) is 1, 2 or 4. If it is 4, then 4|n but it is contradiction since n is squarefree. Also,
2 2
2 2
√ the condition 4a + b n = 2 since n > 3 and 4 > 2. Also, there are no such a, b ∈ Z such
there is no integers a and b satisfying
that 4a + b n = 1. Therefore, (2, −n) is not a principal ideal.

8.3.9
Suppose that the quadratic integer ring O is a P.I.D. Prove that the absolute value of the field norm N on O (cf. Section 7.1)
is a Dedekind-Hasse norm on O. Conclude that if the quadratic integer ring O possesses any Dedekind-Hasses norm, then in fact
the absolute value of the field norm on O already provides a Dedekind-Hasse norm on O. [If α, β ∈ O then (α, β) = (γ) for some
γ ∈ O. Show that if β does not divide α then 0 < |N (γ)| < |N (β)|– use the fact that the units in O are precisely the elements
whose norm is ±1.]

Page 76
MATH 611 Editor : Byeongho Ban

Proof. (B. Ban) .

Prove that the absolute value of the field norm N on O (cf. Section 7.1) is a Dedekind-Hasse norm on O.
Suppose O is a P ID. Let N be the field norm on O. Now, let α, β ∈ O be given such that β 6= 0 . Since O is a PID, ∃γ ∈ O such
that (α, β) = (γ). Then γ is a greatest common divisor of α and β. Then ∃δ ∈ O such that γδ = β so |N (β)| = |N (γ)||N (δ)|.
Since δ 6= 0, |N (δ)| > 0. If N (δ) = 1, then δ is a unit, so ∃η ∈ O such that δη = 1 so γ = ηβ. However, since ∃ξ ∈ O such that
α = ξγ, we have α = ξηβ so β divides α. If N (δ) > 1, note 0 < |N (γ)| < |N (γ)| < |N (γ)||N (δ)| = |N (β)|. Since γ is a greatest
common divisor, ∃a, b ∈ O such that γ = aα − bβ. Therefore, |N | is a Dedekind-Hasse norm.

Conclude that if the quadratic integer ring O possesses any Dedekind-Hasses norm, then in fact the absolute
value of the field norm on O already provides a Dedekind-Hasse norm on O.
By the argument above, in fact absolute field norm on the quadratic integer ring provides a Dedekind Hasse norm.

Proof. (B. Ban)
9.1.1

(a)
p(y, z)(x) = (2y)x2 − (3y 3 z)x + 4y 2 z 5 and q(y, z)(x) = (7 + 5y 3 z 4 − 3z 3 )x2

(b)
deg(p) = 7 and deg(q) = 9

(c)
deg(px ) = 2, deg(py ) = 3 and deg(pz ) = 5.
deg(qx ) = 2, deg(qy ) = 3 and deg(qz ) = 4.

(d)
pq(x, y, z) = 9x4 y + 10x4 y 4 z 4 − 6x4 yz 3 − 21x3 y 3 z − 15x3 y 6 z 5 + 9x3 y 3 z 4 + 28x2 y 2 z 5 + 20x2 y 5 z 9 − 12x2 y 2 z 8 . So
deg(pq) = 16, deg(pqx ) = 4, deg(pqy ) = 6 and deg(pqz ) = 9.

(e)
pq(x, y)(z) = (20x2 y 5 )z 9 − (12x2 y 2 )z 8 + (28x2 y 2 − 15x3 y 6 )z 5 + (10x4 y 4 + 9x3 y 3 )z 4 − (6x4 y)z 3 − (21x3 y 3 )z + 9x4 y.

Proof. (B. Ban)
9.1.2

(a)
p(y, z)(x) = (2y)x2 + y 2 z 5 and q(y, z)(x) = (1 + 2y 3 z 4 )x2

(b)
deg(p) = 7 and deg(q) = 9

(c)
deg(px ) = 2, deg(py ) = 2 and deg(pz ) = 5.
deg(qx ) = 2, deg(qy ) = 3 and deg(qz ) = 4.

(d)
pq(x, y, z) = x4 y 4 z 4 + x2 y 2 z 5 + 2x2 y 5 z 9 . So
deg(pq) = 16, deg(pqx ) = 4, deg(pqy ) = 5 and deg(pqz ) = 9.

(e)
pq(x, y)(z) = (2x2 y 5 )z 9 + (x2 y 2 )z 5 + (x4 y 4 )z 4 .


Page 77
MATH 611 Editor : Byeongho Ban

Proof. (B. Ban)


9.1.3

Let π ∈ Sn be given. Let ϕπ : R[x1 , x2 , . . . , xn ] → R[xπ(1) , xπ(2) , . . . , xπ(n) ] be a map defined by ϕπ (p(x1 , x2 , . . . , xn )) =
p(xπ(1) , xπ(2) , . . . , xπ(n) ). For convenience let (x1 , x2 , . . . , xn ) = x and (xπ(1) , xπ(2) , . . . , xπ(n) ) = π(x). And observe that
ϕπ (p(x) + q(x)) = p(π(x)) + q(π(x)) = ϕπ (p(x)) + ϕπ (q(x))
ϕπ (p(x)q(x)) = p(π(x))q(π(x)) = ϕπ (p(x))ϕπ (q(x)).
Thus, ϕπ is a ring homomorphism. Also observe that
ϕπ ◦ ϕπ−1 (p(x)) = ϕπ (p(π −1 (x))) = p(π ◦ π −1 (x)) = p(x)
ϕπ−1 ◦ ϕπ (p(x)) = ϕπ−1 (p(π(x))) = p(π −1 ◦ π(x)) = p(x)
which implies ϕπ is a bijection so it is a isomorphism. Therefore, R[x] ∼
= R[π(x)].

Proof. (B. Ban)
9.1.4

Suppose that pq ∈ (x) for some p, q ∈ Q[x, y] = R and assume that p, q 6∈ R. Then each of p and q has at least one term with
degree in x is zero let p0 (y) and q 0 (y). Then p0 (y)q 0 (y) is a term of pq so pq 6∈ (x). Therefore, either p or q is in R which means
(x) is a prime ideal.

Suppose that pq ∈ (x, y) for some p, q ∈ R and assume that neither of p and q is in (x, y). Then each of p and q has at least one
constant term, let p0 and q 0 respectively. Then pq has a constant term p0 q 0 which implies that pq 6∈ (x, y) which is a contradiction.
Therefore, either p or q is in R and so (x, y) is a prime ideal.

Note that (x) ( (x, y) since y 6∈ (x). Also, note that (x, y) ( R since 1 6∈ (x, y). Thus, (x) cannot be a maximal ideal. Moreover,
assume that (x, y) ⊆ I ( R. Now, let q ∈ I. Since I 6= R, I does not contain any element from Q. And if q has nonzero
constant term, we can collect all the terms of q which has nonzero degree in x, let’s call it xp(x, y), and the rest of them which
are not constant, let’s call it yh(x, y). Then q(x, y) = xp(x, y) + yh(x, y) + c where c is the constant term. However, since
xp + yh ∈ (x, y) ⊆ I, c = q(x, y) − (xp + yh) ∈ I which means I has a unit so R = I which is a contradiction. Thus, we know
that q has no constant term. Then with same procedure with above, we collect appropriate terms together, we can say q ∈ (x, y)
which proves that I = (x, y). Thus, (x, y) is a maximal ideal.

Proof. (B. Ban)
9.1.5

Suppose that pq ∈ (x, y) for some p, q ∈ R = Z[x] and assume that p, q 6∈ (x, y). It means both p and q has a constant term.
However, if so, pq has a constant term since Z is an integral domain which shows pq 6∈ (x, y) and it is a contradiction. Therefore,
either p or q is in (x, y) is it is a prime ideal.

Suppose that pq ∈ (x, y, 2) for some p, q ∈ R and assume that p, q 6∈ (2, x, y). It means both p and q has a odd constant term
and it means pq has odd constant term since Z is an integral domain and odd element times odd element is odd. Therefore,
pq 6∈ (2, x, y) which is a contradiction. Therefore, either p or q is in (2, x, y) and so it is a prime ideal.

Clearly, (x, y) ( (2, x, y) since 2 6∈ (x, y). Also, (2, x, y) 6= R since 3 ∈ R \ (2, x, y). Thus, (x, y) is not a maximal ideal.

Suppose that (2, x, y) ⊆ I ( R. Assume that ∃q ∈ I such that q has a odd constant term. Then collect the monomial terms of q
which have degree bigger than 1 in x, let’s call it xp(x, y), and collect rest of the nonconstant monomial terms, let’s call it yh(x, y).
Then q = xp + yh + c where c is an odd integer. Let c = 2k + 1 for some integer k and observe that 1 = q − (xp + yh + 2k) ∈ I
which means I contains a unit so I = R which is a contradiction. Thus, q does not have a odd constant. Then, q = xq + yh + 2k
for some k ∈ Z, q, h ∈ R. Therefore, q ∈ (2, x, y) so I = (2, x, y). It implies (2, x, y) is a maximal ideal.

9.1.6
Prove that (x, y) is not a principal ideal in Q[x, y].

Page 78
MATH 611 Editor : Byeongho Ban

Proof. (B. Ban) .

Suppose that (x, y) = (p) for some p ∈ Q[x, y]. Then ∃g, h ∈ Q[x, y] such that x = gp and y = hp. Then the degree of gp in x is 1
and that of in y is 0. Therefore, the degree of p in y should be 0. Similarly, the degree of hp in x is 0 and that of in y is 1. Thus,
p should be degree 0 in both variable x and y which means p ∈ Q. Since p is a unit, (p) = Q[x, y] which contradicts to the fact
that Q ∩ (x, y) = ∅ but Q ⊆ Q[x, y].

Proof. (B. Ban)
9.1.7

Note that, when x and y are two distinct indeterminants, we will argue that (x, y) is not a principal ideal in Q = R[x, y, . . . ].
Assume that (p) = (x, y) for some p ∈ Q. Then x, y ∈ (p) so x = ps and y = pw for some s, w ∈ Q. Then p have degree 0 in y due
to first equation and have degree 0 in x due to the second equation. Therefore, p is a constant polynomial. However, since (x, y)
does not have nonzero constant, p = 0 but it is a contradiction since (x, y) 6= 0. Therefore, Q is not a PID.

9.1.8
Let F be a field and let R = F [x, x2 y, x3 y 2 , . . . , xn y n−1 , . . . ] be a subring of the polynomial ring F [x, y].
(a) Prove that the fields of fractions of R and F [x, y] are the same.
(b) Prove that R contains an ideal that is not finitely generated.
Proof. (B. Ban) .

(a)
Let K and E be a fields of fractions of R and F [x, y] respectively. And note that R ⊆ F [x, y] ⊆ E so K ⊆ E. Conversely, note
2
that xx2y ∼ y, so y ∈ K. Therefore, x, y ∈ K so F [x, y] ⊆ K which implies that E ⊆ K so E = K.

(b)
Let I = (x, x2 y, x3 y 2 , . . . , xn y n−1 , . . . ) ⊆ R. Assume that I is finitely generated. Then
I ⊆ (x, x2 y, x3 y 2 , . . . , xN y N −1 )
for some N ∈ Z+ . Then, we have I = (x, x2 y, x3 y 2 , . . . , xN y N −1 ). Then note that we don’t have xN +1 y N in I since if there is
α ∈ R such that α(xn y n−1 ) = xN +1 y N , so α = xN −n+1 y N −n+1 . Since α have same degree in both of variable x and y and there
is no such element in R, α 6∈ R so it is a contradiction. Therefore, I is the ideal of R that is not finitely generated.

Proof. (B. Ban)
9.1.9

Let R be a given commutative ring and let S = R[x1 , x2 , . . . ] be the polynomial ring in infinitely many variables with coefficient in
R. Let T = (x1 , x2 , . . . ). Assume that T be a finitely generated ideal in S so T = (p1 , p2 , . . . , pn ) for some pi ∈ S ∀i ∈ {1, . . . , n}.
Note that each polynomial pi has finitely many monomials, so let ki be an integer such that degree of pi in xki , degxki (pi ) > 0
and ki is the biggest such integer. And let k = max1≤i≤n ki . Then xk+1 6∈ T since T does not have a unit. Therefore, T ( S
which is a contradiction. Therefore, S contains non-finitely generated ideal.

Proof. (B. Ban)
9.1.11

Clearly, since x, y 2 ∈ I, we have x, y ⊆ rad(I). so (x, y) ⊆ rad(I). Let p ∈ rad(I) be given. Note that p = (xq + y 2 h)n for some
q, h ∈ R = Q[x, y] and for some positive integer n. But xq + y(yh) ∈ (x, y) so (xq + y 2 h)n ∈ (x, y) and so rad(I) ⊆ (x, y) which
implies (x, y) = rad(I).

9.1.13
Prove that the rings F [x, y]/(y 2 − x) and F [x, y]/(y 2 − x2 ) are not isomorphic for any field F .
Proof. (B. Ban) .

Let a field F be given. Let ϕ : F [x, y] → F [t] be an homomorphism given by ϕ(x) = t2 , ϕ(y) = t and ϕ(f ) = f ∀f ∈ F . Note
that the kernel of the homomorphism is precisely (y 2 − x), therefore by the 1st isomorphism theorem F [x, y]/(y 2 − x) ∼ = F [t].
Furthermore since F [t] is an integral domain, so is F [x, y]/(y 2 − x). Now, we will show that F [x, y]/(y 2 − x2 ) has zero divisors,
thus is not an integral domain and the conclusion will follow. Observe that (y − x)(y + x) = y 2 − x2 = 0. We are done as neither
(y − x) nor (y + x) is zero.


Page 79
MATH 611 Editor : Byeongho Ban

Proof. (B. Ban)


9.2.1

Let g(x) be given. Then there exists q(x) and r(x) such that
g(x) = q(x)f (x) + r(x) r(x) or N (r) < N (f ) = deg(f ) = n.
Then observe that, since qf ∈ (f (x)), by setting r = g0
g(x) = qf + r = r = g0 .
And, since g0 (x) = 0 or deg(g0 ) < deg(f ) = n, clearly, deg(g0 ) ≤ n − 1.


Proof. (B. Ban)
9.2.2

Note that, from previous exercise, F [x]/(f (x)) is a vector space of dimension n so the order of it should be q n .

Proof. (B. Ban)
9.2.3

Suppose that F [x]/(f (x)) is a field. Then (f (x)) is a maximal ideal. Let pq = f for some p, q ∈ F [x], then, without loss of
generality, (f (x)) ∈ (p(x)). Since (f (x)) is maximal, (p(x)) is (f (x)) or F [x]. If (p) = (f ), p ∈ (f ) so p = f s for some s ∈ F [x].
Then observe that
f = pq = f sq =⇒ f (1 − sq) = 0.
Since f 6= 0, sq = 1 so q is a unit. Otherwise, if (p) = F [x], 1 ∈ (p) so 1 = ph for some h ∈ F [x] which means p is a unit.
Therefore, either p or q is unit and so f is irreducible.

Conversely, suppose that f is irreducible and let (f ) ⊆ M ⊆ F [x]. Since F [x] is a PID, M = (g) for some g ∈ F [x]. Then f ∈ (g)
so f = gk for some k ∈ F [x]. Since f is irreducible, either g or k is a unit. If g is a unit, (g) = F [x]. If k is a unit, then kg ∈ (f )
so g = k −1 kg ∈ k −1 (f ) = (f ) so (g) ⊆ (f ) which means (f ) = (g).

9.2.4
Let F be a finite field. Prove that F [x] contains infinitely many primes. (Note that over an infinite field the polynomials of degree
1 are an infinite set of primes in the ring of polynomials.)
Proof. (B. Ban) .

Assume there are finitely many primes in F [x], let such prime polynomials p1 , p2 , . . . , pn . And let P = p1 p2 · · · pn + 1. Since F [x]
is U.F.D, we have a prime factorization of P and let one of the factor be p. Then p|P and it should not be in the previous list
since otherwise, p should divided 1 which is impossible. Therefore, it is contradiction so we should have infinitely many primes in
F [x].

9.2.5
Exhibit all the ideals in the ring F [x]/(p(x)), where F is a field and p(x) is a polynomial in F [x] (Describe them in terms of the
factorization of p(x).).
Proof. (B. Ban) .

Note that, by fourth isomorphism theorem, there is a one to one correspondence between the ideals of F [x]/(p(x)) and the ideals
containing (p(x)). Since F [x] is PID, every ideal of F [x] has one generator. Let (a(x)) be the ideal of F [x] that contains (p(x)).
Then p(x) ∈ (a(x)) so a(x) should divides p(x). From here, we know every ideal of F [x]/(p(x)) can be described by (a(x))/(p(x))
where a(x) is a divisor of p(x).


Page 80
MATH 611 Editor : Byeongho Ban

Proof. (B. Ban)


9.2.8

By the division algorithm, observe that


a(x) = (x3 − 2) = (x + 1)(x2 − x + 1) + (−3)
 
1
(x + 1) = (−3) − (x + 1) + 0
3
Thus, greatest common divisor of a(x) and b(x) is −3 and
−3 = a(x) − (x2 − x + 1)b(x).

Proof. (B. Ban)
9.2.9

By the division algorithm, observe that


a(x) = (x5 + 2x3 + x2 + x + 1) = (1)(x5 + x4 + 2x3 + 2x2 + 2x + 1) + (−x4 − x2 − x) = b(x) + (−x4 − x2 − x)
b(x) = (−x)(−x4 − x2 − x) + (x4 + x3 + x2 + 2x + 1)
(−x4 − x2 − x) = (−1)(x4 + x3 + x2 + 2x + 1) + (x3 + x + 1)
(x4 + x3 + x2 + 2x + 1) = (x + 1)(x3 + x + 1).
Thus, the greatest common divisor of them is, after simple calculations, g(x) = x3 + x + 1 = (x + 1)a(x) − xb(x).

9.3.1
Let R be an integral domain with quotient field F and let p(x) be a monic polynomial in R[x]. Assume that p(x) = a(x)b(x)
√ in F [x] of smaller degree than p(x). Prove that if a(x) 6∈ R[x] then R is not a Unique
where a(x) and b(x) are monic polynomials
Factorization Domain. Deduce that Z[2 2] is not a U.F.D.
Proof. (B. Ban) .

Assume that R is UFD, then, by Gauss’s lemma, there are r, s ∈ F such that ra(x) = a0 (x) ∈ R[x] and sb(x) = b0 (x) ∈ R[x]. Since
a(x) and b(x) are monic, s, r ∈ R and note that p(x) is monic, so rs = 1.Therefore, a(x) = sra0 (x) ∈ R[x] and b(x) = rsb0 (x) ∈ R[x].
Thus, we proved contrapositive√ of the √
statement. √ √ √ √
Furthermore, note that (x + 2)(x − 2) = x2 − 2 ∈ Z[2 2][x] but x + 2 6∈ Z[2 2][x] so Z[2 2] is not a UFD.

9.3.4
Let R = Z + xQ[x] ⊂ Q[x] be the set of polynomials in x with rational coefficients whose constant term is an integer.
(a) Prove that R is an integral domain and its units are ±1.
(b) Show that the irreducibles in R are ±p where p is a prime in Z and the polynomials f (x) that are irreducible in Q[x] and
have constant term ±1. Prove that these irreducibles are prime in R.
(c) Show that x cannot be written as the product of irreducibles in R ( in particular, x is not irreducible) and conclude that R
is not a U.F.D.
(d) Show that x is not a prime in R and describe the quotient ring R/(x).

Page 81
MATH 611 Editor : Byeongho Ban

Proof. (B. Ban) .

(a)
Note that 1 ∈ R, so R 6= ∅. And observe that, for any n, m ∈ Z and for any q(x), p(x) ∈ Q[x],
(n + xp(x)) − (m + xq(x)) = (n − m) + x(p(x) − q(x)) ∈ R
(n + xp(x))(m + xq(x)) = nm + x(nq(x) + mp(x) + xp(x)q(x)) ∈ R.
This says R is a subring of Q[x]. Now, suppose that, for given n, m ∈ Z and p(x), q(x) ∈ Q[x], 0 = (n + xp(x))(m + xq(x)). Then
since 0 is of zero degree, nm = 0. Since Z is integral domain, n or m should be zero. Then n + xp(x) = 0 or m + xq(x) = 0 so R
is an integral domain. Lastly, if n + xp(x) ∈ R is a unit such that (n + xp(x))(m + xq(x)) = 1, since 1 is of zero degree, mn = 1.
Then m = ±1 and n = ±1. Therefore, n + xp(x) = ±1 so only units are ±1.

(b)
Firstly, consider ±p for some prime p ∈ Z. Note that if ±p = a(x)b(x) ∈ R, the degrees of a(x) and (b) should be 0. Since primes
in Z are irreducible, a(x) or b(x) is a unit. Hence f (x) is irreducible in R.

Secondly, consider when f (x) is an irreducible polynomial in Q[x] of degree at least 1, and constant term ±1. If f (x) = a(x)b(x) in
R, then since f (x) is irreducible, one of a(x) or b(x) is a unit. Without loss of generality, let a(x) be a unit. Then a(x) ∈ Q \ {0}.
However, since a(x) ∈ R, and a(x) is degree 0, a(x) ∈ Z which implies that the constant term of b(x) is an integer since the
constant term of f (x) is an integer. Thus a(x) = ±1. Thus, f (x) is irreducible in R.

Now suppose f (x) is an irreducible polynomial in R. If the degree of f (x) is 0, then f (x) = ±p for some prime number p ∈ Z.

If the degree of f (x) is greater than 0, then its constant term c of f (x) may only be ±1. If c = 0, we have f (x) = d1 xg(x) where d
is the denominator of the linear term of f (x). Note that g(x) and xd are nonunit so it makes f (x) reducible. If c 6= 0 and c would
not be a unit, then f (x) = c( 1c (f (x)−c)+1). Note that c and 1c (f (x)−c)+1 are nonunit so makes f (x) reducible. Therefore, c = ±1.

Assume that f (x) = g(x)x ± 1 where g(x) ∈ Q[x] is reducible in Q[x] and factors as a product of nonunits f (x) = a(x)b(x). Both
of a(x) and b(x) must have nonzero constant terms, call them a0 and b0 in Q, then a0 b0 = ±1. Then f (x) = ±1( a10 a(x))( b10 b(x))
is a factorization of f (x) into nonunits in R. Hence f (x) must be reducible in R. Thus, if f (x) has degree more than 1 and
irreducible, it should be irreducible in Q[x] and should have a constant term ±1.

(c)
If x = p1 (x) · · · pn (x) for irreducible elements pi (x), then at least on of pi ’s constant term should be 0 and it is a contradiction
since then the pi should be 0 or reducible.

(d)
First, consider the elements 2 and 12 x of R. Note that neither is contained in (x). But 2( 12 x) = x ∈ (x). Hence (x) is not a prime
ideal, and x is not prime in R.

In particular, the ring R/(x) is not an integral domain. Two elements f (x) and g(x) are equal in R/(x) if f (x) − g(x) is a
polynomial with no constant term, and whose degree 1 term has an integer coefficient. Therefore every element can be represented
uniquely by a polynomial ax + b, where a ∈ [0, 1) ∩ Q, and b ∈ Z. Further, all elements other than ±1 are zero divisors. This is
because if ax + b 6= ±1, 0, then if b 6= 0, 1b x 6∈ (x) and 1b x(ax + b) ∈ (x). If b = 0, 21 x(ax) ∈ (x).


Page 82
MATH 611 Editor : Byeongho Ban

Proof. (B. Ban)


9.4.1

(a)
p1 (x) = x2 + x + 1 is irreducible in F2 [x].

Recall that a polynomial degree less then 4 is reducible if and only if it had a linear factor if and only if p1 has a root. However,
p1 (0) = 1 6= 0 and p1 (1) = 1 6= 0 so p1 does not have a root. Therefore, p1 is irreducible.

(b)
p2 = x3 + x + 1 is reducible in F3 [x].

From the argument above, p2 is reducible if and only if p2 has a root in F3 . Observe that p2 (1) = 0. Thus, p2 is reducible.

(c)
p3 (x) = x4 + 1 is reducible in F5 [x].

Note that x4 + 1 = x4 − 4 = (x2 − 2)(x2 + 2). Thus, it is reducible.

(d)
p4 (x) = x4 + 10x2 + 1 is irreducible.

If it was reducible, there are two possibility that p4 has linear factor or not. If it has, p4 has a root in Z. Note that possible
rational root of it are ±1. However, p4 (±1) = 12 6= 0. Thus, it should be factored into two quadratic polynomials. Then we have
p4 (x) = (x2 + nx + 1)(x2 + mx + 1) = x4 + (n + m)x2 + (2 + nm)x2 + (n + m)x + 1.
Then n + m = 0 and 2 + nm = 10 which implies n2 = −8 which is a contradiction. Therefore, p4 should have been irreducible.

9.4.2
Prove that the following polynomials are irreducible in Z[x] :
(a) x4 − 4x3 + 6
(b) x6 + 30x5 − 15x3 + 6x − 120
(c) x4 + 4x3 + 6x2 + 2x + 1 [Substitute x − 1 for x.]
p p
(d) (x+2)x −2 , where p is an odd prime.
Proof. (B. Ban) .

(a)
Note that 2 is a prime and 2|6 and 2|4 but 22 6 |6. Therefore, by Eisenstein’s Criterion with p = 2, the given polynomial is irreducible.

(b)
Note that 3 is a prime. And observe that 3 divides every coefficients of the polynomial except for leading coefficient. Also, since
32 does not divides constant term, by Eisenstein’s Criteria, the polynomial is irreducible.

(c)
Let p(x) = x4 + 4x3 + 6x2 + 2x + 1. Then observe that
p(x − 1) = (x − 1)4 + 4(x − 1)3 + 6(x − 1)2 + 2(x − 1) + 1 = x4 − 2x + 2.
Since 2 divides each coefficients except for leading coefficient and 22 does not divides the constant term, by Eisenstein’s Criteria,
p(x − 1) is irreducible. Thus, p(x) is irreducible.

(d)
Observe that
Pp p! k p−k
Pp−1 p! p−1
(x + 2)p − 2p k=0 (p−k)!k! 2 x − 2p k p−k
k=0 (p−k)!k! 2 x X p!
= = = 2k xp−k−1 .
x x x (p − k)!k!
k=0
2
Note that p divides each coefficients except for the leading coefficient and p does not divides the constant term p. Therefore, by
Eisenstein’s Criteria, the polynomial is irreducible.


Page 83
MATH 611 Editor : Byeongho Ban

Proof. (B. Ban)


9.4.3


9.4.5
Find all the monic irreducible polynomials of degree ≤ 3 in F2 [x], and the same in F3 [x].
Proof. (B. Ban) .

In F2 [x], every form of x + a for all a ∈ F2 [x] are monic polynomials of the ring: x, x + 1. And recall that when the degree is two
or three, the given polynomial is irreducible if and only if it does not have any root in F2 . Note that
RE = {x3 , x2 (x + 1), x(x + 1)2 , (x + 1)3 , x2 , x(x + 1), (x + 1)2 } = {x3 , x3 + x2 , x3 + x, x3 + x2 + x + 1, x2 , x2 + x, x2 + 1}
is a set of some reducible polynomials of F2 [x]. And note that
B = {x3 + x2 + x + 1, x3 + x + 1, x3 + x2 + 1, x3 + x2 + x, x3 + x2 , x3 + x, x3 + 1, x3 , x2 + x + 1, x2 + 1, x2 + x, x2 }
is a set of all polynomials in F2 [x] of degree two and three. Then let
B \ RE ∪ {x, (x + 1)} = {x3 + x2 + x, x3 + x + 1, x3 + x2 + 1, x3 + 1, x2 + x + 1, x, x + 1}
Note that we lastly need to check if any of the polynomial has a root in F2 . Note that x3 + x2 + x = x(x2 + x + 1) and
x3 + 1 = (x + 1)(x2 + x + 1). Thus, the set of monic irreducible polynomials if F2 [x] is
{x3 + x + 1, x3 + x2 + 1, x2 + x + 1, x, x + 1}.

Similarly, note that x, x + 1 x + 2 are only degree one monic polynomials in F3 [x]. And note that
RE3 = {x, x + 1, x + 2, x3 , x2 (x + 1), x2 (x + 2), x(x + 1)2 , x(x + 1)(x + 2), x(x + 2)2 ,
(x + 1)3 , (x + 1)2 (x + 2), (x + 1)(x + 2)2 , (x + 2)3 , x2 , x(x + 1), x(x + 2), (x + 1)2
, (x + 1)(x + 2), (x + 2)2 }
is a set of all reducible polynomials of degree ≤ 3 in F3 [x]. Note |RE3 | = 19 and |F3,≤3 [x]| = 3 + 9 + 27 = 39 where F3,≤3 [x] is the
set of all monic polynomials in F3 [x] of degree ≤ 3. Therefore, there are 20 number of monic irreducible polynomials of degree
≤ 3 in F3 [x] and the set of them is F3 [x] \ RE3 .

9.4.8
Prove that K1 = F11 [x]/(x2 + 1) and K2 = F11 [y]/(y 2 + 2y + 2) are both fields with 121 elements. Prove that the map which
sends the element p(x) of K1 to the element p(y + 1) of K2 ( where p is any polynomial with coefficients in F11 ) is well defined
and gives a ring (hence field) isomorphism from K1 to K2 .
Proof. (B. Ban) .

Note that any element of K1 can be described by ax + b + (x2 + 1) for some a, b ∈ F11 . Then, since there are 11 possible values for a
and b so |K1 | = 11·11 = 121. Similarly, any element of K2 can be described by ay +b+(y 2 +2y +2) so |K2 | = 121. Note that x2 +1
and y 2 +2y+2 has no root in F11 which implies (x2 +1) and (y 2 +2y+2) are maximal ideals in each PID. Thus, K1 and K2 are field.

Let ϕ : K1 → K2 defined by ax + b + (x2 + 1) 7→ ay + (a + b) + (y + 2y + 2). Observe that


ϕ(ax + b + (x2 + 1)) = ϕ(cx + d + (x2 + 1)) ⇐⇒ ay + (a + b) + (y + 2y + 2) = cy + (c + d) + (y 2 + 2y + 2)
⇐⇒ a = c and b = d
⇐⇒ ax + b + (x2 + 1) = cx + d + (x2 + 1).
So ϕ is well defined and injective. Nextly, note that
ϕ((ax + b + (x2 + 1)) + (cx + d + (x2 + 1))) = (a + c)y + (a + b + c + d) + (y 2 + 2y + 2)
= ay + (a + b) + (y 2 + 2y + 2) + cy + (c + d) + (y 2 + 2y + 2)
= ϕ(ax + b + (x2 + 1)) + ϕ(cy + d + (x2 + 1))
and that

ϕ((ax + b + (x2 + 1))(cx + d + (x2 + 1))) = (ay + (a + b))(cy + (c + d)) + (y 2 + 2y + 2)


= ϕ(ax + b + (x2 + 1))ϕ(cy + d + (x2 + 1)).
Therefore, ϕ is an injective ring homomorphism between K1 and K2 . Since |K1 | = |K2 |, ϕ is an isomorphism.


Page 84
MATH 611 Editor : Byeongho Ban

9.4.11
Prove that x2 + y 2 − 1 is irreducible in Q[x, y].
Proof. (B. Ban) .

Let R = Q[y]. Since R is a UFD and y − 1 ∈ R is irreducible, y − 1 is a prime. Since y 2 − 1 = (y − 1)(y + 1), y − 1|y 2 − 1. Now,
note that x2 + (y 2 − 1) ∈ R[x] and (y − 1)2 does not divides (y 2 − 1), so by Eisenstein’s Criteria with the prime p = (y − 1),
x2 + y 2 − 1 is irreducible in R[x] = Q[x, y].

9.5.1
Let F be a field and let f (x) be a nonconstant polynomial in F [x]. Describe the nilradical of F [x]/(f (x)) in terms of the
factorization of f (x) (cf. Exercise 29, Section 7.3).
Proof. (B. Ban) .

Note that F [x] is UFD. Thus, there is a unique factorization of f (x) = upα α2 αn
1 (x)p2 (x) · · · pn (x) where n and each αi is a positive
1

integer, each pi is irreducible polynomial in F [x] and u is an element of F which is also a leading term. Since q(x) = 0 if and only
if q m (x) ∈ (f (x)) for some positive integer m if and only if q(x) has all pi as a factor. Therefore, the nilradical of R = F [x]/(f (x)),
N(R), should be
N(R) = {s(x)pβ1 1 (x)pβ2 2 (x) · · · pβnn : βi > 0, 1 ≤ ∀i ≤ n and s(x) ∈ F [x]}.

Throughout section 10.1, R is a ring with identity ,1, and M is a left R−module.

Proof. (B. Ban)


10.1.1

Let m ∈ M be given. Then observe that


0m = (0 + 0)m = 0m + 0m =⇒ 0m = 0
(−1)m + m = (−1)m + (1)m = (1 + (−1))m = 0m = 0 =⇒ (−1)m = −m.

Proof. (B. Ban)
10.1.2

Let a, b ∈ R× and m ∈ M be given. Then observe that am ∈ M since a ∈ R× ⊂ R. Also, observe that (ab)m = a(bm) by the
definition of module. Also, 1m = m by the definition of unitality. Therefore, R× acts on M .

Proof. (B. Ban)
10.1.3

Assume that r does have the left inverse s ∈ R so sr = 1. Then observe that
0 = s0 = s(rm) = (sr)m = 1m = m
and it is a contradiction since m 6= 0.


Page 85
MATH 611 Editor : Byeongho Ban

Proof. (B. Ban) 10.1.4

Let M = Rn be a module and let I1 , . . . , In be left ideals of R.

(a)
Let S = {(x1 , . . . , xn ) : xi ∈ Ii }. Firstly, note that (0, . . . , 0) ∈ S since 0 ∈ Ii for each i, so S 6= ∅. Let (x1 , . . . , xn ), (y1 , . . . , yn ) ∈ S
and r ∈ R be given. And observe that
(x1 , . . . , xn ) + r(y1 , . . . , yn ) = (x1 + ry1 , . . . , xn + ryn ) ∈ S
since Ii is an ideal so xi + ryi ∈ Ii for each i. Therefore, S is a submodule of M .

(b)
Let T = {(x1 , . . . , xn ) : xi ∈ R and x1 + · · · + xn = 0}. Note that (0, . . . , 0) ∈ T since 0 + · · · + 0 = 0, so T 6= ∅. Let
(x1 , . . . , xn ), (y1 , . . . , yn ) ∈ T and r ∈ R be given. And observe that
(x1 , . . . , xn ) + r(y1 , . . . , yn ) = (x1 + ry1 , . . . , xn + ryn ).
Further observe that
(x1 + ry1 ) + · · · + (xn + ryn ) = (x1 + · · · + xn ) + r(y1 + · · · + yn ) = 0.
Thus, (x1 , . . . , xn ) + r(y1 , . . . , yn ) ∈ T , so T is a submodule of M .

Proof. (B. Ban) 10.1.5

For any left ideal I of R, define


 
X 
IM = ai mi : ai ∈ I, mi ∈ M
 
f inite

to be the collection of all finite sums of elements of the form am where a ∈ I and m ∈ M .

Let a left ideal I of R be given. Note that 0 ∈ I and 0 ∈ M , so 0 ∈ IM so IM 6= ∅. And let


X X
ai mi , bj nj ∈ IM
1≤i≤n 1≤j≤m

and r ∈ R be given. And observe that


X X X
ai mi + r bj n j = ck zk
1≤i≤n 1≤j≤m 1≤k≤n+m

where
( (
1≤k≤n
ak mk 1 ≤ k ≤ n
ck = zk =
n+1≤k ≤n+m
rbk nk n + 1 ≤ k ≤ n + m.
P
Since I is an ideal and bk ∈ I, rbk ∈ I, so the sum 1≤k≤n+m ck zk ∈ IM . Therefore, IM is a submodule of M .

Proof. (B. Ban) 10.1.6
T
Let {Sα }α∈A be a nonempty collection(A 6= ∅) of submodules of a R-module M . Note that 0 ∈ Sn for each α ∈ A, so 0 ∈ α Sn 6= ∅.

Let
\
S= Sα .
α∈A
Let x, y ∈ S and r ∈ R, then x, y ∈ Sα for each α, so x + ry ∈ Sα for each α ∈ A. Therefore, x + ry ∈ S, so S is an submodule.

Proof. (B. Ban)
10.1.7
S∞
Let N = i=1 Ni . Since ∅ =6 N1 ⊂ N , N 6= ∅. Now, let m, n ∈ N and r ∈ R be given. Then there exists positive integers km and
kn such that m ∈ Nkm and n ∈ Nkn . Without loss of generality, let kn ≤ km . Then by the ascending condition, n, m ∈ Nkm so
n + rm ∈ Nkm since it is a submodule. However, since Nkm ⊆ N , n + rm ∈ N so N is again a submodule.


Page 86
MATH 611 Editor : Byeongho Ban

Proof. (B. Ban) 10.1.8

(a)
Note that r0 = 0 for any nonzero r ∈ R, so 0 ∈ T or(M ), so T or(M ) 6= ∅. Let m, n ∈ T or(M ) and r ∈ R be given. Then
∃a, b ∈ R \ {0} such that am = 0 and bn = 0. Now, note that ab 6= 0 since R is an integral domain. And, due to commutativity,
observe that
ab(m + rn) = abm + abrn = b(am) + ar(bn) = 0.
Therefore, m + rn ∈ T or(M ), so T or(M ) is a submodule.

(b)
Let M = Z/6Z = R. Note that
T or(M ) = {0, 2, 3, 4}
since there is no n ∈ {1, 2, 3, 4, 5} such that 5n ≡ 0 and 1n ≡ 0 (mod6). However, 3 − 2 = 1 6∈ T or(M ) so T or(M ) is not a
submodule.

(c)
Suppose that R has zero divisors and let M be a given R−module. Let m ∈ M be given and let u ∈ R \ {0} be the zero divisor.
Then ∃v ∈ R \ {0} such that vu = 0. Now, note that um ∈ M and v(um) = (vu)m = 0. Thus, um is the torsion element.

Proof. (B. Ban) 10.1.9

Suppose that N is a submodule of M . Let A = {r ∈ R : rn = 0 ∀n ∈ N }. And let r, s ∈ A be given. Observe that


(r − s)n = rn − sn = 0 − 0 = 0 ∀n ∈ N.
So r − s ∈ A. Also, for given t ∈ R, note that (tr)n = t(rn) = t0 = 0 and (rt)n = r(tn) = 0 since tn ∈ N . Therefore, tA ⊆ A and
At ⊆ A for any t ∈ R. Therefore, A is a two sided ideal of R.

Proof. (B. Ban)
10.1.10

Let N = {m ∈ M : am = 0 for all a ∈ I}. Note that a0 = 0 ∀a ∈ I, so N 6= ∅. Nextly, Let n, m ∈ N and r ∈ R be given. Then
observe that, for given a ∈ I,
a(n + rm) = an + arm = 0 + 0 = 0.
since ar ∈ I where I is the right ideal. Therefore, since a ∈ I was arbitrary, n + rm ∈ N so N is a submodule.

Proof. (B. Ban)
10.1.11

(a)
If am = 0 for all m ∈ M , 24|a, 15|a and 50|a. Then the set of all a satisfying such condition would be kZ where k is a least
common multiplier of 24, 15 and 50 which is 600. So the annihilaor of M in Z is 600Z.

(b)
12 (Z/24Z) × {0} × 25 (Z/50Z).


Page 87
MATH 611 Editor : Byeongho Ban

Proof. (B. Ban)


10.1.12

(a)
Let n ∈ N be given. Then rn = 0 for all r ∈ I since I is the annihilator of N . Thus, n is in the annihilator of I. Therefore, the
annihilator of I contains N .

Suppose that R = Z and M = Z/2Z × Z/6Z. Then the annihilator of a submodule of M , N = Z/2Z × {0}, in R is I = 2Z.
However, the annihilator of I in M is Z/2Z × 3 (Z/6Z) which properly contains N .

(b)
Let r ∈ I be given. Then for any n ∈ N , rn = 0 since N is the annihilator of I in R. Thus, r is in the annihilator of N and it
contains I.

Let R = Z and M = Z/2Z × Z/6Z. Note I = 4Z is a right ideal of R and its annihilator in M is N = Z/2Z × 3 (Z/6Z). However,
the annihilator of N in R is 2Z which contains 4Z properly.

Proof. (B. Ban)
10.1.13

that I ⊇ I 2 ⊇ I 3 ⊇ · · · . Let the annihilator of I k in M be Mk . Then M1 ⊆ M2 ⊆ M3 ⊆ · · · . Since Mk ⊆ M 0 for each k,


Note S

N = k=1 Mk ⊆ M 0 . Conversely, for given n ∈ M 0 , ∃k such that n ∈ Mk so M 0 ⊆ N , so M 0 = N . Then, from the exercise
10.1.7, N is a submodule so M 0 is a submodule of M .

Proof. (B. Ban)
10.1.14

Firstly, since z0 = 0 ∈ M so zM 6= ∅. Then observe that, for any zn, zm ∈ zM and any r ∈ R,
zn + r(zm) = zn + (rz)m = zn + (zr)m = zn + z(rm) = z(n + rm) ∈ zM
since M is a module which implies that zM is a submodule.

Observe that
            
1 0 0 1 0 1 1 0 1 0 0 1 0 0 0 1
+ = + = 6∈ eR.
0 0 1 0 1 0 0 0 0 1 0 0 1 0 1 0
Thus, eR is not a left submodule of M = R.

Proof. (B. Ban)
10.1.15

Suppose when M = Z/2Z and R = Z. If we can extend M into Q−module, we should have 12 n ∈ M for any n ∈ M . And observe
that
 
1 1 1 1
n+ n= + n = n.
2 2 2 2
However, for any m ∈ M , m + m = 2m = 0 which implies the contradiction. Therefore, we cannot extend any finite abelian group
into Q−module.

Proof. (B. Ban)
10.1.17

(am xm + am−1 xm−1 + · · · + a0 )en = an−1 e1 + an−2 e2 + · · · + a0 en . So we are done.



Proof. (B. Ban) 10.1.18

Suppose that N be a nonzero submodule of V . Then ∃m ∈ V \ {(0, 0)}. Since N is R[x] module, x · m = T m ∈ V . Let
n = T m. Then {n, m} is a basis of V . Thus, for any (a, b) ∈ V , ∃r, s ∈ R such that rm + sn = (a, b). But m + (s/r)n ∈ N so
r(m + (s/r)n) = rm + sn ∈ N . Thus, N = V . Therefore, only submodule of V is (0) and V .


Page 88
MATH 611 Editor : Byeongho Ban

Proof. (B. Ban) 10.1.19

Firstly, observe that the action of xn for any n ≤ 1 on V is equivalent to the action of x on V since projecting element onto y−axis
two times is just same as projecting one time. Thus, every action of R[x] onto V can be reduced to action of ax + b for arbitrary
a, b ∈ R. And observe that
(u, v) + (a + bx) · (u0 , v 0 ) = (u, v) + (au0 , av 0 ) + (0, bv 0 ) = (u + au0 , v + (a + b)v 0 ).
If N is a submodule, the element above should be in N for any (u, v), (u0 , v 0 ) ∈ N and any a, b ∈ R. In particular, setting b = 0
and a = 1, it should be closed under addition, by setting b = 0 and (u, v) = (0, 0), it should be closed under scalar multiplication .
Therefore, any submodule is a subspace. Clearly, V , 0, every lines through origin are subspace. Clearly, V and 0 are submodule.
And observe that
(u, 0) + (a + bx) · (u0 , 0) = (u, 0) + (au0 , 0) + (0, 0) = (u + au0 , 0) ∈ x − axis
(0, v) + (a + bx) · (0, v 0 ) = (0, v) + (0, av 0 ) + (0, bv 0 ) = (0, v + av 0 + bv 0 ) ∈ y − axis
Thus, x, y−axis are two submodule. It we exclude these four submodule, we have tilted lines through the origin. However, the
tilted lines are not submodule. Suppose that, for some s, r ∈ R \ {0}, l = {(st, rt) : t ∈ R}. And note that x · (s, r) = (0, r) 6∈ l.
Thus, the four submodules are only submodules.

Proof. (B. Ban) 10.1.20

Clearly, V and 0 are submodules. Since submodule should be closed under addition and R action (scalar multiplication in this
case), the only possible submodules are lines through the origin. Let s, r ∈ R be given and let L = {(st, rt) : t ∈ R}. Since
(0, 0) ∈ L, L 6= ∅. Also, note that any two consecutive rotations make the action identity. Thus, we can represent all action of
R[x] onto V as action of {a + bx : a, b ∈ R}. Now observe that, for any (st, rt), (st0 , rt0 ) and a, b ∈ R,
(st, rt) + (a + bx)(st0 , rt0 ) = (st, rt) + (ast0 , art0 ) + (−bst0 , −brt0 ) = (s(t + at0 − bt0 ), r(t + at0 − bt0 )) ∈ L.
Therefore, any lines are also submodules. Thus, any subspace of V is submodule.

Proof. (B. Ban)
10.2.1

Let ϕ : M → N be a R−module homomorphism. And observee that 0 ∈ ker(ϕ) and 0 ∈ Im(ϕ) so ker(ϕ) 6= ∅ and Im(ϕ) 6= ∅.
Now, let x, y ∈ ker(ϕ) and r ∈ R be given. Then observe that
ϕ(x + ry) = ϕ(x) + rϕ(y) = 0 =⇒ x + ry ∈ ker(ϕ).
Thus, ker(ϕ) is a submodule.

Also, observe that, for given x, y ∈ Im(ϕ) there exist x0 , y 0 ∈ M such that ϕ(x0 ) = x and ϕ(y 0 ) = y so that
x + ry = ϕ(x0 ) + rϕ(y 0 ) = ϕ(x0 + ry 0 ) =⇒ x + ry ∈ Im(ϕ) ∀r ∈ R.
Thus, Im(ϕ) is a submodule.


Page 89
MATH 611 Editor : Byeongho Ban

Proof. (B. Ban) 10.2.2

Let M , N and V be R−modules.


Reflexive
The identity map I : M → M is a clearly R−module homomorphism which is bijective. Therefore, M ∼
= M.

Symmetry
Suppose that M ∼= N so there is a R−module isomorphism ϕ : M → N . We argue that ϕ−1 : N → M is a R−module isomorhpism.
Let x, y ∈ N and r ∈ R be given. Then there are x0 , y 0 ∈ M such that ϕ(x0 ) = x and ϕ(y 0 ) = y. Then observe that
ϕ−1 (x + ry) = ϕ−1 (ϕ(x0 ) + rϕ(y 0 )) = ϕ−1 (ϕ(x0 + ry 0 )) = x0 + ry 0 = ϕ−1 (x) + rϕ−1 (y).
Thus, ϕ−1 is a R−module isomorphism so N ∼ = M.

Transitive
Suppose that ϕ : M → N and ψ : N → V are given R−module isomorphisms (i.e. M ∼
= N and N ∼
= V ). Now, let x, y ∈ M and
r ∈ R be given. Then observe that
ψ ◦ ϕ(x + ry) = ψ(ϕ(x) + rϕ(y)) = ψ(ϕ(x)) + rψ(ϕ(y)) = ψ ◦ ϕ(x) + rψ ◦ ϕ(y).
Thus, ψ ◦ ϕ : M → V is a bijective R−module homomorphism so M ∼ =V.

Therefore, R−module isomorphic relation is a equivalence relation.



Proof. (B. Ban) 10.2.3

Let’s consider R = C and its module itself. Let’s define a map ϕ : C → C defined by ϕ(z) = z where z is a complex conjugation
of z. Observe that ϕ(z + w) = z + w = z + w = ϕ(z) + ϕ(w). Thus, ϕ is a group homomorphism under addition. However, it
cannot be a C−module homomorphism. For example observe that
i · ϕ(i) = i(−i) = 1 6= −1ϕ(−1) = ϕ(i · i).
Therefore, ϕ is the group homomorphism but not a R−module homomorphism we were findining.


Page 90
MATH 611 Editor : Byeongho Ban

Proof. (B. Ban)


10.2.4

Suppose that ϕa : Z/nZ → A given by ϕa (k) = ka is well defined Z−module homomorphism. Then na = ϕa (n) = ϕa ((0)) = 0
since ϕa is a module homomorhpism.

Conversely, suppose that na = 0. Then since Z is commutative,


(k + nm)a = ka + nma = ka + mna = ka + m(na) = ka + 0 = ka = ka + 0 = ka + j(na) = ka + (jn)a = ka + (nj)a = (k + nj)a.
Thus, ϕ is well defined. Also, observe that, for given k, j ∈ Z/nZ and m ∈ Z,
ϕa (k + mj) = ϕa (k + mj) = (k + mj)a = ka + (mj)a = ϕa (k) + mϕa (j).
Thus, ϕa is a Z-module homomorphism.

Secondly let ψ : An → HomZ (Z/nZ, A) be given by ψ(a) = ϕa defined as above. For any a, b ∈ An and m ∈ Z, observe that
ψ(a + mb)(k) = ϕa+mb (k) = k(a + mb)
= ka + k(mb) = ka + (km)b
= ka + (mk)b = ka + m(kb)
= ϕa (k) + mϕb (k)
= (ψ(a) + mψ(b))(k) ∀k ∈ Z/nZ.
Thus, ψ is a Z−module homomorphism.
Now, let e : HomZ (Z/nZ, A) → An defined by e(ϕ) = ϕ(1). Then observe that, for given ϕ, η ∈ HomZ (Z/nZ, A) and m ∈ Z,
e(ϕ + mη) = (ϕ + mη)(1) = ϕ(1) + mη(1) = e(ϕ) + me(η).
Thus, e is also Z−module homomorphism. Then observe that, for given η ∈ HomZ (Z/nZ, A),
ψ ◦ e(η) = ψ(η(1)) = ϕη(1)
and that
ϕη(1) (k) = kη(1) = η(k) ∀k ∈ Z/nZ =⇒ ψ ◦ e(η) = η.
And, also observe that, for given a ∈ An ,
e ◦ ψ(a) = e(ϕa ) = ϕa (1) = a.
∼ An .
Thus, e is two sided inverse of ψ so HomZ (Z/nZ, A) =

Proof. (B. Ban)
10.2.5

Proof. (B. Ban)
10.2.6

By the exercise 10.2.4, we know


HomZ (Z/nZ, Z/mZ) ∼
= {k ∈ Z/mZ : nk = 0} = Z.
Let ψ : Z → Z/(n, m)Z given by ψ(k) = k. For given k, g ∈ Z and m ∈ Z, observe that
ψ(k + mg) = ψ(k + mg) = k + mg = k + mg
so ψ is a Z−module homomorphism.
Also, let η : Z/(n, m)Z → Z defined by ηk = k. Then, for given k, h ∈ Z/(n, m)Z and m ∈ Z, observe that
η(k + mh) = η(k + mh) = k + mh = k + mh
so η is also Z−module homomorphism.

It is clear that η and ψ are well defined. Also, we can note that ψ ◦ η and η ◦ ψ are identity maps so Z ∼
= Z/(n, m)Z. We are done.


Page 91
MATH 611 Editor : Byeongho Ban

Proof. (B. Ban)


10.2.7

Let ψ : z 7→ mz. Then, for given m, n ∈ M and r ∈ R, observe that, ψ(m) = zm ∈ M and
ψ(m + rn) = z(m + rn) = zm + z(rn) = zm + (zr)n = zm + (rz)n = zm + r(zn) = ψ(m) + rψ(n).
Thus, ψ is a R−module homomorphism from M to itself.

Let ϕ(r) = rI given in the problem. Let r, s, v ∈ R be given. Then observe that
ϕ(r + vs) = (r + vs)I = rI + (vs)I = rI + v(sI) = ϕ(r) + vϕ(s).
Thus, ϕ is a R−module homomorphism.

Proof. (B. Ban) 10.2.8

Let m ∈ T or(M ) be given. Then there is r ∈ R \ {0} such that rm = 0. Then observe that
rϕ(m) = ϕ(rm) = ϕ(0) = 0.
Thus, ϕ(m) ∈ T or(N ) which implies that ϕ(T or(M )) ⊆ T or(N ).

Proof. (B. Ban) 10.2.9

Let R be a commutative ring. And consider the map Φ : HomR (R, M ) → M defined by Φ(ϕ) = ϕ(1). Observe that it is a
R−module homomorphism since, for any ϕ, ψ ∈ HomR (R, M ),
Φ(ϕ + ψ) = (ϕ + ψ)(1) = ϕ(1) + ψ(1) = Φ(ϕ) + Φ(ψ)
rΦ(ψ) = rϕ(1) = Φ(rϕ).
Observe that Ker(Φ) = {ϕ ∈ HomR (R, M ) : ϕ(1) = 0}. Note that
ϕ(1) = 0 ⇐⇒ ϕ(r) = rϕ(1) = 0 ∀r ∈ R ⇐⇒ ϕ ≡ 0.
Thus, Ker(Φ) = {0}, so it is injective. Lastly, let m ∈ M be given. Then consider φ : R → M defined by φ(r) = rm. Observe
that, for any s, r, t ∈ R
φ(s + rt) = (s + rt)m = sm + r(tm) = φ(s) + rφ(t).
Therefore, φ ∈ HomR (R, M ) and Φ(φ) = φ(1) = m so Φ is surjective. In conclusion, Φ is an isomorphism between HomR (R, M )
and M , so they are isomorphic.

Proof. (B. Ban) 10.2.10

Let ψ : HomR (R, R) → R be defined by ψ(ϕ) = ϕ(1). Then, for any η, φ ∈ HomR (R, R) and r ∈ R,
ψ(φ + rη) = (φ + rη)(1) = φ(1) + rη(1) = ψ(φ) + rψ(η)
so ψ is a R− module homomorphism. On the other hands, let ξ : R → HomR (R, R) be defined by ξ(r)(s) = sr for all s ∈ R.
Then, for any r, s, v ∈ R, observe that
ξ(r + vs)(h) = h(r + vs) = hr + h(vs) = hr + (hv)s = hr + v(hs) = ξ(r)(h) + vξ(s)(h) ∀h ∈ R.
Thus, ξ is a R−module homomorphism. Then observe that
ψ ◦ ξ(r) = ψ(ξ(r)) = ξ(r)(1) = r
so it is identity.

Also, observe that


ξ ◦ ψ(η) = ξ(η(1)) ∀η ∈ HomR (R, R)
and that
ξ(η(1))(r) = rη(1) = η(r) ∀r ∈ R.
Thus, ξ(η(1)) = η so ξ ◦ ψ is identity. Therefore, ξ is a two sided inverse of ψ so R ∼
= HomR (R, R).


Page 92
MATH 611 Editor : Byeongho Ban

Proof. (B. Ban) 10.2.11

For convenience, let A = A1 × · · · × An , B = B1 × · · · Bn and C = (A1 /B1 ) × · · · (An /Bn ). Let ψ : A → C be given by
ψ(a1 , . . . , an ) = (a1 mod B1 , . . . , an mod Bn ). Then clearly, ψ is surjective. Then by first module isomorphism theorem, we have
A/ker(ψ) = ∼ C.
And it is also clear that ker(ψ) = B. Therefore, A/B ∼
= C as we desired.

Proof. (B. Ban) 10.2.12

Due to the exercise 10.2.11, we only need to show that IRn = (IR)n . Since any element of IRn is a finite sum Pmiof a(r1 , . . . , rn )
for ri ∈ R and a ∈ I, clearly, IRn ⊆ (IR)n . Now, let (a1 , . . . , an ) ∈ (IR)n be given. Then for each i, ai = k=1 aik rik . Then
observe that
m1 mn
! m1 mn
X X X X
(a1 , . . . , an ) = a1k r1k , . . . , ank rnk = a1k (r1k , 0, . . . , 0) + · · · + ank (0, . . . , 0, rnk ) ∈ IRn .
k=1 k=1 k=1 k=1
n n n n
Thus, (IR) ⊆ IR which prove that IR = (IR) . Then, by the previous exercise,
Rn /IRn = Rn /(IR)n = (R/IR)n .

Proof. (B. Ban) 10.2.13

Let n ∈ N be given. Since ϕ is surjective, we know there is m ∈ M such that ϕ(m+IM ) = n+IN . Since ϕ(m+IM ) = ϕ(m)+IN ,
Pf inite
we have ϕ(m) = n + i ai mi for some {ai } ⊆ I and {ni } ⊆ N . Then also note that, by the same reason, for each i, there is
Pf inite
mi ∈ M such that ni = ϕ(mi ) + k aik nik for some {aik } ⊆ I and {nik } ⊆ N . Then observe that
 0

fX
inite f inite fX
inite fX
inite
!
X X
ϕ(m) = n + ai ni = n + ai ϕ(mi ) +
 aik nik ⇐⇒ ϕ m −
 ai mi = n + ai aik nik ∈ N + I 2 N.
f inite i k i i,k
+
By the induction, we conclude that, for any n ∈ N and k ∈ Z , there exists m ∈ M such that
fX
inite
ϕ(m) = n + ai1 · · · aik ni1 ...ik ∈ N + I k N.
i1 ,dots,ik
+ J
Since I is idempotent, there is J ∈ Z such that I = 0. Thus, for any n ∈ N there is m ∈ M and α ∈ I J N = 0 such that
ϕ(m) = n + α = n which prove ϕ is surjective.


Page 93
MATH 611 Editor : Byeongho Ban

Proof. (B. Ban) 10.3.1

Let ιA : A ,→ F (A) and ιB : B ,→ F (B). Since A and B have same cardinality, there is an bijection ϕ : A → B. Now, by the
universal property of the free module, there are two unique R−module homomorphism ΦAP: F (A) → F (B) and ΦB : F (B) → F (A)
corresponding to the map ιB ◦ ϕ and ιA ◦ ϕ−1 respectively. Then observe that, for any i ri ai ∈ F (A),
! !
X X
ΦB ◦ ΦA ri ai = ΦB ri ΦA (ai ) ∵ ΦA : R−module homomorhpism
i i
!
X
= ΦB ri (ΦA ◦ ιA )(ai ) ∵ ai ∈ A
i
!
X
= ΦB ri (ιB ◦ ϕ)(ai ) ∵ ΦA ◦ ι A = ι B ◦ ϕ
i
!
X
= ΦB ri ϕ(ai ) ∵ ϕ(ai ) ∈ B
i
X
= ri ΦB (ϕ(ai )) ∵ ΦB : R−module homomorphism
i
X
= ri ΦB ◦ ιB (ϕ(ai )) ∵ ϕ(ai ) ∈ B
i
X
= ri ιA ◦ ϕ−1 (ϕ(ai )) ∵ ΦB ◦ ιB = ιA ◦ ϕ−1
i
X
= ri ai ∵ ai ∈ A
i
Thus, ΦB ◦ ΦA is an identity map. Since this calculation does not depend on the choice of A and B, with similar calculation
ΦA ◦ ΦB is also identity map which proves that F (A) ∼
= F (B).

Proof. (B. Ban) 10.3.2

Suppose that Rn ∼ = Rm . Let I be a maximal ideal of R. Let ϕ : Rn → Rm be the R−module isomorphism and ϕ : Rm /IRm →
R /IR be the induced homomorphism, i.e. ϕ((r1 , . . . , rn ) + IRn ) = ϕ(r1 , . . . , rn ) + IRm . First of all, ϕ is surjective since, for
n n

any (r1 , . . . , rm ) + IRm ∈ Rm /IRm we have


ϕ(ϕ−1 (r1 , . . . , rm ) + IRn ) = (r1 , . . . , rm ) + IRm .
Also, it is injective because
ker(ϕ) = {(r1 , . . . , rn ) + IRn : ϕ(r1 , . . . , rn ) + IRm = 0 + IRm }
= {(r1 , . . . , rn ) + IRn : ϕ(r1 , . . . , rn ) ∈ IRm }
( `
)
X
n m
= (r1 , . . . , rn ) + IR : ϕ(r1 , . . . , rn ) = ak rk , ak ∈ I, rk ∈ R
k=1
( `
! `
)
X X
n −1 −1 m
= (r1 , . . . , rn ) + IR : (r1 , . . . , rn ) = ϕ ak rk = ak ϕ (rk ), ak ∈ I, rk ∈ R
k=1 k=1
= 0 + IRn .
Thus, we have Rn /IRn =∼ Rm /IRm . Then, by the exercise 10.2.12, this implies that (R/I)n ∼
= (R/I)m since IR = I due to
commutativity of R. Since these are vector spaces over a field R/I, it should be n = m.

Conversely, if n = m, clearly Rn = Rm , so Rn ∼
= Rm .


Page 94
MATH 611 Editor : Byeongho Ban

Proof. (B. Ban) 10.3.5

Let M be the finitely generated torsion R−module. Then any m ∈ M has the form
Xn
m= ri gi
i=1
where ri ∈ R, gi is a generator, and n ∈ N. Since M is a torsion module, there is ai ∈ R such that ai gi = 0 ∀i. Consider
n
Y
p= ai .
i=1
Observe that p is non zero since R is an integral domain and ai 6= 0 ∀i. Then, for an any m ∈ M , observe that
 
Xn Y
pm = ri gi ai  = 0.
i=1 n6=i

Thus, p is the non zero annihilator.



Proof. (B. Ban) 10.3.9

Suppose that M is irreducible, then M 6= 0. Let m ∈ M \ {0} be given. Then sine 1 ∈ R, Rm is nonzero submodule of M , so
M = Rm. It implies that M is cyclic.

Conversely, suppose that M 6= 0 and M = Rm ∀m ∈ M \ {0}. Suppose that N be a nonzero submodule of M . Then, ∃n ∈ N \ {0}.
However, M = Rn ⊆ N ⊆ M , so we have N = M .

From the proof, we can conclude that every irreducible Z−module is isomorphic to Zp for some prime p.

Proof. (B. Ban) 10.3.10

Suppose that M is irreducible. Then consider the natural map ϕ : R → M = Rm defined by ϕ(r) = rm for some fixed
m ∈ M \ {0}. Clearly, ϕ is a group homomorphism by the axioms of module. Thus, we have M ∼ = R/ker(ϕ). Suppose that there
is an ideal, J, of R containing ker(ϕ), then it is a submodule of R. Then, by the fourth module isomorphism theorem, J/ker(ϕ)
is a submodule of R/ker(ϕ). Then, by the isomorphism, M contains a submodule, N , isomorphic to J/ker(ϕ). Since M have
only 0 as a proper submodule and N contains ker(ϕ), N ∼ = M or N ∼ = 0. If N ∼
= M , then R/ker(ϕ) ∼
=M ∼ = J/ker(ϕ) so R = J.
If N ∼= 0, then J/ker(ϕ) ∼= 0 so J = ker(ϕ). Therefore, ker(ϕ) is the maximal ideal I we were searching for.

Conversely, suppose that M is an isomorphic to R/I where I is a maximal ideal of R. Assume that N is a nonzero submodule of
M . Then, by the isomorphism, R/I also has a nonzero submodule K. Since R is commutative, the nonzero submodule should be
an ideal of R/I. But, by the fourth ring isomorphism theorem, K = J/I for some ideal J ⊆ R. However, since I is maximal and
K is nonzero, J = R which implies N = M . Therefore, M is irreducible.

Proof. (B. Ban) 10.3.11

Let ϕ : M1 → M2 be a given nonzero homomorphism. Assume that ∃m0 ∈ ker(ϕ) \ {0}. Then we can say M = Rm0 . Then,
∀m ∈ M ∃r ∈ R such that rm0 = m, so ϕ(m) = ϕ(rm0 ) = rϕ(m0 ) = 0. It implies that ϕ(m) = 0 ∀m ∈ M . And it is a
contradiction to our assumption that ϕ is nonzero homomorphism. Therefore, ϕ is injective.

Let m ∈ M2 be given. Note that ∃m00 ∈ M2 \ {0} ∃m0 ∈ M1 \ {0} such that ϕ(m0 ) = m00 since ϕ is nonzero homomorphism. Note
that M2 is irreducible so M2 = Rm00 . Then ∃r ∈ R such that m = rm00 . Now, observe that
m = rm00 = rϕ(m0 ) = ϕ(rm0 ).
Thus, m ∈ ϕ(M1 ), so ϕ is surjective. Therefore, ϕ is an isomorphism.

Let M be a given irreducible module and consider EndR (M ). Clearly, EndR (M ) is a ring where addition is defined by pointwise
addition and multiplication is defined by function composition. Let φ ∈ EndR (M ) \ {0}. Then, by what we have proved, φ is an
isomorphism, so is a unit. Therefore, every nonzero elements are units so EndR (M ) is a division ring.


Page 95
MATH 611 Editor : Byeongho Ban

Proof. (B. Ban) 10.3.18(fixing)

∼ M . Since (pαi )M and (pαj )M are comaximal ∀i, j


Note that (a)M = 0 since M is annihilated by (a). Thus, we have M/(a)M = i j
with i 6= j, by Chinese Remainder Theorem,
∼ M/(a)M ∼ = M/(pα α2 αk
M= 1 )M × M/(p2 )M × · · · × M/(pk )M.
1

Let (m1 , m2 , . . . , mk ) be a given element of right hand side. Suppose pα αi αi


i m = 0 for some i. Then pi mj = 0 ∀j. Observe that (pi )
i

αj αi αi
and (pj ) are comaximal. Thus, mj = 0 for j 6= i. It implies that pi primary component of the right hand side is M/(pi )M .
The isomorphism obtained from the Chineses Remainder Theorem thus gives a direct sum decomposition into its pα i −primary
i

components. But by the definition,


M = M1 ⊕ M2 ⊕ · · · ⊕ Mk .

Proof. (B. Ban) 10.4.3

Note that C ⊗R C is a C−module extended from R−midule C and C ⊗C C is a C−module extended from C module C. Thus,
naturally, both of them are R−module since R ⊆ C, so R acts on them.

Observe that
4 4
C ⊗R C ∼
= R2 ⊗ R R2 ∼
= (R ⊕ R) ⊗R (R ⊕ R) ∼ (R ⊕R R) ∼ R∼
M M
= = = R4
i=1 i=1
C ⊗C C ∼
=C∼ = R2 .
Since R2 ∼ ∼ C ⊗C C.
6 R4 , we should have C ⊗R C 6=
=

Proof. (B. Ban) 10.4.4

Note that Q ⊗Z Q and Q ⊗Q Q are vector spaces over Q. Firstly, observe that Q ⊗Q Q ∼
= Q, so it is of dimension 1. Further note
n
that, for any q ⊗Z p ∈ Q ⊗Z Q with p = m and q = ab of n, m, a, b ∈ Z,
a n 1 1 m 1 1 m an 1 an
q⊗p= ⊗ = an( ⊗ ) = an( ⊗ ) = an( ⊗ )= bm( ⊗ 1) = 1 ⊗ 1.
b m b m bm m bm m bm bm bm
Therefore, any element in Q ⊗Z Q can be obtained by multiplying rational number to 1 ⊗ 1, so it is also of dimension 1. Since
Q ⊗Z Q is of dimension 1 over Q, it is isomorphic to Q. Therefore,
Q ⊗Z Q ∼
=Q∼ = Q ⊗Q Q.

Proof. (B. Ban) 10.4.5
Ql
Note that ∃{pi }li=1 such that n = i=1 pαi . Without loss of generality, let p1 = p. Now, let Gi ≤ A be the Sylow pi −subgroup.
i

Since A is abelian group, Gi E A, so each Gi is unique Sylow pi −subgroup. Thus, we have that
l
A∼
M
= Gi .
i=1
Pr
And, also, note that ∀i ∃{βji }rj=1
i
such that j=1 βji = αi and
ri
βi
Gi ∼
M
= Z/pi j Z.
j=1

Since, for any i 6= 1,


ri  
βi
Z/pk1 ⊗Z Gi ∼ Z/pk1 Z ⊗ Z/pi j Z ∼
M
= = 0,
j=1

we have
l r1   Mr1
1 1
Z/pk Z ⊗Z A ∼ (Z/pk Z ⊗ Gi ) ∼ Z/pk Z ⊗Z Z/pβj Z ∼ Z/pβj Z ∼
M M
= = Z/pk Z ⊗ G1 = = = G1 .
i=1 j=1 j=1

Here, note that we have used the property


Z/nZ ⊗ Z/mZ ∼
= Z/dZ
where d = gcd(n, m). Thus, Z/pk Z ⊗Z A ∼
= G1 where G1 is the Sylow p−subgroup.


Page 96
MATH 611 Editor : Byeongho Ban

Proof. (B. Ban) 10.4.11

Assume that
e1 ⊗ e2 + e2 ⊗ e1 = v ⊗ w
for some v, w ∈ R2 \ {0}. Then, ∃a, b, c, d ∈ R such that
v = ae1 + be2 w = ce1 + de4 .
Observe that
v ⊗ w = (ae1 + be2 ) ⊗ (ce1 + de2 ) = ac(e1 ⊗ e1 ) + ad(e1 ⊗ e2 ) + bc(e2 ⊗ e1 ) + bd(e2 ⊗ e2 ) = e1 ⊗ e2 + e2 × e1 .
Since {ei ⊗ ej : i, j ∈ {1, 2}} is a basis of R2 ⊗ R2 , we should have ac = 0 = bd and ad = 1 = bc. However, it is inconsistent since
ac = 0 implies, a = 0 or c = 0, but then 1 = ad = 0 or 1 = bc = 0. Thus, it is a contradiction and e1 ⊗ e2 + e2 ⊗ e1 can not be
written as a simple tensor.

Proof. (B. Ban) 10.4.12

Suppose that v 0 = av for some a ∈ F , observe that


v ⊗ v 0 = v ⊗ av = a(v ⊗ v) = (av) ⊗ v = v 0 ⊗ v.
Thus, v ⊗ v 0 = v 0 ⊗ v.

0 0 0 0
Conversely,
P P v 0⊗ v = v ⊗ v and v 6= av ∀a ∈ F . Let B = {ei } be the basis of V . Then, ∃{ai }, {ai } ⊆ F such that
assume that
0
v = i ai ei and v = i ai ei . Then observe that
X X
v ⊗ v0 = v ⊗ a0i ei = a0i v ⊗ ei
i i
X X
0 0
v ⊗v =v ⊗ ai ei = ai v 0 ⊗ ei
i i
Since v ⊗ v 0 = v 0 ⊗ v, observe that
a0i v ⊗ ei = ai v 0 ⊗ ei ⇐⇒ (a0i v − ai v 0 ) ⊗ ei = 0 ∀i ∈ Z+ .
Since v and v 0 are linearly independent, we should have ei = 0 and it is a contradiction. Therefore, ∃a ∈ F such that v 0 = av.


Page 97
MATH 611 Editor : Byeongho Ban

Proof. (B. Ban) 10.4.21

(a) Show there is a surjective R−module homomorphism from I ⊗R J to the product ideal IJ mapping i ⊗ j to the element ij.

( =⇒ ) Note that IJ is an ideal of R, so IJ,I, and J are R−modules. Consider ϕ : I × J → IJ defined by ϕ(i, j) = ij. Observe
that, for any i1 , i2 ∈ I, j1 , j2 ∈ J and r ∈ R,
ϕ(i1 + i2 , j1 ) = (i1 + i2 )j2 = i1 j1 + i1 j2 = ϕ(i1 , j1 ) + ϕ(i2 , j1 )
ϕ(i1 , j1 + j2 ) = i1 (j1 + j2 ) = i1 j1 + i1 j2 = ϕ(i1 , j1 ) + ϕ(i1 , j2 )
ϕ(ri1 , j1 ) = rij = rϕ(i, j) = rij = ϕ(i, rj).
Thus, ϕ is R−bilinear map, so there is a R−module homomorphism Φ : I ⊗ J → IJ such that Φ ◦ ι = ϕ where ι(i, j) = i ⊗ j.
Thus, note that, for any i ∈ I and j ∈ J,
ij = ϕ(i, j) = Φ(ι(i, j)) = Φ(i ⊗ j).
Now, let s ∈ IJ be given. And note that, ∃{ik } ⊆ I and {jk } ⊆ J such that
Xn
s= ik jk .
k=1
Pn
Observe that k=1 ik ⊗ jk ∈ I ⊗ J and
n
! n n
X X X
Φ ik ⊗ jk = Φ(ik ⊗ jk ) = ik jk = s.
k=1 k=1 k=1
Therefore, Φ is a surjective R−module homomorphism mapping i ⊗ j to ij.

(b) Give an example to show that the map in (a) need not be injective.
( =⇒ ) Consider R = Z[x] and the ideal I = (2, x). And consider ϕ : I × I → Z/2Z defined by
n m
!
X
i
X
i a0
ϕ ai x , bi x = b1 mod2.
i=1 i=1
2
Observe that, for any finite subsets {ai }, {bi }, {ci } ⊆ Z and z1 , z2 ∈ Z,
!
X X X z1 a0 + z2 c0
ϕ z1 ai xi + z2 ci x i , bi xi = b1 mod2
i i i
2
z1 a0 z2 c0
= b1 mod2 + b1 mod2
2 2 ! !
X X X X
= z1 ϕ ai xi , bi xi + z2 ϕ ci x i , bi x i
i i i i
and that
!
X
i
X
i
X
i a0
ϕ ai x , z1 ci x + z2 bi x = (z1 c1 + z2 b1 ) mod2
i i i
2
a0 a0
= z1 c1 mod2 + z2 b1 mod2
2 2 ! !
X X X X
i i i i
= z1 ϕ ai x , ci x + z2 ϕ ai x , bi x .
i i i i
Thus, ϕ is a R−bilinear map. Thus, ∃R − module homomorphism Φ : I ⊗ I → Z/2Z such that ϕ = Φ ◦ ι where ι(p(x), q(z)) =
p(x)⊗q(x). Then, by (a), Φ is surjective. However, note that (2+x)⊗(2+x) 6= 2⊗x but Φ((2+x)⊗(2+x)) = Φ(ι(2+x, 2+x)) =
ϕ(2 + x, 2 + x) = 1 mod 2 = ϕ(2, x) = Φ(ι(2, x)) = Φ(2 ⊗ x). Therefore, Φ is not an injection. 

Page 98
MATH 611 Editor : Byeongho Ban

Proof. (B. Ban) 10.4.24


Let I : R → C defined by I(r) = r. Clearly, it is a Z−module homomorphism. Then, by the universal property,
∃ a Z[i]−module homomorhpismΦ : Z[i] ⊗Z R → C such that I = Φ ◦ ι where ι : R → Z[i] ⊗Z R defined by ι(r) = 1 ⊗ r.
Note that I is injection, so ι is also an injection and 1 ⊗ r 6= 1 ⊗ s iff s 6= r. Let (a + bi) ⊗ c ∈ Z[i] ⊗Z R be given such that
Φ ((a + bi) ⊗ c) = 0. Observe that
Φ((a + bi) ⊗ c) = Φ(1 ⊗ ac) + iΦ(1 ⊗ bc) = I(ac) + iI(bc) = ac + ibc.
Thus, Φ((a + bi) ⊗ c) = 0 ⇐⇒ c = 0 or a + bi = 0 ⇐⇒ (a + bi) ⊗ c = 0. Thus, kerΦ = 0 and Φ is an injection. Also, for given
a + bi ∈ C, note that (a + bi) ⊗ 1 ∈ Z[i] ⊗Z R and that
Φ((a + bi) ⊗ 1) = Φ(a ⊗ 1) + iΦ(b ⊗ 1) = aΦ(1 ⊗ 1) + ibΦ(1 ⊗ 1) = a + bi.
Thus, Φ : Z[i] ⊗Z R is a bijection and thus it is isomorphism. Therefore,
Z[i] ⊗Z R ∼
= C.

Proof. (B. Ban) 11.5.1

Let M =< a > and let (m1 ⊗ m2 ⊗ · · · mt ), (m01 ⊗ m02 ⊗ · · · ⊗ m0s ) ∈ T (M ) be given. Then for any i and j ∃{ri }, {rj0 } ⊆ R such
that mi = ri a and m0j = rj0 a. Now, observe that
(m1 ⊗ m2 ⊗ · · · mt ) ⊗ (m01 ⊗ m02 ⊗ · · · ⊗ m0s ) = r1 a ⊗ r2 a ⊗ · · · rt a ⊗ r10 a ⊗ r20 a ⊗ · · · ⊗ rs0 a
= (r1 · · · rt r10 · · · rs0 )a ⊗ a ⊗ · · · a ⊗ a ⊗ a ⊗ · · · ⊗ a
= r10 a ⊗ r20 a ⊗ · · · rs0 a ⊗ r1 a ⊗ r2 a ⊗ · · · ⊗ rt a
= (m01 ⊗ m02 ⊗ · · · ⊗ m0s ) ⊗ (m1 ⊗ m2 ⊗ · · · mt ).
Therefore, T (M ) is commutative, so T (M ) = S(M ).

Proof. (B. Ban) 11.5.2

Consider

M ∞
M
ϕ:S= Sk → (Sk /Ik )
k=0 k=0
(. . . , sk , . . . ) 7→ (. . . , sk mod Ik , . . . )
where Ik = I ∩ Sk .

Firstly, we show that Si Ij ⊆ Ii+j . Note that Si Ij = Si (Sj ∩ I). Let si ∈ Si and sj ∈ Sj ∩ I be given. Observe that si sj ∈ Si+j
since Si Sj ⊆ Si+j and that si sk ∈ I since I is an ideal and sk ∈ I. Thus, Si Ij = Si (Sj ∩ I) ⊆ (Si+j ) ∩ I = Ii+j .

Then we show that the multiplication (Si /Ii )(Sj /Ij ) ⊆ Si+j /Ii+j is well-defined.
Observe that, be the argument above,
(Si /Ii )(Sj /Ij ) = (Si + Ii )(Sj + Ij ) = Si Sj + Si Ij + Ij Si + Ii Ij ⊆ Si+j + Ii+j + Ii+j + Ii+j = Si+j + Ii+j = Si+j /Ii+j .
And, the multiplication is clearly well defined.

Now, observe that



M
ϕ(s1 , s2 , . . . ) = 0 ⇐⇒ sk ∈ Ik ∀k ⇐⇒ (s1 , s2 , . . . ) ∈ Ik .
i=0
L∞
Thus, ker(ϕ) = i=0 Ik . And observe that, for any (. . . , sk mod Ik , . . . ), ϕ(. . . , sk , . . . ) = (. . . , sk mod Ik , . . . ), so ϕ is surjective.
Therefore,
∞ ∞ ∞
! !
(Sk /Ik ) ∼
M M M
= S/ker(ϕ) = Sk / Ik = S/I.
k=0 k=0 i=0
Thus, S/I is a graded ideal.


Page 99
MATH 611 Editor : Byeongho Ban

Proof. (B. Ban) 11.5.8


(a)
Let α, β ∈ F \ {0} be given. Then ∃r, r0 , s, s0 ∈ R such that α = r/r0 and β = s/s0 . Then observe that
r s 1 r s 1  r s 1 r s
α ∧ β = 0 ∧ 0 = 0 0 r0 s0 0 ∧ 0 = 0 0 r0 0 ∧ s0 0 = 0 0 (r ∧ s) = 0 0 (1 ∧ 1) = 0.
r s rs r s rs r s rs r s
Thus, Λ2 F = 0.

(b)
Let n ∈ Z ∩ [2, ∞) be given. And let f1 ∧ f2 ∧ · · · ∧ fn ∈ Λn I be given. Then, since fi ∈ I for each i, ∀i ∈ {1, 2, . . . , n} ∃ri , ri0 ∈ R
such that fi = rri0 . Then observe that
i

(r1 · · · rn )(f1 ∧ · · · ∧ fn ) = ((r1 f1 ) ∧ · · · ∧ (rn fn )) = r1 ∧ · · · rn = (r1 ∧ · · · rn )(1 ∧ · · · ∧ 1) = 0.


Therefore, Λi I is a torsion R−module.

(c) Qn
Let R = Z[x, y] and I = (x, y). Trivially, Λ1 I 6= 0. Let i ∈ Z ∩ [2, ∞) be given. Consider ϕ : i=1 (x, y) → Z[x, y]/(x, y) defined by
ϕ(a1 x + b1 y, . . . , an x + bn y) = (a1 b2 − a2 b1 )(a2 b3 − a3 b2 ) · · · (an−1 bn − an bn−1 ) mod (x, y).
This map is clearly R−multilinear and alternating map. Note that
ϕ(2x + y, x + 2y, 2x + y, x + 2y, . . . , 2x + y, x + 2y) = (−1)N/2 3n−1 6= 0
where N = n if n is even, N = n − 1 if n is odd. Thus, ϕ is nontrivial map so Λn (x, y) 6= 0.

Proof. (B. Ban) 12.1.2

(a) Pn Pn
Let {ei }ni=1 be a standard basis of Rn . Consider the map ϕ : Rn → N defined by ϕ( i=1 ai ei ) = i=1 ai xi . Note that
n n
! n n n n
! n
!
X X X X X X X
ϕ r ai ei + s bi e i = (rai + sbi )xi = r ai xi + s bi xi = rϕ ai ei + sϕ bi e i .
i=1 i=1 i=1 i=1 i=1 i=1 i=1
Thus, ϕ is R−linear map and R−module homomorphism. Let {qi }ni=1 ⊆ R be given and observe that
n
! n
X X
ϕ qi ei = q i xi ,
i=1 i=1
so ϕ is surjective. Also, observe that
n
! n n
X X X
ϕ ai ei = 0 ⇐⇒ ai xi = 0 ⇐⇒ ai = 0 ∀i ⇐⇒ ai ei = 0.
i=1 i=1 i=1
Thus, ker(ϕ) = 0. Therefore, ϕ is a R−module isomorphism between N and Rn , so N ∼
= Rn .

Let y ∈ M \ {0} be given. Then, since x1 , x2 , . . . , xn , x are linearly dependent, ∃ a nontrivial subset{ai , r}ni=1 ⊆ R such that
Xn
ai xi + rx = 0.
i=1
Since {xi }ni=1 is linearly independent and {ai }ni=1 6= {0}, r 6= 0. Then observe that
ry = −a1 x1 + · · · + an xn .

Proof. (B. Ban) 12.1.2(b)

Let {yi }n+1 n+1


i=1 ⊆ M \ {0} be given. Since M/N is torsion, ∀i ∈ {1, 2, . . . , n + 1} ∃ri ∈ R such that ri yi ∈ N . Note that {ri yi }i=1
n+1
is linearly dependent since N is free of rank n. Thus, we have linearly dependence set {yi }i=1 since R is an integral domain so
there is no zero divisor. Thus, the rank of M is smaller than n + 1. Also, since N ⊆ M , rank of M is bigger than n. Therefore,
the rank of M is n.


Page 100
MATH 611 Editor : Byeongho Ban

Proof. (B. Ban) 12.1.4

Let A1 = {xi }si=1 ⊆ M be given such that its image in M/N is the maximal linearly independent set. And let A2 = {xi }s+r
i=s+1 be
the maximal linearly independent set of N . Firstly, note that A1 ∩ A2 = ∅. Also, A1 ∩ N = ∅ and no element from A2 is in M/N
since if the image of xi in M/N is zero iff xi ∈ N .
Assume that {ri } ⊆ R be non trivial set such that
s+r
X
t= ri xi = 0.
i=1
Note that under the quotient map, the image of t satisfies
s
X
ri xi = 0.
i=1
Since {xi }si=1 is linearly independent in M/N , ri = 0 ∀i ∈ {1, 2, . . . , s}. Then
s+r
X
t= ri xi = 0.
i=s+1

However, since {xi }s+r


i=s+1 is linearly independent in N , ri = 0 ∀i ∈ {s + 1, s + 2, . . . , s + r}. Therefore, if t = 0, the only choice of
{ri }i=1 is {0} and it is a contradiction to our assumption. So, it implies that {xi }s+r
s+r s+r
i=1 is linearly indepentdent. Thus, {xi }i=1 is
a basis of M , n = dim(M ) = s + r.

Proof. (B. Ban) 12.1.6

Clearly, M is torsion free since M is an ideal of an integral domain R. Let m ∈ M \ {0} be given. And note that rm = 0 if and
only if r = 0 since R is integral domain and M ⊆ R. Then {m} is linearly independent. Further suppose that n ∈ M \ {0, m} be
given. Then, observe that
(−n)m + (m)n = 0.
Thus, any two element in M is linearly dependent. Therefore, M is torsion free of rank 1.

Assume that M is free, then M ∼ = R. Then, observe that , for given m ∈ M \ {0}, < m >= Rm = M . However it is contradiction
since M is non principal ideal. Therefore, M cannot be free.

Proof. (B. Ban) 12.1.9
Q∞
Let R = Z and M = i=1 Z/pi Z where {pi }i∈Z+ is a sequence of distinct prime numbers. Then for any m ∈ M ,
m = (q1 , q2 , · · · , qn , 0, 0, · · · ). Let q = p1 p2 · · · pn , then qm = 0. Thus, M is torsion Z−module. However, Ann(M ) = 0.
αk
Otherwise, ∃r ∈ Ann(M ) \ {0},so rm = 0 ∀m ∈ M . Note that r = pα 1 · · · pk . So, r(0, . . . , 0, qk+1 , 0, 0, 0, . . . , ) 6= 0 and it is a
1

contradiction . Therefore, Ann(M ) = 0.

Suppose that N is finitely generated torsion R−module. Then


N∼ α
= Rr ⊕ R/(pα1 ) ⊕ · · · R/(p k )
1 k
αk
where r ≤ 0 and pα
1 , . . . , pk
1
are positive powers of primes pi in R. Since N is torsion, we should have r = 0. For example,
αk
otherwise, note that 6 ∃r ∈ R such that r(1, 0, 0, . . . ) = 0 where (1, 0, 0 . . . ) ∈ Rr ⊕ R/(pα
1 ) ⊕ · · · ⊕ R/(pk ). Therefore,
1

N∼ α
= R/(pα1 ) ⊕ · · · R/(p k ).
1 k
αk
Then, let q = pα
1 · · · pk . Then q 6= 0 and qm = 0 ∀m ∈ N . Therefore, Ann(N ) 6= 0.
1

Page 101
MATH 611 Editor : Byeongho Ban

Proof. (B. Ban) 12.2.3

Suppose that two non scalar 2 × 2 matrices A and B are similar, then A = S −1 BS for some invertible matrix S. Then observe
that the characteristic polynomial is
det(λI − A) = det(λS −1 S − S −1 BS) = det(S −1 (λI − B)S) = det(S −1 )det(λI − B)det(S) = det(λI − B).
Thus, the characteristic polynomial for A and B are same.

Conversely, suppose that two non scalar matrices A and B have same characteristic polynomial, p(x). Note that deg(p) = 2.
Firstly, suppose that p(x) = x2 + ax + b is irreducible. Then the only possible rational canonical form would be
 
0 −b
T =
1 −a
so A and B are both similar to T and so A and B are similar. Secondly, suppose that p is reducible, so p(x) = (x − a)(x − b)
for some a, b ∈ F . If a 6= b, then invariant factor of the linear transformation corresponding to p would have just one companion
matrix so A and B are similar again. If a = b, then, letting p(x) = (x − a)2 , there are tow cases when the invariant factors are
(x − a) and (x − a) or when that is just (x − a)2 . In the first case, the rational canonical form would be
 
a 0
R=
0 a
And it is a scalar matrix. If one of A and B is similar to R, for example, if A ∼ R, then A = P RP −1 for some invertible matrix
P . However, then note that
A = P RP −1 = P (aI)P −1 = a(P IP −1) = a(P P −1 ) = aI
 

which shows that A is scalar matrix which is contradiction to the assumption that neither A nor B is scalar matrix. Therefore,
A and B should have one invariant factor (x − a)2 which implies
0 −a2
 
A∼ ∼B
1 2a
Therefore, in any cases, if neither of A nor B is scalar matrix, and if cA = cB then A ∼ B.

Proof. (B. Ban) 12.2.4

Suppose that A and B are similar 3 × 3 matrices. Then ∃ an invertible matrixP such that A = P BP −1 . Then observe that
det(λI − A) = det(P (λI − B)P −1 ) = det(P )det(λI − B)det(P ) = det(λI − B)
since det(p−1 ) = 1/ det(P ). Thus, the characteristic polynomials of A and B are same. Also, those of minimal polynomials are
same since A and B have same Smith normal form.

Conversely suppose that two 3 × 3 matrices have some character and minimal polynomials. Let the characteristic polynomial p(x)
and the minimal polynomial m(x).
Note that, if deg(m(x)) = 3, then we have p(x) = m(x). Then the only possible rational canonical form of any linear transformation
corresponding to p(x) is a companion matrix of m(x), let R. Thus, A ∼ R ∼ B. If deg(m(x)) = 2, then p(x)/m(x) is linear.
In this case, the only possible rational canonical form corresponding to p(x) and m(x) would be direct sum of rational canonical
form of p/m and m(x) which implies A and B should be similar. When deg(m(x)) = 1, the only possible rational canonical form
of the linear transformation corresponding to p(x) and m(x) = x − a would be scalar matrix aI which restricts to the case when
A ∼ B.
In all cases, the invariant factors are determined by the minimal and characteristic polynomials. Therefore, A and B have same
invariant factors, so have same rational canonical form. It implies that A and B are similar.

Consider two matrices


   
0 1 0 0 0 1 0 0
0 0 0 0 0 0 0 0
A=
0
 B= 
0 0 1 0 0 0 0
0 0 0 0 0 0 0 0
Observe that the characteristic polynomial is pA (x) = x4 = pB (x). Further observe that A2 = B 2 = 0, so the minimal polynomials
are mA (x) = x2 = mB (x). Thus, the two matrices have same characteristic and minimal polynomial. However, they are not
similar since they are already Jordan canonical form and they are different.


Page 102
MATH 611 Editor : Byeongho Ban

Proof. (B. Ban) 12.2.9

Let
 
    422 465 15 −30
0 −1 −1 c 0 −1 −420 −463 −15 30 
A= 0 0 0 B= 0 c 1 C= 
 840 930 32 −60
−1 0 0 −1 1 c
−140 −155 −5 12
Observe that
 √  √ 
cA (x) = det(xI − A) = x(x2 − 1) cB (x) = det(xI − B) = (x − c) x − (c − 2) (x − (c + 2)
Note both cA and cB has three different linear factors. Thus, x3 − x is the minimal polynomial. Thus, the rational canonical form
of A is
 
0 0 0
RCF (A) = 1 0 1 .
0 1 0
and taht of B is

c3 − 2c
 
0 0
RCF (B) = 1 0 −3c2 + 2
0 1 3c
where det(xI − B) = x3 − 3cx2 + (3c2 − 2)x − c3 + 2c.

Now, observe that


cC (x) = det(xI − C) = x4 − 3x3 − 6x2 + 28x − 24 = (x − 2)3 (x + 3).
Further observe that
(C − 2I)(C + 3I) = 0.
Thus, (x − 2)(x + 3) is the minimal polynomial. So there is only one possibility that the invariant factors are
(x − 2), (x − 2), (x − 2)(x + 3).
Therefore,
 
2 0 0 0
0 2 0 0
RCF (C) = 
0
.
0 0 6
0 0 1 −1

Proof. (B. Ban) 12.2.11

Note that we can characterize the similarity class of the 6 × 6 matrices by their minimal polynomial based on their given
characteristic polynomial. Note that
(x4 − 1)(x2 − 1) = (x2 + 1)(x2 − 1)(x2 − 1) = (x + i)(x − i)(x − 1)2 (x + 1)2 .
Then we have four different cases.
1. When m(x) = (x + i)(x − i)(x − 1)2 (x + 1)2 , the invariant factors are {(x + i)(x − i)(x − 1)2 (x + 1)2 }.

2. When m(x) = (x + i)(x − i)(x − 1)(x + 1)2 , the invariant factors are {(x + i)(x − i)(x − 1)(x + 1)2 , (x − 1)}

3. When m(x) = (x + i)(x − i)(x − 1)2 (x + 1), the invariant factors are {(x + i)(x − i)(x − 1)2 (x + 1), (x + 1)}.

4. When m(x) = (x + i)(x − i)(x − 1)(x + 1), the invariant factors are {(x + i)(x − i)(x − 1)(x + 1), (x + 1)(x − 1)}.
These four cases specifies all possible simiarity of the given 3 × 3 matrix with the characteristic polynomial.


Page 103

You might also like